Sei sulla pagina 1di 385

Pathoma Lecture Notes

Chapter 1: Growth Adaptations,


Cellular Injury, and Cell Death
1.1 Growth adaptations

Hyperplasia Metaplasia Dysplasia


- Increase in cell number - Change in cell type due to - Proliferation of pre-
reprogramming of stem cell cancerous cells
- Reversible - Reversible - Reversible
- Pathologic hyperplasia - Can progress to dysplasia - Can progress to cancer
can progress to cancer and cancer
- BPH don't increase risk - Apocrine metaplasia don't
of cancer increase risk of breast cancer
- Etiology: - Etiology: - Etiology:
• Physiologic (ex- • Vit A deficiency • Long standing
pregnancy) (keratomalacia, pathologic hyperplasia
• Pathologic (ex - …) myositis ossificans) or metaplaisa
• Cellular stress (barret's
esophagus)

Processes Properties
Hyperplasia • Usually, hyperplasia and hypertrophy occur together
and • Permanent cell undergro hypertrophy only – cardiac myocytes, skeletal
Hypertrophy cells, and nerves.
• Pathologic hyperplasia (not physiologic ex- pregnancy) can progress to
dysplasia and cancer.
• Exception is benign prostatic hyperplasia (is pathologic but does not
increase risk of cancer).
Atrophy • can occur by decrease in cell number (apoptosis) or decrease in cell size.
• Decrease in cell size occurs by – ubiquitin-proteosome degradation of
cytoskeleton and autophagy of cellular components
Metaplasia • change in cell type.
• Mostly affects surface epithelium. Ex – barret’s esophagus (change from
squamous to columnar)
• MOA: occurs by reporgramming of stem cells
• Reversible
• Can progress to dysplasia and cancer. Ex – Barret’s. Exception –
Apocrine metaplasia doesn’t increase risk of breast cancer. Apocrine
metaplasia seen when fibrocystic changes in breast occur.
• Vitamin A deficiency can result in metaplasia. Ex – keratomalacia and
myositis ossificans.
• Mesenchymal tissue can undergo metaplasia (ex – bone, blood vessel,
fat etc). ex – myositis ossificans – inflammation of skeletal muscle reads
to metaplasia to bony tissue.

Cellular injury Page 1.1


Fig – myositis ossificans
Dysplasia • disordered cell growth; proliferation of precancerous cells. Ex - Cervical
intraepithelial neoplasia
• Arises from longstanding pathologic hyperplasia or metaplasia
• Reversible
• If dysplasia persists, it can progress to cancer.

Aplasia • Failure of cell production during embryogenesis. Ex – unilateral renal


agenesis (failure to make 1 kidney)
Hypoplasia • decrease in cell production in embryogenesis. Results in relatively small
organ. Ex – streak ovary in Turner syndrome.

Extras – Vitamin A is necessary for maturation of immune system. 15-17 translocation in


pt genome causes acute promyelocytic leukemia. The translocation causes mutation in vit-
A receptor – retinoic acid receptor. Mutation of receptors cause immune cells to be
trapped in blast stage. Treatment for Promyelocytic leukemia is all trans retinoic acid
(drug) which can bind to mutated receptor. Cells can mature and be neutrophil.
• Vitamin A is necessary for maintainence of special epithelial tissue in body – ex –
conjucitva of eye. Conjuctiva is a thin delicate membrane that covers eye. Deficiency
of vit A can result in metaplasia of cells in conjucitva where cells thicken which is
called keratomalacia.

• Fig – keratomalacia.

Cellular injury Page 1.2


1.2 Cellular injury

1. What is cellular injury?


- If cellular stress overcomes cell's ability to adapt, then cell gets injured.

2. What does cellular injury depend on?


- Type of stress
- Severity
- Type of cell - neurons can withstand hypoxia only for 3-5mins. Skeletal muscle can withstand for a
very long time.

3. What are causes of hypoxia(low O2 delivery to tissue)?


- ischemia,
- hypoxemia (PaO2<60 mm Hg),
- low O2 carrying capacity (ex - anaemia, CO poisoning, methmeglobinemia)

4. What is shock? What are it's causes?


- Shock is a generalized decreased perfusion of a vital organ. Can occur by hypovolemia,
cardiogenic, spetic, neurogenic, anaphylactic causes

5. What is Budd chiari syndrome?


- Blockade of hepatic vein can result in hepatic infraction, aka budd chiari syndrome.
- Most common cause of budd chiari syndrome is polycythemia vera. Polycythemia vera is a disease
with too much RBC which leads to clot formation. Another cause is lupus.

6. What will PaO2 and O2 sat be in anaemia?


- PaO2 and O2 sat will be normal in anaemia

7. What will PaO2 and O2 sat be in CO poisoning?


- PaO2 will be the same and O2 sat will reduce. CO binds to Hb 100 times stronger than O2.

8. What are signs/symptoms of CO poisoning?


- Early sign is headache (ask a pt if he has headache if you suspect CO poisoning). Pt may have
cherry red color skin.

9. What is methemoglobinemia?
- Normally, Fe in hemoglobin is in 2+ state and can bind to O2. If it gets oxidized to 3+, it can't bind
to O2 and this is methemoglobinemia.

10. What will PaO2 and O2 sat be in methemoglobinemia?


- PaO2 will be the same and O2 sat will reduce.

11. What are causes of methemoglobinemia?


- Oxidative stress - sulfa drugs, nitrate drugs. Also seen in newborns because their machinery to
reduce Fe 3+ isn't good.

12. What are signs/symptoms of methemoglobinemia?


- Cyanosis with chocolate colored blood.

13. What is treatment of methemoglobinemia?

Cellular injury Page 2.1


13. What is treatment of methemoglobinemia?
- IV methylene blue - it generates moderators that will reduce Fe3+

14. What are consequences of low ATP in cell?


- Lactic acidosis due to lots of glycolysis
- Disrupted Na/K pump result in Na accumulation and resulting water retention in cell
- Disrupted Ca pump result in Ca accumulation and random enzyme activation in cytosol

15. What are reversible finding in cell of low ATP?


- Hallmark is cellular swelling - loss of microvilli, membrane blebing as it pulls away from the
cytoskeleton, swelling of RER and ribosomes fall off (low protein synthesis)

16. What are irreversible finding in cell of low ATP?


- Hallmark is membrane damage. End result is cell death.
- Cellular enzymes leak out (liver enzyme in hepatitis and cardiac enzyme in MI) and intracellular Ca
increases.
- Cytochrome C from mitochondria leaks out to cytosol and activates apoptosis
- Lysozome enzymes will leak out and digest the cells. Ca in cytosol activates them.

Cellular injury Page 2.2


1.3 Cell death

1. What is morphologic hallmark of cell death?


- Loss of nucleus

2. What are the steps by which nucleus is lost?


• Pyknosis - shrinking of nucleus
• Karyorrhexis - breaking of nucleus to big pieces
• Karyolysis - big nuclear pieces broken down to molecular building blocks

3. Explain necrosis
• Necrosis occurs as a result of acute inflammation to a large group of cells and is always pathologic
• Types
1. Coagulative necrosis
2. Liquefactive necrosis
3. Gangrenous necrosis - dry and wet
4. Caseous necrosis
5. Fat necrosis
6. Fibrinoid necrosis

4. Describe coagulative necrosis


- Cell and organ structure is preserved by coagulation of cellular proteins
- Characteristic of ischemic infarction of any organ except brain

Cellular injury Page 3.1


Fig - right is normal glomerulus and left is coagulative necrosis of glomerulus. Note cellular
and organization structure is preserved with loss of nucleus.

- Area of infracted tissue is often wedge shaped and pale. Wedge points to the blocked blood
vessel.

5. When does red infraction (hemorrhagic infraction) occur?


• Red infraction occurs when blood reenters a loosely organized tissue. Ex - ischemia of testis, lungs.
(is red infraction a type of coagulative necrosis?)

6. Describe liquefactive necrosis


- Necrosis in which enzymatic lysis of cells and proteins result in liquefaction
- Characteristic of brain (microglial cells destroy the tissue), abscess (neutrophils contain hydrolytic
enzyme that destroy the tissue), and pancreatitis (pancreatic enzyme digest the pancreas and
liquify; however surrounding fat will have fat necrosis)

7. Describe gangrenous necrosis


- It is coagulative necrosis that resembles mummified tissue (dry gangrene). If infection occurs on
dry gangrene, liquifactive necrosis occurs and is called wet gangrene.
- Characterstic of ischemia of lower limb and GI tract

8. Describe caseous necrosis


- It is soft, friable necrosis with 'cottage cheese' like appearance
- It's liquefactive necrosis with debris that thickens the soup to make it cheese like
- Characteristic of granulomatous inflammation of TB or fungal infection (fungal cell wall and
micobacterium thickens the soup).

Cellular injury Page 3.2


Fig - caseous necrosis

9. Describe fat necrosis


- Necrotic fat with Chalky white appearance due to deposition of Ca. MOA - When fat cells die by
trauma or when fat is broken down by lipase, fatty acids are released. In presence of Ca,
saponification occurs which gives it the chalky white apperance. Ca accumulation happens by
dystropic calcification.
- Characteristic cases - trauma to fat (ex - as in car accident trauma in breast), pancreatitis mediated
damage of peripancreatic fat

Fig - fat necrosis around pancreas

10. What is fibrinoid necrosis


- Necrotic damage to blood vessel
- Proteins leak into vessel wall which results in bright pink staining of vessel wall in H&E
- Characteristic of malignant hypertension and vasculitis. Pre-eclampsia can cause fibrinoid necrosis
of placenta. (benign HTN is chronic low increase in BP that cause slow damage. Malignant
hypertension is sudden super elevated BP that is a medical emergency - findings such as
headache, renal failure, papilledema). Super high BP in malignant HTN can kill vessel wall cells.

11. What are mechanism by which calcium can deposit in human tissue?
- Dystropic calcification
- Metastatic calcification

12. What is dystropic calcification?


- It's a mechanism by which Ca deposits in a human body (it's abnormal as calcium is not supposed
to just randomly accumulate). Mechanism - when there's a dead tissue, calcium can accumulate
even if serum Ca is normal. Ex - in fat necrosis, in tumor as center of tumor might not get enough
blood supply and die.

13. What is metastatic calcification?

Cellular injury Page 3.3


13. What is metastatic calcification?
- Serum Ca or PO4 is elevated. This elevated stuff can force Ca to accumulate on tissue. It doesn't
mean that pt has metastatic cancer.

Apoptosis
14. Explain apoptosis. (apoptosis means falling of leaves)
- It is purposeful, energy dependent killing of small group of cells
- Examples - Endometrial shredding during menstruation, removal of cells in embryogenesis, CD8
cell mediated killing of virally infected cells

15. What is mechanism of apoptosis?


- Cell shrinks (and becomes eosinophilic due to concentration of cytoplasm) and nucleus also
shrinks and fragments.
- Apoptotic bodies fall from cell like leaves fall from tree. Then they are eaten by macrophage. No
inflammation occurs

Fig - compare the pink apoptotic cell to it's surrounding neighbors

16. What is biochemical pathway leading to apoptosis?


- Caspase activation is the key step.
○ Caspases activate proteases that break down cytoskeleton.
○ Caspases also activate endonucleases that break down nucleus.

17. What 3 pathways lead to caspase activation? (HY)


- Intrinsic mitochondrial pathway - Bcl2 is a protein whose function is to stabilize mitochondrial
membrane so that cytochrome C from mitochondria can't leak out. Due to membrane damage
(cellular injury), DNA damage, or decreased hormonal stimulation of cell (ex - endometrial cells in
lack of estrogen), Bcl2 can be inactivated. Leakage of cytochrome C will then activate caspase.
- Extrinsic receptor-ligand pathway -
○ Example- FAS ligand binds to FAS death receptor (CD95) on target cell and target cell dies
via activation of caspases. Case report - new CD8+ T cells from bone marrow undergo
positive selection (I.e., if they can bind to self antigen on MHC-I molecule, they get to live)
and negative selection (i.e., if they bind to self antigen on MHC-I molecule very strongly,
they die) in thymus. The killing of these young T-cells in negative selection happens by
binding of FAS ligand to FAS death receptor on T-cells.
○ Another example - TNF binds to TNF receptor on target cell and cell dies.
- Cytotoxic CD8+ T-cell pathway - When CD8+ cells recognize foreign antigen on MHC-I receptors,
they release perforins that make holes on target cell. Then, they release granzyme that enter
target cell and activate caspases.

Cellular injury Page 3.4


1.4 Free radical injury (HY)

1. What is free radical?


• Atoms or molecules with unpaired electron in outer orbit

2. Give example of physiologic and pathologic free radical?


• Physiologic - in oxidative phosphorylation, when oxygen receives 1, 2 or 3 electron, it's partially
reduced and creates superoxide (O2.), hydrogen peroxide (H2O2), and hydroxide(.OH) free radical
respectively. Only if O2 receives 4 electron, it becomes water. Of all free radicals, hydroxide free
radical is most damaging (HY).

Oxygen ----(accept 1 electron)-----> Superoxide (.O2) ----(accept 1 electron)-----> Hydrogen


peroxide (H2O2) ----(accept 1 electron)-----> Hydroxide (.OH) ----(accept 1 electron)-----> Water
(H2O)

• Pathologic -
▪ ionizing radiation (creates hydroxide radical .OH),
▪ inflammation (in neutrophils, O2 is converted to superoxide by NADPH oxidase)
▪ Metals - iron and copper. Hemochromatosis and Wilson disease are disease of Fe and
Cu accumulation in body - creation of free radicals are main way that the disease
affect us. (Fenten reaction - reaction by which Fe generates hydroxide free radical
(HY))
▪ Drugs and chemicals - acetaminophen, carbon tetrachloride. High dose of
acetaminophen causes liver necrosis by free radical generation
▪ Thyroid hormones increase free radical formation

3. How do free radicals harm the cell?


• Oxidation of DNA (lead to fast aging and cancer) and oxidation of proteins
• Peroxidation of lipids

4. What are 3 mechanism of removal of free radicals? Give example of 3 enzymes.


• Antioxidants - vit K
• Metal carrier proteins - transferrin, ferretin
• Enzymes (HY)
○ Superoxide dismutase (gets rid of superoxide)
○ Catalase ( gets rid of hydrogen peroxide)
○ Glutathione peroxidase (get rid of hydroxide free radical)

5. Give example of 2 free radical injury? (HY)


• Carbon tetrachloride (dry cleaning chemical)- CCl4 is converted to CCl3 (radical) by Cyp 450 in
liver. It causes reversible damage to hepatocytes - cell swells --> swelling of RER causes ribosomes
to fall off and apolipoprotein synthesis goes down --> liver can't repackage and send fat away -->
fatty liver disease (VHY)

Cellular injury Page 4.1


Fig - T represents fat in hepatocyte; a hepatocyte is shown in circle
• Reperfusion injury - During reperfusion of ischemic organ, immune cells will attack the dead tissue
in presence of oxygen which will generate free radicals. More injury ensues. Classic example is
blood troponin continuing to rise after reperfusion of heart after MI.

Cellular injury Page 4.2


1.5 Amyloidosis

1. What is amyloid?
• Misfolded proteins that accumulates in extracellular space and damages tissue. Amyloid can't be
removed and damaged organs must be transplanted

2. What are characters of amyloid?


• Misfolded proteins have B-sheet arrangement
• Amyloid are congo red positive (appear apple-green under polarized light)

Fig - apple-green appearing amyloid under polarized light


• Amyloid can be deposited systemically or locally

3. What are 2 types of systemic amyloidosis?


• Primary amyloidosis - When Ig light chain deposits systemically (in form of AL amyloid), it's called
primary amyloidosis. It's associated with plasma cell dyscrasias (disease) - when plasma cells make
ton of light chain, they will deposit.
• Secondary amyloidosis - when SAA protein deposits systemically (in from of AA amyloid), it's called
2ndary amyloidosis. SAA is acute phase reactant that is increased in chronic inflammatory states,
malignancy, and Familial Mediterranean fever.

4. What is familial mediterranean fever (HY)?


• It's genetic disease with dysfunction of neutrophils that causes systemic amyloidosis. It's usually
seen in people of mediterranean origin.
• Pt will have high SAA that deposits at AA amyloid
• Pt may present with acute fever and serosal inflammation (serosal inflammation of heart might
mimic MI, of gut may mimic appendicitis)

5. What are classic findings of systemic amyloidosis?


• Kidney is most commonly involved - nephrotic syndrome is seen
• Restrictive cardiomyopathy or arrhythmia
• Hepatosplenomegaly, malabsorption, large tongue

6. How do you diagnose amyloidosis?


• Requires tissue biopsy (needs to be congo-red positive) - usually take from rectum or abdominal
fat pads

7. Describe senile cardiac amyloidosis (a type of local amyloidosis).


• Caused due to deposition of non-mutated serum transthyretin in heart. Transthyretin is 2nd most
common protein in blood.

Cellular injury Page 5.1


common protein in blood.
• Usually asymptomatic and seen in 25% of people >80 year

8. Describe familial amyloid cardiomyopathy (a type of local amyloidosis).


• Caused due to deposition of mutated serum transthyretin in heart.
• Leads to restrictive cardiomyopathy
• Usually seen in african americans (5% carry mutation)

9. Describe how type 2 diabetes has local amyloidosis in it.


• Due to insulin resistance, pancreas produces lots of insulin. A byproduct amylin is also produced
that deposits in islets of pancreas and cause amyloidosis.

10. Describe how alzhimer's has local amyloidosis in it.


• B-amyloid precursor protein (BAPP) on chromosome 21 makes AB amyloid plaques. (Most
individuals with Down's syndrome have Alzheimer's by 40 year age)

11. Describe how dialysis can cause local amyloidosis in joints.


• Beta2 microglobulin is a protein that structurally supports MHC-I on cell surface. In dialysis, B2-
microglobulin isn't filtered well from blood. It then builds up in joints and cause amyloidosis.

12. Describe how medullary thyroid cancer can cause local amyloidosis in thyroid.
• In MTC, overproduction of calcitonin leads to formation of calcitonin amyloid plaques in thyroid.
(In FNA of thyroid, seeing tumor cells in amyloid background makes MTC more probable)

Cellular injury Page 5.2


Chapter 2: Inflammation,
Inflammatory Disorders, and Wound
Healing
2.1 Acute Inflammation (Part 1)

1. What is inflammation?
• Process by which inflammatory cells, plasma proteins and fluid exit blood vessel to enter
interstitial space

2. Explain acute inflammation


- Characterized by edema and neutrophil in tissue (presence of neutrophils define acute
inflammation)
- Arises in response to infection or necrosis (necrosis is always followed by acute inflammation (HY))
- Goal is to clear pathogen or necrotic debris
- Part of innate immunity (very quick response)

3. How does TLR work?


- Toll like receptors (TLR)- present on cells of innate immune system (macrophage, dendritic cells)
and adaptive immune system. In innate system, TLR recognize PAMP (pathogen associated
molecular patterns)and DAMP (damage associated molecular pattern). Ex - CD14 (aka TLR-4) is a
TLR on macrophage that recognizes lipopolysaccharide (LPS) which is shared by gram negative
bacteria. (HY)

3.1 What chemical is upregulated by TLR?


- NF-KB- NF-KB is a molecular switch that turns on acute inflammation (master regulator for both
innate and adaptive immunity).
- Type 1 interferon (alpha and beta) - inhibit viral replication in cells

Arachidonic acid metabolism


4. Explain production and metabolism of arachidonic acid (AA).
- Phospholipase A2 releases AA from phospholipid cell.
- AA can then be metabolized by cyclooxygenase or 5-lipooxygenase.

5. What are the products of cyclooxygenase pathway and what are their functions.
- Cyclooxygenase pathway produces prostaglandins (PG) - PGI2, PGD2 and PGE2. They increase
vascular permeability and vasodilation. PGE2 also mediates feeeever and pain. Note that
vasodilation happens at arteriole level and vascular permeability at post capillary venule level.

6. What are the products of lipooxygenase pathway and what are their functions
- 5-Lipooxygenase pathway produces leukotrienes (LT). LTB4 attracts and activates neutrophils.
LTC4, LTD4 and LTE4 mediate smooth muscle contraction (vasoconstriction, bronchospasm, and
increased vascular permeability (contraction of pericytes opens up space between endothelial
cells)).

7. What are 4 important things that attract and activate neutrophils?


- LTB4 (leukotriene)
- C5A (complement protein)
- IL8 (mechanism by which macrophage call neutrophil)
- Bacterial products
- TNF and IL1 (based on Abbas book)

Mast cells
7. What are 3 ways that activate mast cells?
- Mast cells are found throughout connective tissue of body and are activated by -

Inflammation Page 1.1


- Mast cells are found throughout connective tissue of body and are activated by -
○ Tissue trauma
○ Complement proteins C3a and C5a
○ Cross linking of cell surface IgE by antigen (2 or more IgE on a mast cell binds same antigen)

8. What is acute response of mast cells activation?


- Histamine is released which causes vasodilation and increased vascular permeability.
- Release proteolytic enzymes that kill bacteria or inactivate toxins

9. What is delayed response (after 4-5 hrs) of mast cells activation?


- Mast cells produces arachidonic acid metabolites, particularly leukotrienes. (HY)

Complement
10. What are three pathways of complement activation?
- Classic pathway - C1 protein binds to IgG or IgM which is bound to antigen (pneumonic - GM
makes classic cars)
- Alternative pathway - microbial products directly activate complement
- Mannose binding lectin (MBL) pathway - MBL binds to mannose on microorganisms and activate
complement

11. How is membrane attack complex (MAC) formed?


- C3 convertase converts C3 to C3a and C3b
- C5 convertase converts C5 to C5a and C5b
- C5b joins with C6 and C9 to make membrane attack complex (MAC) which pokes hole in cell and
kill it.

12. What are some key functions of complement proteins? (HY)


- C3a and C5a activate mast cells (stimulate degranulation)
- C5a is chemotactic for neutrophil
- C3b is opsonin for phagocytosis. Opsonins tag cells/waste for phagocytosis.
- MAC pokes hole in cell and kill it

13. What is hageman factor (aka factor 12 in coagulation cascade)? What systems does it activate?
- It's inactive proinflammatory protein produced in liver that's activated by exposure to
subendothelial or tissue collagen, and during severe gram negative sepsis
- Plays imp role in DIC (disseminated intravascular coagulation - pathologic activation of coagulation
cascade) and severe gram negative sepsis (sepsis activates hageman factor which in turn results in
DIC) (HY).
- It activates complement system, coagulation and fibrinolytic system (this results in DIC), and kinin
system

13.1. Explain kinin system.


- Kinin system cleaves high molecular weight kinin to bradykinin. Bradykinin results in vasodilation,
increased vascular permeability and pain). Note - bradykinin system has histamine + pain functions

14. What two molecules mediate pain?


- Prostaglandin E2 (cause feeever and pain)
- Bradykinin

Cardinal signs of inflammation - rubor, calor


15. What causes rubor (redness) and calor (warmth)?
- Vasodialation due to arteriole relaxation mediated by - histamine (primary), bradykinin and PG.

16. What causes tumor (swelling)?

Inflammation Page 1.2


16. What causes tumor (swelling)?
- Increased vascular permeability at post capillary venule mediated by - histamine, bradykinin,
lukotrienes, PG, and tissue damage.

17. What causes dolor (pain)?


- Bradykinin and PGE2.

18. What causes fever? (VHY)


- Macrophage release IL-1 and TNF. They go to perivascular cells of hypothalamus and increase
activity of COX. (inflammosome also makes IL1)
- COX causes increase in PGE2 in hypothalamus which increases temp set point.

Inflammation Page 1.3


2.1 Acute Inflammation (Part 2)

1. What are 3 phases of acute inflammation?


○ Fluid phase - edema, complement activation etc
○ Neutrophil phase - peaks about 24 hrs
○ Macorphage phase - peaks about 2-3 days
(Note that acute inflammation is not defined by time. It can happen for days or weeks. It
is defined by whether neutorphils are primary player. If pus is present, it's still acute.)

2. What are steps of neutorphil exit? What proteins are key in each step?
- Margination - Usually, heavy particles in blood are in center of vessel lumen. When blood
vessel dilates, heavy particles come to edge of vessel and this is margination. It's first step of
neutrophil arrival.
- Rolling - endothelial cells express proteins called selectins which act as speed bump so that
neutrophils roll and slow down.
- Adhesion - cellular adhesion molecules (CAM) on endothelium binds to integrins on
neutrophil resulting in adhesion. (HY)
- Transmigration and chemotaxis - transmigration happens across vessel wall of postcapillary
venules, and chemotaxis is movement to infection site after transmigration. Neutrophils are
attracted by bacterial products, IL8, C5a and LTB4
- Phagocytosis - enhanced by opsonins (IgG and C3b)

3. What induces P selectin and E selectin?


- P selectin release is induced by histamine. They are released from Weibel-Palade bodies in
endothelial cells (another stuff made by weibel-palade body is Von-willieband factor).
- E selectin release is induced by TNF and IL-1.

4. What protein on neutrophils does selectins bind to?


- Sialyl Lewis X. It results in rolling .

5. What upregulates cellular adhesion molecules (CAM) on endothelium?


- IL1 and TNF

5.5. What are major innate immunity cytokines?


- IL-1 and TNF

6. What upregulates integrins on neutrophils?


- C5a and LTB4 (recall that C5a and LTB4 are 2 of the 4 things that attract and activate
neutorphils - other 2 being bacterial product and IL8)

7. What is leukocyte adhesion deficiency? Where is the mutation?


- It's autosomal recessive mutation of integrins at CD18 subunit.

8. What are sign/symptoms of leukocyte adhesion deficiency (LAD)? (HY)


- Delayed seperation of umbilical cord (HY) - After childbirth, the umbilical cord shuts. Due to
no blood through it, it undergoes necrosis. Active inflammation ensues. Neutrophils come and
eat up umbilical cord. However, in LAD, neutrophils won't come much because adhesion
process is bad.
- Increased circulating neutrophils (HY) - normally, half of neutrophils are in circulation and
remaining half are stuck to endothelium of lung. Due to loss of adhesion, circulating

Inflammation Page 2.1


remaining half are stuck to endothelium of lung. Due to loss of adhesion, circulating
neutrophils will increase.
- Recurrent bacterial infection but lack pus formation (HY) - Pus is dead neutorphil in fluid. If
neutrophils can't get into tissue, pus can't form.

9. Where do neutrophils exit blood vessel?


- Post capillary venule.

10. How does phagocytosis occur?


- Neutrophils engulf stuff and make phagosome. Phagosome merges with lysosome to make
phagolysososme. (C3b and IgG are major opsonins)

11. What is Chediak-Higashi syndrome?


- It's an autosomal recessive protein trafficking defect (microtubule railroad tracks in the cells
are defective).

12. What are sign/symptoms of Chediak-Higashi syndrome? (HY)


- It's symptoms are based on impaired microtubule railroad tracks
○ Immune fingings-
 Neutropenia (impaired cell division so low neutrophils)
 Giant granules in leukocytes (newly made granules from golgi won't travel well to
other parts of cell and pile up)
 Increased pyogenic infections (phagosome merging with lysosome is impaired, so
immune cells are less efficient at killing pathogens).
○ Defective primary hemostasis (granules in platelets won't move well)
○ Albinism (a melanocyte supplies melanin to about 25 keratinocytes. As railroad
transport is defective, it's melanin won't be transported to other keratinocytes)
○ Peripheral neuropathy - stuff from cell body of neuron won't be transported to axon
endings well.

Inflammation Page 2.2


2.1 Acute Inflammation (Part 3)

1. What are two mechanism of phagocytosis? Which is more efficient


• Oxygen dependent and oxygen independent.
• Oxygen dependent is more efficient

2. Explain O2 dependent mechanism of phagocytosis. (HY)


• It occurs in phagolysosome
• Oxygen is converted to superoxide (radical) by NADPH oxidase (rxn called oxidative burst).
• Superoxide is coverted to hydrogen peroxide by superoxide dismutase.
• Hyrodgen peroxide is converted to bleach (HOCl) by myloperoxide (MPO)
• HOCl kills organism

3. What is mechanism of Chronic granulomatous disease (CGD)?


• Due to defect in NADPH oxidase, O2 dependent phagocytosis is bad and patient has chronic
granulomas. Disease is autosomal recessive or X-linked.
• As pt can't phagocytose, more and more macrophage and neutrophils are called to site resulting in
formation of granulomas.

4. What are 5 catalase positive organism that cause symptomatic CGD? (HY)
○ S Aureus
○ P cepacia (often tested)
○ S marcescens
○ Nocardia
○ Aspergillus

5. Why do only catalase positive organism cause CGD?


• Most bacterias produce H2O2 which can be converted to bleach by myloperoxidase in body.
Therefore, most bacteria can't cause CGD despite mutation in human NADPH oxidase. However,
when bacteria have catalase, that destroys H2O2 and formation of bleach is reduced which results
in CGD.

Inflammation Page 3.1


in CGD.

6. What is nitroblue tetrazolium (NBT) test?


• It's a screening test for CGD. In the test, solution turns blue if superoxide radical is present. It tests
if NADPH oxidase is functional.

7. What is clinical significance and NBT test result for people with myloperoxide (MPO) deficiency?
• Most MPO patients are asymptomatic but have increased risk for candida infection.
• They have normal NBT test.

8. Explain oxygen independent phagocytosis.


• It's less effective and occurs via enzyme in secondary granules (ex - lsyozome and major basic
protein).

9. Describe the resolution phase of acute inflammation.


• Within 24 hours of resolution of inflammatory stimulus, neutrophils die by apoptosis. It creates
pus. Pus is dead neutrophils in fluid.

10. Describe macrophage phase of acute inflammation.


• It peaks 2-3 days after inflammation begins.
• When monocytes get into tissue, they are called macrophage. They come to tissue in same steps
as neutrophils.

11. Contrast phagocytosis of macrophage with that of neutrophil.


• Phagocytotic killing of macrophage is mainly oxygen independent - via the enzymes in secondary
granules. Lysozyme is main enzyme.
• Phagocytotic killing of neutrophil is mainly oxygen dependent.

12. What happens after fluid, neutrophil and macrophage phase of acute inflammation? (HY)
• Macrophage are managers which come in to check after 2-3 days of acute inflammation if
neutrophils did a good job at clearing the damage.
• If good job is done, macrophage secrete IL-10 and TGF-B. They are anti-inflammatory and induce
healing.
• If job is done poorly, macrophage secrete IL-8. It calls additional neutrophils to the site.
• If macrophage sense that the offending organism needs to walled off, they create abscess.
Abscess is walled off area of inflammation.
• If macrophage sense that neutrophils can't do the job well (ex- neutrophils don't work well in viral
injury), they initiate chronic inflammation by presenting antigen in MHC-2.

Inflammation Page 3.2


2.2 Chronic inflammation

1. What is chronic inflammation?


• Inflammation is bringing out immune cells into the tissue. In chronic inflammation, mainly
lymphocytes are brought out. In acute inflammation, mainly neutrophils are brought out.
• It's delayed response (adaptive immunity) and more specific.

Fig - chronic inflammation. Note the absence of multilobed neutrophil. Cell on left is lymphocyte.
Middle cell is plasma cell (nucleus pushed to side, slight perinuclear halo). Cell on right is
macrophage (clock face nucleus).

2. What are stimuli of chronic inflammation?


• Persistent infection (most common)
• Infection with virus, mycobacteria, parasites, fungi
• Autoimmune disease
• Foreign material
• Some cancers

T cells
3. Explain development and maturation of T-cells.
• They are made in bone marrow and go to thymus.
• In thymus, they become specific to an antigen and develop into either to CD4 helper or CD8
cytotoxic cell.

4. How are CD4 and CD8 T-cells activated? What is their second message for activation?
• T cells are activated when they recognize antigens presented on MHC molecules only while a
second signal is present.
• Helper CD4 T cells read antigen from MHC II. MHC II present extracellular antigen and are present
on antigen present cell.
• Cytotoxic CD8 T cells read antigen form MHC I. MHC I present intracellular or viral antigen and are
present on all cells in body
• For CD4 cells, B7 on antigen presenting cell is the second signal that binds to CD28 on CD4 cells.
CD40 on B cells also act as second signal that bind to CD40L on CD4 cells.
• For CD8 cells, IL2 from CD4 cells is second signal for activation.

5. What do activated CD4 cells do?


• Activated CD4 helper cells secrete cytokines. They have two subtypes - TH1 and TH2.

6. Explain purpose and function of cytokines secreted by activated CD4 TH1 cells.
• TH1 secrete IFN gamma that enhances phagocytic ability of macrophage
• TH1 also secrete IL2 which is T cell growth factor and CD8 cells activator

7. Explain purpose and function of cytokines secreted by activated CD4 TH2 cells.
• TH2 cause eosinophil rich inflammation and/or reduce inflammation by secreting IL4, IL5 and IL10.

Inflammation Page 4.1


• TH2 cause eosinophil rich inflammation and/or reduce inflammation by secreting IL4, IL5 and IL10.
• IL 4 cause class switching to IgG and IgE
• IL5 attracts and activates eosionophil, helps in maturation of B cells to plasma cells and cause class
switching to IgA
• Il10 (antiinflammatory cytokine) inhibits TH1 phenotype.

8. How do CD8 T cells kill cells?


• They secrete perforins that make holes in host cell and secrete granzymes that activate caspase
and induce apoptosis.
• They express Fas ligand which binds to Fas receptor on host cell that activates apoptosis.

8.1. How is a self-reactive T-cell killed?


- If there's a self-reactive T-cell, it will bind to MHC-antigen but the second signal won't be present.
It will result in apoptosis or anergy.

B cells
9. How are B cells produced?
• Immature B cells are produced in bone marrow. Naïve B cells express IgM and IgD.

10. How are 2 ways by which B cells activated? How does isotype switching occur?
• Binding of antigen to IgM or IgD activates B cells. Then they become IgM or IgD secreting plasma
cells.
• B cells can phagocytose and present antigen to CD4 cells via MHC II. CD40 on B cell can bind to
CD40L on helper T cell providing 2nd signal for activation.
• TH2 Helper T cell will secrete IL4 and IL5. IL4 and IL5 help in isotype switching of B cells, somatic
hypermutation (aka affinity maturation), and maturation to plasma cells.

Granulomatous inflammation
11. What is granulomatous inflammation?
• It’s a type of chronic inflammation. It's defined by presence of epitheloid histiocytes (macrophage
with abundant pink cytoplasm and elongated nuclei) (HY). You may also see multinucleated
macrophages. In healthy tissue, macrophage have clear (aka foamy) cytoplasm.
• Granulomas are surrounded by rim of lymphocytes and giant cells.
• Only a certain conditions give granuloma so they help to identify diseases.

12. What cause noncaseating granulomas?


• Defining feature of noncaseating granuloma is lack of central necrosis.
• Causes:
○ Reaction to foreign material - ex- leaking of breast implants
○ Sarcoidosis - hallmark of sarcoidosis is production of non-caseating granuloma in multiple
organs (mainly lungs)
○ Beryllium exposure
○ Crohn disease - hallmark of Crohn is noncaseating granuloma. (hallmark of UC is crypt
abscess)
○ Cat scratch disease - give star shaped granuloma in neck

Inflammation Page 4.2


Fig - noncaseating granuloma. Not abundant lymphocytes in periphery that indicates chronic
inflammation. Note multinucleated cell. Presence of nucleus in epitheloid histiocytes means
they are alive - making this noncaseating granuloma.

13. What causes caseating granuloma?


• Characteristic of TB and fungal infections. If you see caseating granuloma, do AFB stain to look for
TB, and GMS silver stain to look for fungal infections.

Fig - caseating granuloma. Note the central necrosis

14. How are granulomas formed? (HY)


• Macrophage present antigen to CD4 T cells via MHC II
• After binding, macrophage also secrete IL-12 that induce CD4 cells to differentiate to Th1 subtype
• Th1 cells secrete IFN gamma which converts macrophage to epitheloid histiocytes and giant cells.
Granuloma is hence formed.
• These steps occur in both caseating and non-caseating granuloma

Inflammation Page 4.3


2.3 Primary immunodeficiency

1. What is DiGeorge syndrome (HY)?


• Failure to develop 3rd and 4th pharyngeal pouch due to 22q11 mutation.

2. What are presentation of DiGeorge syndrome?


• T cell deficiency- problems fighting viral and fungal infection (due to lack of thymus as thymus develops from 3 rd and
4th pouch)
• Hypocalcemia (due to lack of parathyroid)
• Abnormalities of heart, great vessels, and face

3. What is SCID (severe combined immune deficiency)?


• Pt have defective cell-mediated and humoral immunity (both T and B).

4. Etiology of SCID?
• Cytokine receptor defects (main cause)
• Adenosine deaminase deficiency (second main cause)(HY) - enzyme necessary for deamination of adenosine and
deoxyadenosine. In lack of enzyme, adenosine and deoxyadenosine accumulates in immune cells causing toxicity.
• MHC class II deficiency - CD4 won't be able to be activated. It leads to defective function of CD8 and B cells.

5. Presentations of SCID?
• Lack of T cells result in fungal and viral infections
• Lack of B cells result in bacterial and protozoal infections
• Increased opportunistic infection and be wary to give live vaccines.

6. Treatment of SCID?
• Sterile isolation (bubble babies)
• Stem cell transplant

8. What is X-linked agammaglobulinemia? What is the mutation?


• Name suggests pt lacks gammaglobulin (antibodies) in blood. There's a complete lack.
• Occurs because naïve B cells can't mature to plasma cells.
• Mutation is Bruton tyrosine kinase. It's a signaling receptor that helps maturation of B cells.

9. What is presentation of X-linked agammaglobulinemia?


• Recurrent bacterial, enterovirus, and Giardia infection (HY).
• Enterovirus affects mucosa of GI tract. IgA protects mucosal surface, and lack of IgA results in enterovirus infection.
• Bacterial infection occurs because lack of antibodies affect opsonization.
• Giardia occurs for same reasoning as enterovirus.
• Presents after 6 month of life – because mom's antibodies last for about 6 months
• Don't give them live polio vaccines

10. What is common variable immunodeficiency disease?


• Pt has low antibodies due to defective B or T cell

11. Presentation?
• Pt has increased risk of bacterial, enterovirus, giardia infection, often in late childhood.
• They have increased risk of autoimmune disease and lymphoma. (HY)

12. What is IgA deficiency?


• This is most common Ig deficiency. Pt have low serum and mucosal IgA which increases risk of mucosal infections –
especially viral.

Inflammation Page 5.1


especially viral.

Note – pt with celiac disease usually have IgA deficiency.

13. What is Hyper-IgM syndrome?


• Pt have too much IgM.

14. What is pathophysiology of Hyper-IgM syndrome? What is presentation?


• Patient has mutation in CD40 or CD40 receptor. Due to this, B cells can't activate CD4. In lack of good CD4, cytokines
necessary for Ig class switching is not produced. Pt will have low IgA, IgG, and IgE.
• Presentation is recurrent pyogenic infections, especially at mucosal sites. IgG is a opsonin, low amount results in more
formation of pus.

15. What is Wiskott-Aldrich syndrome (WAS)? What causes it? (just memorize)
• Pt have thrombocytopenia, eczema, and recurrent infections. Pt have defective humoral and cellular immunity.
• Happens due to mutation in WASP (Wiskott-Aldrich syndrome protein) gene and is x-linked.

16. What are the consequence of the following complement deficiencies? (just memorize)
• Deficiency from C5-C9 - increased risk of Neisseria infection (HY)
• C1 inhibitor deficiency - hereditary angioedema characterized by edema of skin (especially periorbital) and mucosal

Fig - heriditary angioedema.

Inflammation Page 5.2


2.4 Autoimmune Disorders

1. What causes autoimmune disorders?


- Overactive immune system or loss of self tolerance.

2. What is epidemiology of autoimmune disorders.


- Present in about 1% of US popn.
- Most affects women. Classically affects women of childbearing age.
- Most autoimmune diseases are associated with other autoimmune diseases.

3. What is etiology of autoimmune disorders?


- Environment triggers disease in genetically susceptible individuals.

Systemic Lupus
4. What happens in Lupus?
- Lupus is a systemic autoimmune disease in which autoantibodies can cause type II (cytotoxic -
antibody bind and kill cells) or type III hypersensitivity reaction (antigen-antibody complex form
and deposit in tissue causing damage).

5. What are clinical features of lupus?


- Malar 'butterfly rash' upon exposure to sunlight is classic sign
- Diffuse proliferative glomerulonephritis commonly occurs; other nephritic/nephrotic symptoms
can occur too. Renal damage is common cause of death
- Pleuritis and pericarditis, myocarditis, endocarditis
- Libman-Sacks endocarditis - vegetations on both side of valve
- Antibodies against blood result in - anemia, thrombocytopenia or leukopenia. Infections due to
loss of immune system also a common cause of death.
- CNS psychosis due to inflammation of CNS
- Fever and weight loss (fever is sign of inflammation)
- Arthritis

Fig - malar butterfly rash in lupus pt.

6. Describe Libman-Sacks endocarditis.


- Mostly, endocarditis happens only on one side of valve (up or down). If it happens on both side of
valve, it's called Libman-Sacks endocarditis and is characteristic of lupus.

7. How do we diagnose lupus?


- ANA (anti-nuclear antibody) very sensitive for lupus - used for screening
- Anti-dsDNA very specific for lupus

Inflammation Page 6.1


8. What are three common drug that cause lupus? How do we diagnose? How do we treat?
- Hydralizine, procainamide and isoniazid
- Antihistone antibodies are specific for drug induced lupus
- Removal of drug causes remission of disease

9. What is antiphospholipid syndrome associated with SLE? What lab tests does it affect?
- Antibodies are made against proteins bound to phospholipid.
- Anticardioliptin antibody give false positive syphilis test.
- Lupus anticoagulant antibodies give falsely elevated PTT lab study but makes pt hypercoagulable.

10. Why do you keep pt with antiphospholipid syndrome on lifelong anticoagulation?


- Because lupus anticoagulant antibodies make pt hypercoagulable and increase risk of stroke, DVT,
hepatic vein thrombosis, placental thrombosis (pregnancy loss).

Sjogren syndrome
11. What is sjogren syndrome?
- Type IV (lymphocyte mediated) autoimmune destruction of lacrimal and salivary glands with
resulting fibrosis.

12. What are clinical presentation of Sjogren syndrome?


- Dry eyes, dry mouth and recurrent dental carries (bacteria accumulates as teeth aren't washed) in
older woman.
- Parotids may be enlarged due to fibrosis.
- "Can't chew cracker, dirt in my eyes"

13. How do you diagnose Sjogren syndrome?


- Presence of ANA is sensitive
- Presence of anti-ribonucleoprotein antibodies is specific - (Anti SSA and anti SSB antibodies -
Sjogren syndrome A and sjogren syndrome B)

14. What other conditions is Sjogren syndrome associated with?


- Other autoimmune disorders - especially rheumatoid arthritis
- High risk for B-cell lymphoma (HY) - unilateral enlargement of partoid gland late in disease is
indicative of B-cell lymphoma.

Scleroderma (thick skin)


15. What is scleroderma?
- Autoimmune tissue damage with activation of fibroblast and deposition of collagen (fibrosis) that
thickens the tissue (also makes it tight and less mobile).

Inflammation Page 6.2


16.

Fig- scleroderma hands

17. What is diffuse type scleroderma? What are clinical presentation? How do you diagnose?
- Patient has diffuse skin and early visceral organ involvement.
- Esophagus most commonly affected - see solid and liquid dysphagia
- Diagnosis made by ANA and anti-DNA topoisomerase I antibody (aka SCL-70 antibody). SCL stands
for scleroderma.

17. What is localized type scleroderma? What are clinical presentation? How do you diagnose?
- Patient has local skin and late visceral organ involvement.
- Presentation (CREST)-
○ Calcinosis (calcification of skin), anti-centromere Ab
○ Raynaud phenomena
○ Esophageal dysmotility- see solid and liquid dysphagia
○ Sclerodactyly
○ Telangiectasis of skin
- Diagnosis made by anti-centromere antibody

Mixed connective tissue disease


18. What is mixed connective tissue disease? How is it diagnosed?
- It is autoimmune tissue damage with mixed features of SLE, scleroderma and polymyositis
(autoimmune damage of proximal muscles). Think of patient with bunch of autoimmune
condition.
- Diagnosis made by presence of antibodies against U1 ribonucleoprotein.

Inflammation Page 6.3


2.5 Wound Healing

1. What is regeneration?
- It's replacement of damaged tissue with native tissue.

2. What is repair?
- Repair is replacement of damaged tissue with fibrous scar. It occurs when tissue lacks
regenerative capacity or regenerative stem cells are lost.

3. What are 3 different types of tissue based on regeneration capacity?


- Labile tissue - they are continuously regenerating
○ Intestine - stem cell in mucosal crypt
○ Skin - stem cell in basal layer (most bottom layer of epidermis)
○ Bone marrow - hematopoietic stem cells (HSC). (HY - You can't identify HSC by microscope.
Presence of CD34 molecule tells it's HSC)
○ Lungs - stem cell is type 2 pneumocytes

Fig - circles are the crypts of bowel

Fig - line showing basal layer of skin


- Stable tissue - they don't continuously regenerate but have the ability to regenerate if needed.
○ Liver
○ Proximal tubule of kidney - pt on acute tubular necrosis are put on dialysis until their
proximal tubules regenerate
- Permanent tissue - can't regenerate
○ Myocardium
Skeletal muscle

Inflammation Page 7.1


○ Skeletal muscle
○ Neurons

4. What is granulation tissue (different from granuloma which occurs in chronic inflammation)?
What are 3 important cell types in it?
- It's present in early phase of repair. It contains -
○ Fibroblasts (deposit type 3 collagen)
○ Capillaries
○ Myofibroblast (contract wound)

Fig - granulation tissue showing blood vessels, collagen and fibroblast. Contrast it with granuloma
in chronic inflammation file.

5. What happens when granulation tissue is replaced by scar? What cofactor is needed?
- Collagenase removes type 3 collagen and replaces with type I collagen. It needs zinc as cofactor
(HY).

6. Where are 4 types of collagen found?


- Type 1 - bone (bone) - very high tensile strength
- Type 2 - cartilage (cartwolage)
- Type 3 - granulation tissue, embryonic tissue - very pliable/moldable
- Type 4 - basement membrane

7. How do regeneration and repair happen? What are some key molecules?
- They occur by paracrine signaling via growth factors. Some of them are
○ Fibroblast growth factor (FGF) - powerfully induces angiogenesis and skeletal development
○ VEGF - angiogenesis
○ PDGF - induces growth of endothelium, smooth muscle and fibroblast
○ TGF alpha - epithelial and fibroblast growth factor
○ TGF beta - inhibits inflammation, important fibroblast growth factor

8. Contrast wound healing by primary vs secondary intention.


- In primary intention, wound edges are brought together and there's minimum scar formation
- In secondary intention, edges are far away and granulation tissue fills the gap. Due to presence of
myofibroblasts, wound shrinks quite a lot.

9. Explain formation and structure of collagen.


- Collagen is formed as multiple alpha strands inside cells. Multiple alpha strands intertwine to
make procollagen and come outside cell.
- Structure of alpha strand is Gly-X-Y, where X and Y are proline and lycine.
- In extracellular space, multiple procollagen are crosslinked via hydroxyl groups in proline and
lycine to make collagen.

Inflammation Page 7.2


lycine to make collagen.
- Vitamin C is needed to hydroxylate proline and lysine.
- Crosslinking is done by lysyl oxidase which has copper as cofactor

10. What are some common causes of delayed wound healing? (HY)
- Infection is most common cause
- Vitamin C deficiency - causes poor hydroxylation of lycine and proline.
- Copper deficiency - lysyl oxidase can't cross link procollagen well
- Zinc deficiency - type III collagen can't be converted to type I by collagenase
- Foreign body, ischemia, diabetes, malnutrition etc

11. What is wound dehecence?


- Rupture of wound - commonly seen after abdominal surgery

12. What is hypertrophic scar


- Scar tissue being big but localized to wound. Caused by excess type I collagen

Fig- hypertrophic scar

13. Describe keloid (HY)?


- Scar tissue way out of proportion to the wound.
- Caused by excess type III collagen
- More common in african american
- Classically affects earlobes, face, and upper extremities.

Inflammation Page 7.3


Chapter 3: Principles of Neoplasia
3.1 Neoplasia

1. Define the following


Neoplasia - Neoplasia is cell growth that is monoclonal (arise from single cell),
unregulated, and irreversible.
- Contrast to hyperplasia (ex – hyperplasia of uterus during childbirth) which
is polyclonal and is regulated.
Desmoplasia - Process by which tumor becomes firm (it is because neoplastic cells induce
fibroblasts within the stroma to form abundant collagen)
Carcinoma in - (aka high-grade dysplasia) - when dysplastic changes involve entire
site thickness of epithelium (earliest form of epithelial malignancy)

2. What are two ways to determining monoclonality?


- G6PD or androgen receptor isoforms,
- Ig light chain phenotype for lymphomas.

3. How is monoclanity determined from G6PD?


- G6PD is a protein with multiple isoforms which is encoded in X-chromosome. Let’s say a
female has isoforms A and B in her 2 x chromosomes. Due to random silencing of X-
chromosomes, her A:B protein ratio in hyperplasia or normal condition will be 1:1. If the
ratio is different from 1:1, it strongly suggests monoclonality.

4. How is monoclonality determined from Ig light chain phenotype?


- The ratio of K to Lambda light chain is 3:1. If the ratio is different than this, it suggests
monoclonality

6. What are differences between benign and malignant neoplasm?


Benign neoplasia Malignant neoplasia
Remain localized and do not Invade locally and have potential to metastasize (don't
metastasize have to be already metastasized)
Slow-growing Rapid growing
Distinct Infiltrative
Mobile Fixed to surrounding tissue

7. Name the lineage of following benign and malignant tumors:


Cell lineage Benign Malignant (cancer)
Epithelium Adenoma Adenocarcinoma
Papilloma Papillary carcinoma
Mesenchyme Lipoma Liposarcoma
Angioma Angiosarcoma
Chondroma Chondrosarcoma
Osteoma Osteosarcoma
Lymphocyte doesn't exist Lymphoma
Melanocyte Nevus (mole) Melanoma (not melanosarcoma)

Neoplasia Page 1.1


Adenoma = tumor that makes glands
Papilloma = tumor that makes papillary finger like structures

8. What are leading cause of death in adults in children? (HY)


Adults Children
1. Cardiovascular disease 1. Accidents
2. Cancer 2. Cancer
3. Cerebrovascular disease 3. Congenital defects

9. What are leading cancers by incidence and death toll in male and females?
Cancer by incidence Male 1. Prostrate 2. Lung 3. Colorectal
Female 1. Breast 2. Lung 3. Colorectal
Cancer by death tolls Male 1. Lung 2. Prostrate 3. Colorectal
Female 1. Lung 2. Breast 3. Colorectal
- Table excludes squamous cell and basal cell carcinoma of skin - very common and
malignant but rarely metastasize. Detected early and easily treatable.
- Lung cancer doesn't have easy screening test like others. It's usually detected late.

10. What are some properties of neoplasia?


- Approximately 30 divisions before earliest symptoms arise (2^30 cells)
- Subsequent divisions results in increased mutations
- Cancers that don't produce symptoms till late (ex-ovarian, pancreatic, lung cancer- because
lots of space to expand) will have accumulated tons of mutations and hence poor prognosis

11. What area goals of cancer screening?


- Catch dysplasia before it becomes carcinoma (dysplasia is reversible)
- Detect carcinoma before clinical symptoms
Tests Detection
Pap smear - Cervical intraepithelial neoplasia
Mammography - Breast cancer
- Ductal carcinoma in-situ
PSA (prostrate specific - Prostrate cancer (usually grow on rectal side and doesn't
antigen) and DRE (digital produce urinary syndrome until late; BPH grows centrally
rectal exam) and produce urinary syndrome)
Hemoccult test and - Colorectal cancer
colonoscopy - Detect blood in stool (hemoccult test)

Neoplasia Page 1.2


3.2 Carcinogenesis (Part 1)

1. What do carcinogens do? What are examples?


- They damage DNA. Ex - chemicals, viruses, radiation.

2. What are cancer associations of the following chemicals? (HY)


Chemicals Cancer Remarks
Aflatoxins - Hepatocellular carcinoma (most common cancer - Derived from Aspergillus
in some African countries) flavus;
- Usually contaminates
stored grains
Alkylating - Leukemia - Found in chemotherapy
agents - Lymphoma drugs
Alcohol - Squamous cell carcinoma (SCC)of oropharynx and
upper esophagus(another risk is tobacco)
- Pancreatic carinoma (EtOH causes chronic
pancreatiis that incrases risk of cancer)
- Hepatocellular carcinoma
Arsenic - SCC of skin (women used to apply Arsenic to be
fair-skinned) - test Arsenic poisoning from
fingernail and hair follicles
- Lung cancer (Arsenic present in cigarettes)
- Angiosarcoma of liver
Asbestos - Lung cancer (far more likely) and mesothelioma
Cigarette - SCC of oropharynx and esophagus - MOST COMMON
- Lung cancer CARCINOGENIC
- Kidney and bladder cancer (urithelial carcinoma - WORLDWIDE
cells that line urinary tubes; most IMP risk factor: - Polycyclic hydrocarbons
cigerrate - toxins from smoking in urine irritate main carcinogen
the cells)
Nitrosamine - Stomach carcinoma (intestinal type) (other - Found in smoked food;
stomach carcinoma is diffuse type but responsible for high
nitrosamine not associated) stomach cancer in Japan
Napthylaine - Urothelial carcinoma of bladder - Derived from cigarette
smoke (excreted by
urine)
Vinyl chloride - Angiosarcoma of liver - Occupational exposure
(used to make PVC pipes)
Nickel, - Lung cancers - Occupational exposure
chromium,
beryllium, or
silica
- 1 Billion people smoke worldwide

3. What are ongogenic association of the following viruses?


Virus Cancer type
EBV - Nasopharyngeal carcinoma (Classic pt - Chinese male, African person) -

Neoplasia Page 2.1


EBV - Nasopharyngeal carcinoma (Classic pt - Chinese male, African person) -
metastasizes early; classic presentation is neck mass
- Burkitt lymphoma (Classic pt - African kids)
- CNS lymphoma in AIDS
HHV-8 - Kaposi sarcoma (tumor of endothelial cell) (Classic pt - 1. older Eastern
european males - take tumor out; 2. AIDS pt - treat HIV; 3. transplant pt -
reduce immunosuppression)
HBV and HCV - Hepatocellular carcinoma
HTLV-1 - Adult T-cell lukemia/lymphoma
HPV (type 16, - SCC of anogenital area (vagina, vulva, cervix, anus)
18, 31, 33) - Adenocarcinoma of cervix

4. What are cancer association of ionizing and non-ioninzing radiation?


Radiation type Cancer types MOA
Ionizing (nuclear - AML - Generation of hydroxyl free radical
reactor, - CML
radiotherapy) - Papillary thyroid carcinoma
(Cherbonyl kids have lots of
papillary cancer)
Nonionizing (UBV - Basal cell carcinoma of skin - DNA damage (formation of too much
from sun most - SCC of skin pyrimidine dimers to be excised by
common) - Melanoma of skin restriction endonuclease)
- In xeroderma pigmentosum, restriction endonuclease is bad and pt has high risk of BCC, SCC and
melanoma of skin

5. What is most common cause of ionizing radiation in USA?


- Radon (formed by decay of uranium) - present in soil and accumulates in closed space (basement)
○ 2nd most common cause of lung cancer in USA

Neoplasia Page 2.2


3.2 Carcinogenesis (Part 2)
1. What are three systems to be disturbed in carcinogenesis? (HY)
- Proto-oncogene
- Tumor suppressor genes
- Regulator of apoptosis

Protooncogenes
2. What are protooncogenes and how can they cause cancer?
- Proto-oncogenes are essential for regular cell growth and differentiation
- Mutation produces oncogenes that leads to unregulated cell growth

3. What are examples of protooncogenes?


- Growth factors and their receptors
- Signal transducers
- Nuclear regulators (transcription factors)
- Cell cycle regulators (move cell division stuff around in cell)

- Normally, binding of growth factors to its receptor induces signal transduction. In


nucleus, cell cycle regulators are activated that induce cycling of cells through G1 - S -
G2 - M stages of cell division

4. What cancer are the following protooncogenes associated with and what's their
function and mechanism of causing cancer?
Cancer Function of MOA of cancer
protooncogene
Growth factor
PDGFB - Astrocytoma - Platelet derived Overexpression,
growth factor autocine loop
Growth factor
receptors
ERBB2 - Subset of breast Epidermal growth Amplification
(HER2/neu) cancer factor receptor
RET - MEN2A, MEN2B Neural growth factor Point mutation
- Sporadic MTC receptor
KIT - Gastrointestinal Stem cell growth Point mutation
stromal tumor factor receptor
Signal
Transducers
RAS gene family - Carcinoma GTP-binding protein Point mutation
- Melanoma
- Lymphoma
- (Found in ~70% of
all cancers)
ABL - CML Tyrosine kinase Translocation - t(9,22)
- Some ALL with BCR
Nuclear
regulators
C-MYC - Burkitt lymphoma Transcription factor T (8,14) involving IgH

Neoplasia Page 3.1


C-MYC - Burkitt lymphoma Transcription factor T (8,14) involving IgH
(heavy chain)
N-MYC - Neuroblastoma Transcription factor Amplification
L-MYC - Lung carcinoma Transcription factor Amplification
(small cell)
Cell-cycle
regulators
CCND1 (cyclin Mantle cell Cyclin T(11, 14) involving IgH
D1) carcinoma
CDK4 Melanoma Cyclin dependent Melanoma
kinase

Neoplasia Page 3.2


3.2 Carcinogenesis (Part 3), 3.4 Clinical Characteristics

1. Differentiate benign and malignant neoplasia.


Benign neoplasia Malignant neoplasia
Remain localized and do not Invade locally and have potential to metastasize (don't
metastasize have to be already metastasized)
Slow-growing (years) Rapid growing (weeks/months)
Distinct (well localized, ex - can isolate Infiltrative (can't distinguish from surrounding breast
by breast exam) tissue in physical exam)
Mobile (ex - you can grab a breast Fixed to surrounding tissue
tumor and move it around)
- Classification of benign vs malignant requires biopsy

2. What are histologic differences of benign and malignant tumor?


Benign neoplasia (well-differentiated) Malignant tumor (poor differentiation)
- Organized growth - Disoganized growth (doesn't look like tissue it's
growing in)
- Uniform nuclei - Nuclear pleomorphism with hyperchromasia (very
dark blue)
- Low nuclear to cytoplasm ratio (more - High nuclear to cytoplasm ratio (less cytoplasm, big
cytoplasm) nuclei)
- Minimum mitotic activity - High mitotic activity
- Lack of invasion - Invasion
- No metastatic potential

Fig: Follicular thyroid adenoma (left) - note organized cell growth with colloid in between cells,
uniform nuclei with lots of cytoplasm, minimum mitotic activity. Anaplastic thryoid carcinoma
(right) - disorganized growth (cells look nothing like thyroid), large nuclei:cytoplasm ratio, nuclear
pleomorphism, mitotic figure (very dark nuclei cell).

3. What's the absolute distinguishing feature between benign and malignant tumors?

Neoplasia Page 4.1


3. What's the absolute distinguishing feature between benign and malignant tumors?
- Potential to metastasize (benign tumors never metastasize, malignant tumors can metastasize)

3.5 What are some ways of identifying cell types in tumor?


- Immunohistochemistry
- Serum tumor markers

4. What is intermediate filament? How is immunohistochemistry of intermediate filament helpful in


grouping cancer? (HY)
- Intermediate filament are a type of cytoskeletal protein; different intermediate filament are
present in different cell types. -
- Immunohistochemistry of intermediate filament helps in identifying cell types in tumor.
Cell type Intermediate filament present
Epithelium Keratin
Mesenchyme (connective tissue) VImentin
Muscle Desmin
Neuroglia GFAP
Neurons Neurofilament

5. What immunohistochemical molecules help identify cancer of following cell types?


Cell type Immunohistochemistry
molecule
Prostrate PSA (prostrate surface antigen)
Breast epithelium ER (estrogen receptor)
Thyroid follicular cells Thyroglobulin
Neuroendocrine cells (small cell cancer of lung and carcinoid Chromogranin (HY)
tumor)
Melanoma S-100
- Well differentiated neuroendocrine tumor = carcinoid tumor. Poorly differentiated neuroendocrine
tumor = small cell cancer.

6. What are serum tumor markers and what's their utility?


- They are proteins released by tumors.
- Useful for
○ Screening (ex- high PSA) - still need biopsy for diagnosis
○ monitoring response to treatment (ex- PSA should drop after prostrate cancer surgery)
○ recurrence of tumor (ex - same pt returns 2 year later with bone pain; check for PSA to see if
it's prostrate cancer again)

7. What does tumor grading of well differentiated and poorly differentiated mean?
- Well differentiated - tumor resembles parent tissue - good prognosis
- Poorly differentiated - tumor doesn't resemble parent tissue - poor prognosis
- Look at cellular architecture and nuclear morphology to classify differentiation

8. What's staging of cancer? What's its value?


- Staging of cancer is based on size and spread
- It's no. 1 prognostic factor (more important than grade)
- Determined after final resection of tumor

Neoplasia Page 4.2


9. What's TNM staging of cancer?
- T = tumor size and depth (size important for solid organ tumor, and depth important for tubular
organs like colon)
- N = spread to regional lymph nodes (2nd most important prognostic factor)
- M = metastasis (no. 1 prognostic factor)

Neoplasia Page 4.3


3.3 Tumor progression

1. What are the steps in invasion and spread of tumor?


- Downregulation of e-kedherin (e-kedherin is an adhesion molecules that keeps epithelial cells
attached to each other)
- Tumor cells attach to laminin in basement membrane
- Tumor cells produce collagenase that destroys collagen 4 in basement mebrane. This helps tumor
pass through basement membrane
- Tumor cells bind to fibronectin in extracellular membrane and spread locally. It can now pass to
blood vessel or lymphatics.

2. How do metastatic cancer spread?


- Lymphatic spread is characteristic of carcinomas and less common for sarcoma.
- Hematogenous spread is characteristic of sarcoma and some carcinoma - usually follows venous
drainage and seeds at first capillary bed encountered
- Seeding of body cavities - most common is peritoneum (ex - by ovarian carcinoma (omental
caking)) but may also occur in the pleural or pericardial cavities, the subarachnoid space, and the
joint spaces

Fig: omental caking by ovarian carcinoma

3. What are carcinomas that spread hematogenously?


- Renal cell carcinoma - renal vein
- Hepatocellular carcinoma - hepatic vein
- Follicular thyroid carcinoma
- Choriocarcinoma (placental cancer - of trophoblast cells - function of placenta is to invade blood
vessels)

Neoplasia Page 5.1


Chapter 4: Hemostasis and Related
Disorders
CMAP summary

Hemostasis Page 1.1


4.1 Primary Hemostasis

Primary hemostasis - make a platelet plug


Secondary hemostasis - make fibrin mesh by coagulation cascade to stabilize platelet plug

1. What are steps of primary hemostasis?


- Vasoconstriction
- Platelet adhesion
- Platelet activation
- Platelet aggregation

2. What causes vasoconstriction?


- Vasoconstriction - mediated neurally and chemically. When endothelium is disrupted, endothelial
cells secrete endothelin for vasoconstriction. NO and prostacyclin production goes down (they
vasodialate).
- Endothelin - powerful vasoconstrictor
- NO and prostacyclin (prostaglandin I2)- vasodialator

3. What happens in platelet adhesion?


- Platelet adhesion - VWF (von willieband factor) is a glue that binds to exposed collagen in
damaged tissue. It then binds to glycoprotein VIII, and then glycoprotein IB (integrin IB) on
platelet. VWF mainly comes from endothelial cells storage unit - Weibel Palade body (HY), but are
also found in alpha granules platelets. (another important factor in weibel palade body is P-
selectin)

4. What happens in platelet activation (aka degranulation)?


- Platelet changes shape after adhesion and is activated. It secretes-
○ thromboxane A2 (very powerful degranulator made by cyclooxygenase (COX)),
○ alpha granules (contain fibrinogen and VWF),
○ delta granules (contain SAC - serotonin for vasoconstriction, ADP which is necessary for
expression of GP IIB/IIIA receptor by platelet and stimulates degranulation, and calcium that
helps in secondary hemostasis)

5. What happens in platelet aggregation?


- A single fibrinogen is linked to multiple platelets via their glycoprotein IIB/IIIA. This creates platelet
plug. Platelet plug is weak and needs secondary hemostasis (coagulation cascade) for stabilization.

6. What are 2 classification of platelet disorders?

Hemostasis Page 2.1


6. What are 2 classification of platelet disorders?
- Qualitative disorders - bad platelet
- Quantitative disorders - good platelet but too few in numbers (petechiae (spots in skin due to
bleeding) seen in thrombocytopenia and not usually in qualitative disorder)

6. What are common features of pt with problem in primary hemostasis ?


- Key sign is mucosal and skin bleeding; intracranial bleeding with severe thrombocytopenia (low
platelet) (can kill you).
- Mucosal bleeding - epistaxis (nose bleeding), hemoptysis (coughing blood), GI bleeding,
hematuria, and menorrhagia (heavy menses)
- Skin bleeding - petechiae (bleeding spots on skin), purpura (>3mm bleeding spots), ecchymoses
(>1cm), easy bruising.

7. What are some useful lab studies for platelet?


- Platelet count (normal is 150k-400k per microliter of blood)
- Bleeding time - prick patient and see how long it takes to stop bleeding - normal is 2-7 minute
- Blood smear - look under microscope
- Bone marrow biopsy - look at megakaryocytes (they make platelets)

Quantitative disorders
8. What is idiopathic thrombocytopenic purpura (ITP)?
- Most common cause of thrombocytopenia in kids and adults. Autoimmune production of IgG
against platelet antigens (ex- GP IIB/IIIA)
- Antibodies are made in spleen, and tagged platelets are phagocytosed by spleen macrophages.
Low platelets cause purpura (bleeding spots in skin)

9. Describe acute form of ITP?


- Seen in children weeks after viral infection or immunization.
- Self-limited disorder - resolves within weeks of presentation

10. Describe chronic form of ITP?


- Classically seen in child bearing age woman
- May be primary (idiopathic) or secondary (strong association with lupus- remember antibodies
against blood is a common feature in lupus)
- IgG can cross placenta and cause short-lived thrombocytopenia in children

11. What do we see in labs in ITP?


- Platelet count <50K/microliter
- Normal PT/PTT
- Hyperplasia of megakaryocytes on bone marrow biopsy

12. What are treatment choices of ITP?


Corticosteroids Kids respond well to corticosteroids, adults may show early response but
usually relapse

Hemostasis Page 2.2


usually relapse
IVIG (IV Used for symptomatic bleeding. spleen macrophages will eat them instead
immunoglobulins) of immunoglobulins bound to platelets.
Splenectomy for Recall that spleen is the source of Ab production and site of phagocytosis
severe cases

Microangiopathic hemolytic anemia


11. What is microangiopathic hemolytic anemia (hemolysis in small blood vessels)? What 2 disease is
it commonly seen in?
- Platelet microthrombus are formed in small blood vessels that cut RBC as it passes through. This
results in hemolytic anemia with Schistocytes aka helmet cells (cut RBC).
- Thrombocytopenia is seen because of formation of tons of microthrombi.
- It's a symptom commonly seen in thrombotic thrombocytopenic purpura (TTP) and hemolytic
uremic syndrome (HUS)

Fig - schistocyte - the two pointy things at the end is classic appearance of cut RBC

12. What is thrombotic thrombocytopenic purpura?


- There are lots of thrombus in blood vessels that result in low platelet (thrombocytopenia) and
purpura (skin bleeding due to low platelet).

13. What causes thrombotic thrombocytopenic purpura (HY)?


- Caused due to deficiency ADAMTS13 enzyme. ADAMTS13 normally cleaves VWF multimers into
smaller monomers for eventual degradation. Bad ADAMTS13 = too much VWF = more platelet
adhesion resulting in microthrombi.
- Low ADAMTS13 is usually due to acquired autoantibody and most commonly seen in adult
females

13. What is hemolytic uremic syndrome (HUS)?


- Platelet microthrombi causes hemolysis. This mainly occurs in kidney leading to uremia and in
brain.

14. What causes HUS (HY)?


- Classically seen in children with E Coli 015:H7 dysentery - results from exposure to undercooked
beef. Dysentery = bloody diarrhea
- E Coli verotoxin damages endothelial cells in kidney and also reduces ADAMTS13 resulting platelet
microthrombi

15. What are clinical findings of TTP and HUS?


- Skin and mucosal bleeding
- Microangiopathic hemolytic anaemia

Hemostasis Page 2.3


- Microangiopathic hemolytic anaemia
- Renal insufficiency - predominant problem in HUS
- CNS abnormality - predominant problem in TTP

16. What do we see in labs in TTP and HUS?


- Thrombocytopenia with increased bleeding time
- Normal PT/PTT
- Anemia with schistocytes
- Increased megakaryocytes on bone marrow biopsy

17. How do we treat TTP?


- Plasmapheresis (process by which we remove protein from blood - removes Ab against
ADAMTS13)
- Corticosteroids - reduce production of AB.

Qualitative disorders of platelet


18. What is bernard soulier syndrome? How do we diagnose?
- Genetic GP1b deficiency; platelet adhesion is impaired
- Blood smear shows mild thrombocytopenia (platelet don't live as long because they lack GP1b)
with enlarged platelet. See big suckers in Bernard Soulier- big platelets

Fig: arrow points to big platelets in Bernard Soulier syndrome

19. What is Glanzmann thrombasthenia?


- Genetic GIIB/IIIA deficiency; platelet aggregation is impaired.

20. How do uremia affect platelet?


- It impairs adhesion and aggregation of platelets.

Hemostasis Page 2.4


4.2 Secondary Hemostasis

1. What does secondary hemostasis do?


• It creates fibrin mesh via coagulation cascade to stabilize platelet plug.
• It creates thrombin (factor IIa). Thrombin converts fibrinogen to fibrin (factor Ia). Fibrin is then
cross linked.

2. What causes disorders of secondary hemostasis?


• They occur due to factor abnormalities.

3. What is clinical presentation of disorders of secondary hemostasis?


• Deep bleeding in muscles and joints (recall that disorders of primary hemostasis has bleeding in
skin and mucosal surface)
• Rebleeding after surgical procedures

4. How do extrinsic and intrinsic pathway of coagulation cascade start? What lab tests measures intrinsic
and extrinsic pathways? What lab test is better is coumadin vs heparin?
• Intrinsic pathway - Starts with factor 12 which is activated by subendothelial collagen (SEC).
Measured by PTT lab test. Heparin results is better measured by PTT (PTT measures Hep - 3 letters
each)
• Extrinsic pathway - starts with factor 7 which is activated by tissue thromboplastin. Measured by
PT lab test. PT measures coumadin results.

Hemophilia A
9. What causes hemophilia A (think hemophilia 8)? What is it's clinical presentation?
• Caused by genetic factor 8 deficiency
• x- linked recessive.
• Can arise from new mutation without any family history
• Deep tissue, joint and post-surgical bleeding.

10. What are lab findings in hemophilia A?


• Elevated PTT, normal PT (factor 8 is a part of intrinsic pathway and PTT measures intrinsic
pathway)
• Low factor 8
• Normal platelet count and bleeding time

11. How do you treat hemophilia A?


• Give recombinant factor 8.

Hemophilia B
12. What causes hemophilia B (aka christmas disease)? What are its clinical features? What are the lab
values?
• Caused by genetic deficiency in factor 9.
• Clinically resembles hemophilia A.
• Lab values - same as hemophilia A except low factor 9 instead of factor 8.

Coagulation factor inhibitor disease


13. What is coagulation factor inhibitor disease? What factor is most commonly inhibited?
• Patient acquires antibody against coagulation factor which impairs factor function. Anti-factor 8
antibody most common.
• Clinical presentation same as hemophilia A.

Hemostasis Page 3.1


• Clinical presentation same as hemophilia A.

14. How do you distinguish hemophilia A from coagulation factor inhibitor disease in lab? (HY)
• You mix patient's plasma with normal plasma (i.e, pt gets some normal factor 8).
• If you measure PTT time after that, hemophilia A pt will have lower PTT time whereas coagulation
factor inhibitor disease patient will have same PTT time (as the antibodies will destroy new factor
8).

Von Willebrand disease


15. What is Von Willebrand disease? What is the most common type of vWF disease?
• Most common inherited coagulation disorder. (HY)
• Genetic deficiency in vWF factor (vWF required for stabilization of factor 8). Defect can be
qualitative or quantitative.
• Most common type is autosomal dominant with decreased vWF levels.

16. What are clinical presentation of vWF disease?


• Mild mucosal and skin bleeding. Even though their PTT is elevated, they don't get secondary
hemostasis disease symptoms.

17. What are lab values?


• Increased bleeding time
• Increased PTT time, normal PT
• Abnormal ristocetin test (HY)

18. Why is PTT elevated in vWF disease even though it's a platelet disease? (HY)
• PTT is elevated because vWF is required to stabilize factor 8. Bad factor8 leads to elevated PTT.

19. What is ristocetin test?


• The antibiotic ristocetin causes von Willebrand factor to bind the platelet receptor glycoprotein Ib
(GpIb), so when ristocetin is added to normal blood, it causes agglutination.
Ristocetin drug + pt blood
vWF disease - No agglutination
Bernard-Soulier (Gly Ib deficiency) - No agglutination
Glanzmann thrombasthenia - Yes agglutination
(Gly IIb/IIIa deficiency)

20. What is treatment for vWF disease? What's the mechanism? (HY)
• Desmopressin - they increase vWF release from Weibel Palade bodies of endothelial cells (primary
site of VWF storage, another substance in Weibel palade bodies = P selectin).

21. What is the function of vit K in coagulation cascade? How does coumadin work?(HY)
• They are necessary for gamma carboxylation of factor 2,7,9,10, factor c, factor s
• Vit K is activated by epoxide reductase in liver (coumadin blocks epoxide reductase and prevents
activation of vit K)

22. What causes vitamin K deficiency?


• Vit K is generated by bacteria in our gut. So deficient pt either take less vit K or have abnormal
bacteria.
○ Newborns (their gut is not well inhabited by bacteria) - newborns are prophylactically given
vit K.
○ Long term antibiotic therapy

Hemostasis Page 3.2


○ Long term antibiotic therapy
○ Malabsorption (if fat absorption is impaired)

23. Why does liver failure lead coagulation disease?


• Liver makes coagulation factors
• Epoxide reductase that activates vitamin K is found in liver

24. How to you observe liver failure patients for their coagulation function?
• By measuring their PT time (why?)

25. What does large volume transfusion lead to secondary coagulation problems?
• Because it dilutes the coagulation factors.

Hemostasis Page 3.3


4.3 Other Disorders of Hemostasis

Heparin induced thrombocytopenia


1. How does heparin induced thrombocytopenia occur? What is a complication of this?
- Heparin can bind to platelet factor 4. IgG antibodies can bind to them creating a complex. This
complex activates platelets and makes little clots in blood vessels. Resulting effect is low platelet
in blood.
- Thrombosis - Fragment of destroyed platelets can attract remaining platelet leading to
thrombosis.

2. What drug is risky to give to patients with heparin induced thrombocytopenia?


- Give other thrombolytic drug but not warfarin (coumadin). It is because cumadin can cause tissue
necrosis. MOA - coumadin also inhibits protein C in addition to inhibiting factor 2,7,9 and 10.
Protein C is an anti-clot forming factor and has shorter half-life. Therefore, coumadin acts as
procoagulant initially which leads to microthrombi in penis, skin etc and lead to necrosos.

Disseminated intravascular coagulation


3. What is disseminated intravascular coagulation disorder (DIC)? What is its clinical presentation?
- It's pathologic activation of coagulation cascade.
- Clinical presentation -
○ ischemia and infraction due to widespread microthrombi.
○ Bleeding seen from IV sites and mucosal surface (hematuria, bleeding from gut, mouth)due
to thrombocytopenia (thrombin is one major activator of platelets and can lead to
thrombocytopenia).

4. What are some causes of disseminated intravascular coagulation disorder (DIC)?


- This disease is always secondary to another disease
○ Obstetric complication - amniotic fluid contains tissue thromboplastin that can activate
factor 7 and hence the coagulation cascade.
○ Sepsis - bacterial toxins can activate coagulation cascade. Also, macrophage can produce IL1
and TNF that can activate coagulation cascade indirectly.
○ Adenocarcinoma - mucin from adenocarcinoma can activate coagulation cascade
○ Acute promyelocytic leukemia - Cells are loaded with primary granules that can coalesce to
make hour rods (classic histologic feature). Primary granules can enter circulation and
activate coagulation cascade.
○ Rattlesnake bite (HY) - venom can activate coagulation cascade

5. What are lab findings in disseminated intravascular coagulation disorder? What is the best lab
test?
- Low platelet count
- Increase PT and increase PTT
- Decrease fibrinogen
- Microangiopathic hemolytic anemia
- Elevated D-dimer (HY) - Best lab test - D dimer is product of breaking cross linked fibrin. If lots of
clots are forming, some are dissolving as well, increasing D-dimer in blood.

6. What is treatment for disseminated intravascular coagulation disorder?


- Treat underlying cause
- Transfuse blood products and cryoprecipitate, as needed

Hyperactive plasmin

Hemostasis Page 4.1


Hyperactive plasmin
7. How is plasmin formed? What are its functions? How is it inactivated?
- Normally, plasminogen is converted to plasmin by tPA (tissue plasminogen activator).
- Plasmin's function -
○ dissolves the cross-linked fibrin,
○ cleaves and destroys fibrinogen
○ destroys coagulation factors,
○ blocks platelet aggregation.
- Plasmin is inactivated by alpha-2-antiplasmin

8. What happens if plasmin is hyperactive? What are some conditions that give rise to hyperactive
plasmin?
- It prevents clot formation due to destruction of coagulation factors and fibrinogen
- Radical prostatectomy - releases urokinase that activates plasmin
- Cirrhosis of liver - it causes reduction if production of alpha 2 antiplasmin

9. What is clinical presentation of hyperactive plasmin?


- Presentation is same as DIC

10. What are lab findings in hyperactive plasmin? Contrast it with DIC lab findings.
- Increased PT and PTT
- Increased bleeding time with normal platelet count (DIC has low platelet count)
- Increased fibrinogen split products without D-dimers (low D-dimer as there's no fibrin to cleave)

11. How do you treat hyperactive plasmin conditions?


- Give aminocaproic acid. It blocks activation of plasminogen.

Hemostasis Page 4.2


4.4 Thrombosis

1. What is thrombosis? What is most common site?


- Pathologic intravascular clot that can form in any artery or vein is called thrombosis.
- Most common site is deep vein (DVT) below the knee.

2. What characterizes a thrombus from postmortem clot?


- Line of zahn
- Attachment to vessel wall

Fig - lines of Zahn (the alternate layer of RBC and fibrosis - marked by R & f) is characteristic
of thrombosis

3. What are 3 main risk factor of thrombosis (virchow's triad)? Give examples of each.
- Disruption of blood flow - stasis or turbulence of flow. Ex - immobilization, cardiac wall
dysfunction, MI, aneurism (there's turbulence in an aneurism)
- Endothelial damage. Ex - artherosclerosis, vasculitis, high level of homocysteine (vit B12 or folate
deficiency or homocystinurea)
- Hypercoagulable state (excess procoagulant or defective anticoagulant)

4. How does intact endothelium prevents thrombosis?


- It blocks exposure of tissue collagen and tissue factors from coagulation factors and platelets
- It makes prostaglandin I2 (prostacyclin) that blocks platelet aggregation. It's function is opposite of
thromboxane A2 made by platelets.
- It secretes heparin like molecules (HLM). HLM activate antithrombin 3.
- It's makes t-PA that activates plasmin.
- It makes thrombomodulin. Thrombomodulin enhances activation of protein C by thrombin.
Protein C is inhibitor of coagulation factors - 5 and 8.

5. Why does vit B12 or folate deficiency causes high level of homocysteine?
- Because vit B12 and folate are needed to convert homocysteine to methionine.
- MOA - Folate becomes tetrahydroflorate in body. It gets activated by giving its methyl group to vit
B12. It then participates in DNA synthesis. Vit B12 gives the methyl group to homocysteine and
homocysteine becomes methionine.
- High level of homocysteine is damaging to the endothelium

Homocystinurea
6. What is homocystinurea (aka cystathione beta synthase (CBS) deficiency)?
- Disease with high level of homocystine caused due to deficiency in Cystathione beta synthase
enzyme. CBS converts homocystine to cystathionine.

Hemostasis Page 5.1


enzyme. CBS converts homocystine to cystathionine.

7. What is clinical presentation of homocystinurea?


- Vessel thrombosis (1/4th of pt die in young age due to thrombosis)
- Mental retardation
- Long slender fingers
- Lens dislocation

8. What is classic presentation of thrombosis for pt with hypercoagulable state?


- Pt has recurrent DVT or DVT at young age
- Common sites are DVT, hepatic and cerebral veins.

Protein C and S deficiency


9. Describe protein C or S deficiency?
- It's a hypercoagulable state. Protein C with cofactor protein S are anticoagulants because they
inactivate factor 5 and 8.
- Pt are at increased risk for warfarin necrosis

10. What is MOA for warfarin necrosis?


- Warfarin blocks production of factors 2,7,9, 10, protein C and protein S by inactivating vit K
epoxide synthase (VKOR).
- VKOR's function is to recycle inactive vit K epoxide to active vit K. Vit K is needed to synthesize
factors 2,7,9,10, protein C and protein S.
- Protein C is anticoagulant which inactivates factor 5 and 8. It has the shortest half-life so it's the
first one to go away when warfarin is used.
- Therefore, when warfarin is use, it's first action is procoagulant due to loss of protein C. It can
make thrombus and cause necrosis.

11. Describe factor V leiden condition.


- Most common inherited hypercoagulable state.
- Patient has mutated factor V that lacks cleavage site for deactivation by protein C and S.

12. What is prothrombin 20210A condition?


- Hypercoagulable state due to inherited point mutation in prothrombin that causes increased gene
expression.

13. What is antithrombin 3 deficiency? What happens if you give them heparin? How are these
patients managed for their hypercoagulable state?
- It's a hypercoagulable state.
- Their PTT won't rise if you give normal dose heparin because heaparin works by increasing
antithrombin 3. PTT will rise a little with high doses.
- Start pt in high dose of heparin and then give warfarin.

14. Why are oral contraceptives associated with hypercoagulable state?


- It is because estrogen increases the production of coagulation factors and oral contraceptives
usually work by increasing estrogen levels.

Hemostasis Page 5.2


usually work by increasing estrogen levels.

Hemostasis Page 5.3


4.5 Embolism

1. What is embolism?
- It's a intravascular mass that travels and occludes downstream vessels.
- Thromboembolism is most common type (>95%) - embolus will have line of Zahn?.

2. What is atherosclerotic embolus?


- It's an atherosclerotic plaque that dislodges. It's characterized by presence of cholesterol
clefts in embolus.

Fig - presence of cholesterol crystal (three white columns) indicates that this embolus is
artherosclerotic and not thrombotic.

3. When does fat embolus occur? What is its clinical presentation?


- Associated with bone fractures - while the fracture is present or shortly after repair
- The embolus mostly travels to lung so classic signs are dyspnea and petechiae on skin
overlying the chest

Fig - fat embolus

4. What causes gas embolus? What is classic presentation?


- When a diver goes deep down in water, high pressure causes nitrogen to dissolve in blood.
When they come back up, the nitrogen makes bubbles causing embolus.
- It can also be caused by air pumped in laproscopic surgery
- Presents with 'bends and choke' - respiratory problem and joint, muscle pain due to embolus

Hemostasis Page 6.1


- Presents with 'bends and choke' - respiratory problem and joint, muscle pain due to embolus
in lungs, muscle and joint.

5. What is Caisson disease (aka diver's disease)?


- Chronic form of gas embolus disease. Presents with multifocal ischemic necrosis of bone.

6. What is amniotic fluid embolus?


- Amniotic fluid enters maternal circulation usually during labor and cause embolus.
- Classic presentation is SOB, neurologic symptoms due to embolus in lung and brain. Pt also
might get DIC (disseminated intravascular coagulation disease- pathologic activation of
coagulation cascade).

7. Why do patient with amniotic fluid embolus at high risk for DIC?
- Because amniotic fluid is filled with tissue thromboplastin which activates factor 7 that leads
to activation of extrinsic coagulation cascade.

8. What is characteristic finding in amniotic fluid embolus?


- Embolus will have squamous cell and keratin debris from baby's skin.

Fig - amniotic fluid embolus in a blood vessel near alveoli. Note presence of keratin debris
(squiggly lines) inside the blood vessel.

9. What is most common cause of pulmonary embolism (PE). What is its most common clinical
presentation?
- Most common due to thromembolus from DVT.
- Most PE are clinically silent as lung has dual supply from pulmonary artery and bronchial
artery. Also, the embolus are usually small and self-resolve.

9. What increases chance of pulmonary infraction from PE? What is the gross feature of
infraction?
- Obstruction of large or medium sized artery
- Pre-existing cardiopulmonary compromise
- Hemorrhagic wedge shaped infraction (hemorrhagic because tissue in lung is loosely
organized and reperfusion occurs from the dual arterial supply).

10. What is clinical features of PE in symptomatic cases?


- SOB, hemoptysis, pleuritic chest pain, and pleural effusion
- VQ mismatch with abnormal perfusion
- Spiral CT shows vascular filling defect in lung
- IMP - pt mostly have DVT and elevated D-dimer

11. What causes immediate death in PE?

Hemostasis Page 6.2


11. What causes immediate death in PE?
- Saddle embolus (embolus that blocks at branching of pulmonary artery - both pulmonary
artery blocked) leads to electromechanical disturbance in heart (too much blood backing up)
causes immediate death.

12. Explain how PE can lead to pulmonary HTN.


- When chronic emboli are reorganized over time, it leads to pulmonary HTN.

13. What is the most common source of systemic embolus? Where do they most commonly
dislodge?
- Most commonly arise from left heart. Most commonly dislodge in lower extremities.

Hemostasis Page 6.3


Chapter 5: Red Blood Cell Disorders
CMAP summary

RBC disorders Page 1.1


5.1 Anemia

1. What is anemia? What’s its presentation?


- Reduction in total circulating RBC mass
- Presentation (hypoxia):
o Pale conjunctiva and skin
o Weakness, fatigue, dyspnea
o Headache and lightheadedness
o Angina, especially with CAD

2. How is anemia measured?


- Hb, Hct and RBC count (total RBC mass difficult to measure)
- All of these measures are concentration dependent so have problems. Ex – in pregnancy, blood volume
increases making Hb and Hct concentration low even though total amount might be same. Immediately
after gunshot wound and blood loss, Hb and Hct concentration might be normal even though pt might
have lost lots of blood.

3. What is practical definition of anemia?


- Hb<13.5 g/dl for males and Hb<12.5 g/dl for females. (lower for females because of menstruation)

4. What are different types of anemia?


- Microcytic (MCV – mean corpuscle volume <80) – small RBC
- Normocytic (MCV = 80-100) – normal size RBC
- Macrocytic (MCV > 100) – big RBC

RBC disorders Page 2.1


5.2 Microcytic Anemia (Part 1) - Fe Deficiency
Tuesday, October 27, 2015 9:23 PM

1. What is pathophysiology of microcytic anemia?


- Main problem in microcytic anemia is decreased production of Hb.
- RBC is produced from subsequent division of erythroblast (EB). During Hb deficiency, EB divides too
much. As a result, RBCs become small and microcytic anemia occurs. If erythroblast doesn’t divide
enough, macrocytic anemia occurs.
- Think that by dividing extra, RBC surface area exposed to blood increases and it can carry more O2 – not
correct idea but works for thinking

Fig: microcytic anemia. Normal RBC size is equal to size of nucleus of lymphocyte. Notice that multiple
RBC are smaller than that. Also notice variability in size of RBC and increased pallor in center of RBC

2. What is hemoglobin made up of?


- Hemoglobin = heme + globin (protein).
- Heme = Iron + protoprophyrin.

3. What are etiologies of microcytic anemia (Hb deficiency)?


- Fe deficiency
- Anemia of chronic state- being unable to use Fe. In chronic inflammation, Fe is stored away in
macrophage and can’t be used.
- Cideroblastic anemia – protoprophyrin deficiency
- Thalessemia – decreased production of globin

RBC disorders Page 3.1


Fe deficiency anemia
4. What is epidemiology Fe deficiency anemia.
- Fe deficiency is the most common nutritional deficiency in the world making this the most common type
of anemia (1/3rd of world is deficient in Fe)

5. Describe digestion and storage of Fe (HY).


- Fe is absorbed in duodenum (HY). Protein called FERROPORTEIN plays a key role in Fe transport from
lumen to enterocyte to blood.
- TRANSFERRIN transports iron in blood and takes it to liver and bone marrow macrophage for storage
- Stored intracellular iron is bound to FERRITIN
- There is no real way to get rid of Iron from body. So absorption by enterocytes is regulated. (some lost
during skin sloughing off and menstruation)
- Iron is always bound to something because free Fe generates free radical by fenten reaction.

6. What are lab measurement for Fe in body?


- Serum Fe – measures Fe in blood (most of it is bound to transferretin)
- TIBC (total iron binding capacity) – tells total transferritin in blood. Normally, 1 in every 3 transferritin in
blood is bound to Fe.
- % saturation – % saturation of transferritin by Fe
- Serum ferritin – indication of how much Fe is in storage sites
- When ferritin↓, TIBC ↑ and vice versa(liver makes more TIBC to search for more Fe)

7. What are some etiologies of Fe deficiency anemia?


- Malabsorption –
o Celiac
o Gastrectomy (HY)– Fe 2+ is absorbed easily (Fe 2 goes INTO the body). Acidic environment
promotes Fe 2 conformation. When stomach is resected, due to lost acidity, more Fe will be in 3+
and Fe won’t be absorbed well.

RBC disorders Page 3.2


and Fe won’t be absorbed well.
- Other etiologies are based on blood loss of dietary lack
• Infants – breast feeding (breast milk has no Fe)
• Children – poor diet
• Adults –
▪ peptic ulcer disease (most common cause in adult males);
▪ Menorrhagia (too much bleeding during periods) or pregnancy (females).
• Elderly –
▪ colon polyps/carcinoma (western world);
▪ hookworm (developing world)

8. What are stages of iron deficiency anemia?


- Depletion of iron storage (low serum ferretin)
- Depletion of serum iron
- Normocytic anemia (HY) – first there will be few but normal sized RBC
- Microcytic, hypochromic anemia (central pallor in RBC is big)
- Know that Normocytic anemia is followed by microcytic in Fe deficiency anemia

9. What are clinical presentation of iron deficiency anemia?


- Anemia
- Koilonychia (spoon shaped nails)
- Pica (psychological drive to eat dirt – perhaps to get Fe)

10. What are lab findings of Fe deficiency anemia?


- Microcytic, hypochromic anemia with ↑RDW (RDW is like standard deviation of size of RBC; larger the
variation in RBC sizes, larger the RDW) – due to transition from normocytic to microcytic anemia
- ↓ferritin, ↑TIBC
- ↓serum iron, ↓%saturation
- ↑FEP (free erythrocyte protoporphyrin)

11. Why is there ↑FEP in Fe deficiency anemia?


- As Fe is low but protoporphyrin is normal, some protoporphyrin will be unbound to Fe hence increasing
the FEP.

12. What is blood smear finding in Fe deficiency anemia?

RBC disorders Page 3.3


- Poikilocytosis (variable shapes), anisocytosis (variable size), cigeratte shaped RBC (classic finding), tear
drop RBC
- Microcytic anemia (note RBC smaller than lymphocyte nucleus)

13. What is treatment of Fe deficiency anemia?


- Iron supplement – Ferrous sulfate
- Rule out any risk factors (ex – if old people, rule out colon carcinoma)

14. What is Plummer-Vinson syndrome?


- Iron deficiency anemia with esophageal web and atrophic glossitis (smooth tongue due to lack of white
papillae- beefy red appearance)

RBC disorders Page 3.4


5.2 Microcytic Anemia (Part 2) - Anemia of Chronic Disease

Anemia of chronic disease


1. What is epidemiology of anemia of chronic disease (ACD)?
- Most common anemia in hospitalized pt.

2. What is pathophysiology of ACD?


- During acute/chronic inflammation, acute phase proteins are produced. One of them is Hepcidin. It has
2 functions that results in anemia
o ↓Erythropoietin production
o Increased sequestering of Fe in macrophage. Less transfer of Fe to erythroid precursors will result
in Fe deficiency, which results in anemia.
- Evolutionary advantage of hepcidin is that bacteria need Fe to grow and flourish.

3. What are lab findings in ACD?


- ↑ferritin, ↓TIBC
- ↓serum iron (bone marrow takes Fe from serum as macrophage isn’t giving it), ↓% saturation
- ↑FEP (free erythrocyte protoporphyrin) (due to low available Fe but normal protoporphyrin, free
protoporphyrin will ↑)

4. What are stages of ACD?


- Normocytic anemia is followed by microcytic anemia (same as Fe deficiency anemia)

5. What is treatment of ACD?


- Treat underlying cause of chronic disease (to reduce hepcidin)
- Exogenous erythropoietin (especially helpful in cancer pt)

6. What are lab difference between Fe deficiency anemia and ACD?


Fe deficiency Anemia of chronic Hemochromato Oral contraceptive pill
anemia disease sis (OCP)
Serum Fe ↓ ↓ ↑
Transferrin/TIB ↑ ↓ ↓ ↑
C
Ferretin ↓ ↑ ↑
- Transferrin/TIBC change is always opposite of ferretin. Look at ferretin first.
- TIBC (total iron binding capacity) is a measure of Transferrin

Sideroblastic anemia (sidero = related to iron)


7. What is pathophysiology of sideroblastic anemia?
- Low synthesis of protoporphyrin is main cause.
- If protoporphyrin is deficient, Fe is trapped in mitochondria. As mitochondria surround nucleus, Fe
trapping presents as ringed sideroblasts.

RBC disorders Page 4.1


Fig: ringed sideroblasts seen in bone marrow biopsy (purssian blue stain – marks Fe)

8. What are steps of heme synthesis?

- First and last three reactions take place inside mitochondria

9. What are etiologies of sideorblastic anemia?


Congenital Acquired
Mutation of ALAS (most common cause of Alcoholism (EtOH is mitochondrial poison that damages
congenital sideroblastic anemia) - XR protoporphyrin production)
Lead poisioning (denatures ALAD and ferrocheletase)

RBC disorders Page 4.2


Vit B6 deficiency (ALAS requires Vit B6 as cofactor) –
Isoniazid treatment can cause Vit B6 deficiency

10. What are clinical features of lead poisoning leading to sideroblastic anemia?
- Pt at old house with chipped paint at high risk
- Mnemonic LEAD:
o L - Lead lines on gingivae and metaphyses of long bones (aka Burton lines)
o E - Erythrocyte basophilic stippling and encephalopathy (lead inhibits rRNA degradation causing
RBCs to retain aggregates of rRNA seen as basophilic stippling)
o A - sideroblastic Anemia, Abdominal colic
o D - Dimercaprol and EDTA for treatment; Wrist and foot Drop
o Succimer used for chelation in kids - sucks to be kid that eats lead

Fig: From right to left - basophilic stippling; Burton lines on gum; metaphysis

11. What are lab findings in sideroblastic anemia? What's its treatment?
- Lab findings based on Fe overload as they can’t attach to portoprophyrin
- As Fe increase in erythroblast, cells die due to free radical produced by Fenten reaction. Fe leaks out and
is taken by macrophage.
- ↑ferritin, ↓TIBC
- ↑serum Fe, ↑% saturation
- Very similar lab findings as hemochromatosis
- Treatment:
o Pyrodoxine (B6 - cofactor for ALAS)

RBC disorders Page 4.3


5.2 Microcytic Anemia (Part 3)

Thalassemia
1. What are normal globin molecule in hemoglobin?
Hemoglo a- b- Developmental
bin subunit subunit stage

HbF a Υ Fetal, persists for 6


moths after birth
HbA α β Adult

HbA2 α δ Adult (“Other adult”)

2. How does thalassemia lead to microcytic anemia?


- Thalassemia is decreased synthesis of globin chains. It results in reduced hemoglobin which leads to
microcytic anemia. In sickle cell, there's defect in globin chain.

3. What are patients with thalassemia protected against?


- Malaria by plasmodium falciparum.

4. What causes alpha thalassemia? What chromosome is alpha gene located in?
- Alpha thalassemia is caused due to gene deletion of alpha chain of hemoglobin.
- Normally, 4 alpha alleles are present on chromosome 16 (2 allele per chromosome)
- 'a in alpha looks like d (deletion); if you rotate b in beta, you get m (mutation).

5. What are subtypes of alpha thalassemia?


1 alpha 2 alpha allele deleted 3 alpha allele deleted - 4 alpha allele
allele HbH B chain tetramer deleted - Hb Bart
deleted Hydrops fetalis
- Asymptom - Mild anemia with slightly increased RBC - Severe anemia - Lethal in utero
atic count (remember in microcytic anemia, (hydrops
patient erythroblast divides more) fetalis)

- Cys deletion (deletion of both allele on - No problem in fetus


same chromome) is worse than trans (one good alpha allele
deletion (deletion of two allele on different takes care)
chromosome) because cys is associated
with increased risk of severe thalassemia in
offspring

- Cis deletion classically seen in Asians - When HbA and HbA2 are - Gamma chain
(higher rate of spontaneous abortion in formed, B chain tetramers(aka

RBC disorders Page 5.1


(higher rate of spontaneous abortion in formed, B chain tetramers(aka
Asia is partly due to this) tetramers (aka HbH) are Hb Barts) are
- Trans deletion classically seen in Africans formed (4 B combine formed
due to bad A)
- HbH can be seen on - Hb Barts seen
electrophoresis on
electrophoresis

6. What causes beta thalassemia? What chromosome is beta gene located in?
- Beta thalassemia is caused due to mutation of beta chain of hemoglobin. Mutations result in absent (aka
B0) or diminished (aka B+) production of B-globin chain.
- Normally, 2 beta alleles are present on chromosome 11 (1 allele per chromosome)

7. What are subtypes of beta thalassemia?


B/B+ B0/B0
- Aka Beta thalassemia minor - Aka Beta thalassemia major
- Pt usually asymptomatic with increased - Pt has severe anemia few months after birth as α4
RBC count tetramer will form (instead of α2β2)
- HbF at birth is temporarily protective (no B in fetal
hemoglobin)
- Microcytic, hypochromic RBC and target - Microcytic, hypochromic target cells and nucleated RBC
cells on blood smear
- Hemoglobin electrophoresis findings: - Hemoglobin electrophoresis findings:
• Increased HbA2 to 5% (normal • No HbA (no B chain)
2.5%)- main finding • Increased HbA2 and HbF
• Increased HbF to 2% (normal 1%)
• Slightly decreased HbA

8. What are target cells?


- Normally, hemoglobin is present mainly on edge of RBC giving it biconcave shape with central pallor. In
target cells, there are some hemoglobin in center giving central darkness (like bull's eye target practice).

- MOA- due to reduced hemoglobin in edge of RBC, the membrane in center gets floppy and some
hemoglobin comes to stay there.

9. What are presentation of beta thalassemia major?


- Bad erythropoiesis with severe anemia
- Extravascular hemolysis as spleen will phagocytose these RBC
- Massive erythroid hyperplasia
○ Expansion of hematopoiesis into skull and facial bone marrow - crew cut X-ray of skull and
chipmunk face
Extramedullary hematopoiesis - hematopoiesis in liver and spleen (hepatosplenomegaly)

RBC disorders Page 5.2


○ Extramedullary hematopoiesis - hematopoiesis in liver and spleen (hepatosplenomegaly)
○ Risk of aplastic crisis with parvovirus B19 (parvovirus affects erythrocyte precursors and shuts
down RBC production. In normal person, shutting down of RBC production for a week or so won't
matter. For pt with beta thalassemia major, they can't afford even a single day of RBC production
loss. They depend on every drop of RBC)

Fig: chipmunk face (left) and crewcut appearance (right) seen in massive erythroid hyperplasia

10. What is treatment of beta thalassemia major? What's it's complication?


- Treatment is chronic blood transfusion; splenectomy for swollen spleen and iron chelation to prevent
secondary hemochromatosis

RBC disorders Page 5.3


5.3 Macrocytic anemia
1. What is macrocytic anemia? What are it's causes?
- Macrocytic anemia is anemia with MCV (mean corpuscle volume) >100. RBC precursor doesn't
divide much and the RBC end up being big.
- Causes:
○ Megaloblastic anemia (anemia with big cells) - disruption in production of DNA precursors
results in quick cytoplasmic development relative to nuclear development:
 Folate deficiency
 Vit B12 deficiency
 Orotic aciduria
 Folate and Vit B12 needed for DNA precursor synthesis
○ Alcoholism
○ Liver disease, drugs (ex- 5-FU)

Megaloblastic anemia
Vit B12 and Folate deficiency
2. Describe relationship between folate and Vit B12.
- Folate comes to body as methylated tetrahydrofolate (M-THF).
- THF is the active form. M-THF donates it's methyl group to Vit B12. Vit B12 then gives methyl
group to homocysteine. Homocysteine now becomes methionine.

RBC disorders Page 6.1


3. What is presentation of macrocytic anemia due to folate or vit B12 deficiency?
- Megaloblastic anemia (impaired division of RBC precursors)
- Hypersegmented neutrophil with >5 lobes (normal is 3-5 lobes) (impaired division of granulocytic
precursors)
- Megaloblastic changes in rapidly dividing cells (ex - intestinal epithelial cells)

Fig: hypersegmented neutrophils with large RBC (aka macroovalocyte) on the left- classic
finding in megaloblastic anemia

4. What is difference between megaloblastic anemia and macrocytic anemia that's not
megaloblastic?
- In macrocytic anemia that's not megaloblastic, hypersegmented neutrophils and megaloblastic
changes (ex - large intestinal epithelial cells) won't be seen. Large RBC will be seen.

5. Compare dietary information of folate and vit B12.


Folate Vit B12
Food Dark green vegetable and food Animal derived proteins
Absorption Jejunum Ileum
Deficiency Develops in months as body Takes years to develop due to large hepatic
stores are minimum storage
Causes of - Poor diet (alcoholics, old) - Pernicious anemia (autoimmune destruction
deficiency - Increased demand (pregnancy, of parietal cells of stomach) - most common
cancer, hemolytic anemia) - Using proton pump inhibitor
- Folate antagonists - Pancreatic insufficiency (Vit B12 won't be free
(methotrexate - inhibits DHFR) from R-binder)
- Damage to terminal ileum (Chron's,
Diphylloborthium latum)
- Vegans (dietary deficiency rare otherwise)

6. Compare clinical and lab findings of folate and Vit B12 deficiency.

RBC disorders Page 6.2


6. Compare clinical and lab findings of folate and Vit B12 deficiency.
Folate deficiency Vit B12 deficiency
- Macrocytic RBC and - Macrocytic RBC and hypersegmented neutrophils
hypersegmented neutrophils
- Glossitis (inflammation of - Glossitis
tongue - due to less turnover of
tongue cells)
- Low serum folate - Low serum vit B12
- Increased serum homocysteine - Increased serum homocysteine (increases risk for
(increases risk for thrombosis) thrombosis)
- Normal methylmalonic acid; no - Increased methylmalonic acid in myelin cells which impairs
neuro symptoms spinal cord myelinization resulting in subacute combined
degeneration of spinal cord

7. What are two important reactions that Vit B12 participate in?
- DNA precursor synthesis (with folate)
- Conversion of methylmalonic acid to succinyl Co. A

8. Why do we see increased serum homocysteine in folate deficiency?


- Normally, dietary folate (M-THF) gives it's methyl group to Vit B12 which in turn gives it to
homocysteine. Homocysteine now becomes methionine.
- The reaction won't happen in folate deficiency and we'll see increased serum homocysteine.

9. Why will methylmalonic acid be increased in Vit B12 deficiency?


- Because Vit B12 is necessary to convert methylmalonic acid to succinyl Coenzyme A.

10. Describe absorbtion of Vit B12 in gut (HY)


- R-binder protein from saliva binds to Vit B12. The complex will travel till small bowel. There's Vit
B12 is set free by pancreatic proteases.
- The free Vit B12 binds to intrinsic factor secreted by parietal cells of stomach.
- This complex will go to ileum and get absorbed there.

11. What are 3 P's of parietal cell?


- Proton pump - they pump proton to stomach to make it acidic
- Pink in histology (chief cells appear blue)
- Pernicious anemia if they get damaged - makes IF

Orotic aciduria
Defn - Inability to convert orotic acid to uridine monophosphate (UMP) that leads to
accumulation of orotic acid (defect in de-novo purine synthesis pathway)
- AR inheritence
Presentati - Megaloblastic anemia in children refractive to folate and vit B12
on - Failure to thrive, developmental delay
- Orotic acid in urine but no hyperammonemia
Treatment - UMP to pass the mutated enzyme

Nonmegaloblastic macrocytic anemia

RBC disorders Page 6.3


Defn - Macrocytic anemia where DNA synthesis is unimpaired
- RBC macrocytosis without hypersegmented neutrophils
Causes - Alcoholism
- Liver disease
- Hypothyroidism
- Reticulocytosis

RBC disorders Page 6.4


5.4 Normocytic anemia

1. What is normocytic anemia? What are two types based on etiology?


 Normocytic anemia is decreased RBC mass with normal-sized RBC (MCV - 80-100 µm3)
 Types:
o Peripheral destruction of RBC (will have reticulocyte >3%)
 Extravascular hemolysis (RBC destroyed by liver, spleen and lymph)
 Intravascular hemolysis (RBC destroyed within blood vessel)
o Underproduction of RBC (no increased reticulocytes)

2. What are reticulocytes?


 They are young RBC released from bone marrow to replace dead RBC
 Seen as large cells with bluish cytoplasm (due to RNA) on blood smear
 Normally, 1-2% of RBC die every day and are replaced by reticulocytes.

3. How can reticulocyte be falsely elevated in anemia? How is reticulocyte count corrected?
 Reticulocytes are measured as percent of total RBC. In anemia, total RBC goes down. It will
elevating the percent of reticulocytes.
 It’s corrected by multiplying reticulocyte percent x hematocrit/45.
Total RBC Total % of HCT Corrected
Reticulocyte reticulocyte reticulocyte
Normal pt 100 2 (given) 2%
(given)
Anemic 50 (given) 2(given) 4% 23 (given) 4 x 23/50 = 2%
pt
In this example, if we only look at % of reticulocyte, it looks as if bone marrow is normal. But from
corrected reticulocyte, we know that anemic pt’s bone marrow is not producing adequate
reticulocytes.

4. How can reticulocyte count differentiate cause of anemia?


If corrected reticulocyte >3% If corrected reticulocyte <3%
Good marrow response (suggest peripheral Poor marrow response (suggest
destruction as cause of anemia) underproduction of RBC as cause of anemia)

Extravascular vs intravascular hemolysis


1. What is extravascular hemolysis? What happens to broken down RBC?
• Hemolysis done by reticuloendothelial system (macrophage in liver, spleen and lymph nodes)
• Globin is broken to AA; Iron is recycled
• Protoporphyrin is converted to unconjugated bilirubin which is carried by albumin to liver (its fat
soluble). It’s conjugated in liver and excreted to bile.

2. What are lab and clinical finding of extravascular hemolysis?


• Anemia with splenomegaly
• Jaundice due to unconjugated bilirubin (too much bilirubin to be conjugated by liver)
• High risk for bilirubin gallstones
• Marrow hyperplasia with corrected reticulocyte >3%

3. What happens in intravascular hemolysis? What's the presentation?

RBC disorders Page 7.1


3. What happens in intravascular hemolysis? What's the presentation?
• RBC is destroyed in blood vessels. Unlike macrophage breaking down hemoglobin to bilirubin,
hemoglobin simply leaks out to blood.
• Hemoglobin is carried by haptoglobin. Haptoglobin is not present a lot. So, pt will quickly have
hemoglobinemia and hemoglobinuria (hemoglobin water soluble)
• Hemosiderinuria after few days (HY) - hemoglobin in urine is picked up by renal tubular cells. Iron
is recycled back and stored as hemosiderin. Renal tubular cells slough off (just like skin cells) and
hemosiderin will be seen in urine.
• Presentation:
Immediate After few days
- Decreased serum haptoglobin - Hemosiderinuria
- Hemoglobinemia
- Hemoglobinuria

RBC disorders Page 7.2


5.5 Normocytic Anemia with Extravascular Hemolysis

Hereditary spherocytosis
1. What is hereditary spherocytosis? What are the mutations?
- In the disease, tethering proteins that attach RBC cytoskeleton to RBC membrane are mutated.
RBC membrane blebs and are lost over time. RBC becomes more spherical.
- Most common mutations are in proteins - ankyrin, spectrin, or band 3.

2. What are clinical and lab findings?


- See spherocytes - RBC becomes round instead of disc shaped (loss of central pallor)
- High RDW (some cells have lost tons of membrane and some only a little bit)
- high mean corpuscular hemoglobin concentration (MCHC) - high concentration of hemoglobin as
cells are getting small
- Extravascular hemolysis findings
○ Anemia - spherocytes can't move through splenic sinusoids well and are eaten by splenic
macrophages (this is main problem) - having spherocytes isn't bad
○ Splenomegaly (overworked spleen)
○ Jaundice with unconjugated bilirubin, high risk for bilirubin gallstones

Fig: spherocytes with high RDW (note variability in RBC sizes and loss of central pallor)

3. What is one feared complication?


- Increased risk of aplastic crisis with parvovirus B19 infection of erythroid precursors

4. How is diagnosis of hereditary spherocytosis made?


- Osmotic fragility test - cells bursts in hypotonic solution very easily because cell doesn't have
much membrane to expand out

5. What's it's treatment?


- Splenectomy (having spherocytes isn't problem, spleen eating them is problem)
- Anemia resolves but spherocytes persist and Howell-Jolly bodies are seen

6. What's Howell-Jolly bodies?


- Some RBC's are impefectly made with little nucleus or nuclear material left. It's job of spleen to
take them out or kill the defective RBC. Howell-Jolly bodies are RBC with nuclear remnant. It
indicates splenic dysfunction

RBC disorders Page 8.1


Fig: Howel Jolly bodies

Sickle cell disease


1. What causes sickle cell anemia?
• It’s caused due to mutation in B chain of hemoglobin that changes glutamic acid (hydrophilic) to
valine (hydrophobic). Think GingiVa - from Glutamic acid to Valine
• Disease is due to homozygous recessive mutation. Haterozygotes are protected against
plasmodium falciparum malaria
Phenotype Hemoglobin composition
Sickle cell disease (homozygous mutation) 90% HbS, 8% HbF, 2%HbA2, no HbA
Trait (one mutated and one normal B chain) 55% HbA, 43% HbS, 2% HbA2
• HbS – sickle cell hemoglobin (in α2β2 protein, both copies of β are mutated)

2. What is pathogenesis of sickle cell anemia?


• HbS polymerizes when deoxygenated (reversible). The polymers accumulate into needle shaped
structures and make RBC sickle cell.
• Sickling and de-sickling damages membrane leading to both intravascular and extravascular
hemolysis (spleen eats damaged RBC); sickled RBC cause vaso-occlusion; massive erythroid
hyperplasia to replace RBC.
• Sickling increases with hypoxemia, dehydration and acidosis.
• HbF protects against sickling. Kids protected for first few months of life.

3. What's treatment of sickle cell disease?


- Hydroxyurea - it increases level of HbF. It protects against sickling

4. What are presentations of sickle cell disease?


• Extravascular hemolysis – RBCs being sickle shaped and non-sickle cell repeatedly damages
membranes. Reticuloendothelial system removes these damaged RBC.
○ Anemia
○ Jaundice with unconjugated hyperbilirubinemia
○ Increased risk for bilirubin gallstones
• Intravascular hemolysis – due to membrane damage
○ Decreased haptoglobin
Hemoglobinemia, hemoglobinuria

RBC disorders Page 8.2


○ Hemoglobinemia, hemoglobinuria
○ Hemosiderinuria after few days
○ Target cells - hemoglobin leaks out due to membrane damage and extra membrane
produces target cells
• Massive erythroid hyperplasia (to compensate hemolysis and anemia):
○ Hematopoiesis in skull and facial bones (crewcut on X-ray and chipmunk face)
○ Extramedullary hematopoiesis (in liver, giving hepatomegaly - pt don't have spleen so don't
get splenomegaly)
○ Risk of aplastic crisis with parvo B19 infection

Fig: chipmunk face (left) and crewcut appearance (right) seen in massive erythroid
hyperplasia
• Extensive sickling leads to vaso-occlusion

5.What are some physical findings in sickle cell disease due to vaso-occlusion?
•All findings based on infraction
•Dactylitis – due to vasoocclusive infaracts in bones – common in infants
•Autosplenectomy – shrunken, fibrotic and calcified spleen
○ Increased risk of encapsulated organism infection (staph aureus, strep pneumo,
haemophilus influenza)
○ Salmonella paratyphi osteomyelitis (encapsulated) - most common cause of osteomyelitis
is staph aureus; in sickle cell, it's salmonella.
○ Howel-Jolly bodies on blood smear - nucleated RBC
• Acute chest syndrome (vaso-occlusion of pulmonary microcirculation)
○ Often precipitated by pneumonia
○ Presents with chest pain, SOB, lung infiltrates
• Pain crisis
• Renal papillary necrosis – presents as gross hematuria and proteinuria

Fig: vaso-occlusive complications of sickle cell disease - from left to right - autosplenectomy - small
calcified spleen; renal papillary necrosis; dactilytis; Howel-Jolly bodies

6. What’s the most common cause of death in sickle cell patients?


Kids Hemophilus influenza infection
Adults Acute chest syndrome

RBC disorders Page 8.3


Adults Acute chest syndrome

7. What is sickle cell trait?


• Haterozygote carriers of sickle cell mutation have sickle cell trait. They have one mutated and one
normal beta chain.
• HbS (both beta chain mutated) makes <50% of total hemoglobin because HbA is slightly more
efficiently made than HbS

8. What are presentations of sickle cell trait?


• Generally asymptomatic as RBC with <50% HbS don’t sickle
• Renal medulla problems:
○ Due to extreme hypoxia and hypertonicity in medulla, sickling occurs.
○ Presents as microscopic hematuria and decreased ability to concentrate urine due to
microinfraction of medulla.

9. What are lab findings in sickle cell disease and trait?


Sickle cell disease Sickle cell trait
Sickle cell and target cells seen in blood Don’t see sickle cell or target cells
smear
Metabisulfite screen +ve (cells with any Metabisulfite screen +ve
amount of HbS are sickled by the
screen)
• Confirm amount and presence of HbS with Hb electrophoresis
Sickle cell disease 90% HbS, 8% HbF, 2% HbA2, no HbA
Trait 55% HbA, 43% HbS, 2% HbA2

Hemoglobin C
1. What is hemoglobin C?
- Hemoglobin C is formed due to mutation in Beta chain of hemoglobin (autosomal recessive).
- Glutamic acid is changed to lysine (lyCne for hemoglobin C) - think Gingiva - Glutamic acid to
lyCine as gingiva is C shaped)
- Less common than sickle cell disease

2. What is presentation of hemoglobin C?


- Mild anemia due to predominant extravascular hemolysis
- HbC crystals on blood smear (HY)

Fig: HbC crystals characteristic of hemoglobin C (the rods)

Pyruvate kinase deficiency


Pathophys - RBC depend on glycolysis to synthesize ATP
- Bad pyruvate kinase means ATP deficiency that affects lots of processes in RBC

RBC disorders Page 8.4


Blood - Echinocytosis (echino means hedgehog or sea urchin) - also see this in
smear hyperlipidemia, uremia, hemolytic anemia, hypomagnesemia, hypophosphatemia
etc

- Contrast Echinocytosis which looks similar to acanthocytosis

Fig: acanthocytosis - see in hyperlipidemia or liver damage due to RBC membrane


damage

RBC disorders Page 8.5


5.6 Normocytic Anemia with Intravascular Hemolysis

Paroxymal nocturnal hemoglobinuria (PNH)


1. How do cells in blood protect themselves from complement system?

- DAF (decay accelerating factor) and MIRL (membrane inhibitor of reactive lysis) are present in
RBC, WBC and platelets. They block complement fixation in RBC. DAF decays C3 convertase.
- Protein called GPI (glycosylphophatidylinositol) anchors MIRL and DAF to cells.

2. What causes paroxysmal nocturnal hemoglobinuria?


- It's acquired (not congenital mutation) defect in myeloid stem cell so that GPI is absent in myeloid
stem cells. Complement fixation lyses RBC, WBC and platelets

3. What is presentation of PNH?


- Symptoms are seen paroxysmally at night because breathing becomes swallow and mild acidosis
activates complement at night.
- Dark urine early morning
- Hemoglobinura, hemoglobinemia
- Hemosiderinura seen few days after hemolysis (after tubular cells slough off)
- Thrombosis - due to release of clotting factors from lysed platelets

4. What is main cause of death in PNH?


- Thrombosis of hepatic, portal or cerebral veins - due to release of clotting factors from lysed
platelets

5. What are complications of PNH?


- Fe deficiency anemia (due to chronic loss of Hb in urine)
- Acute myeloid leukemia (10% of patients)

6. How is diagnosis of PNH made?


- Screening - Sucrose test
- Confirmatory test - acidified serum test or flow cytometry to test lack DAF (aka CD55) on RBC

Glucose-6-Phosphatase dehydrogenase (G6PD) deficiency - aka favism


1. What is G6PD deficiency? What's it's pathophysiology
- X linked recessive disorder (see in men) that results in low half-life of G6PD.
- G6PD is first enzyme in pentose phosphate pathway and is required to make NADPH. NADPH is
important to reduce oxidative stress.
- G6PD deficiency presents as increased oxidative stress including hemolytic anemia.

2. What are two major variants of G6PD deficiency?


African variant Mediterranean variant
- Mildly reduced half-life of G6PD - Markedly reduced half-life of G6PD
- Mild intravascular hemolysis with oxidative - High intravascular hemolysis with oxidative

RBC disorders Page 9.1


- Mild intravascular hemolysis with oxidative - High intravascular hemolysis with oxidative
stress stress

3. What protective role does being carrier of G6PD deficiency have?


- Protection against falciparum malaria

4. What's histology finding of G6PD deficiency anemia?


- Heinz bodies - precipitation of Hb due to oxidative stress
- Bite cells - Caused due to removal of Heinz bodies from RBC by macrophage

5. What are some causes of oxidative stress?


- Sulfa drugs
- Antimalarial drugs
- Fava beans

6. What's presentation of G6PD deficiency anemia?


- Hemoglobinuria and back pain (hemoglobin is nephrotoxic) hours after exposure to oxidative
stress

7. What is diagnosis of G6PD deficiency?


- Screening - Heinz preparation - need to see heinz body
- Confirm - enzymatic studies (don’t do it during acute phase because RBC lacking G6PD are already
dead).

Immune hemolytic anemia (IHA)


1. What causes immune hemolytic anemia?
- IgG or IgM mediated destruction of RBC.

2. Differentiate between IgG vs IgM mediated IHA.


IgG mediated IHA IgM mediated IHA
- Hemolysis is usually extravascular - tagged RBC are eaten - Hemolysis is usually extravascular-
tagged RBC are eaten
- Warm agglutination - IgG binds to RBC in warm temp - Cold agglutination - IgM binds to
(central parts of body). RBC in cold temp (extremities).
- Splenic macrophage phagocytose tagged RBC leading to - RBC can inactivate complement, but
formation of spherocytes (when RBC are eaten only C3b acts as opsonin for splenic
halfway, remaining RBC makes sphere) macrophages - see spherocytes

- Associated with: - Associated with:


- Lupus - pt have anti-blood Ab • Mycoplasma pneumoniae

RBC disorders Page 9.2


- Lupus - pt have anti-blood Ab • Mycoplasma pneumoniae
- CLL (chronic lymphocytic lukemia) - cause hemolytic (cold agglutination test)
anemia • infectious mononucleosis (+ve
- drugs (classically penicillin and cephalosporins) - drug haterophile agglutination - Ab
induces autoantibody production or Ab can bind to drug- made against sheep blood)
RBC complex • CLL
- Treatment
• Stop offending drug
• Steroids
• IVIG (distract spleen)
• Splenectomy - spleen is the one that eats RBC

3. How do you diagnose IHA?


Direct coombs test Indirect coombs test
- Confirms presence of Ab or complement coated - Confirm presence of anti-RBC Ab in
RBC patient's blood
- When anti-IgG or anti-complement Ab are added - Anti IgG and test RBC(normal RBC) are
to pt RBC, agglutination occurs only if RBC are mixed in patient serum (agglutination
already coated with IgG or complement occurs only if serum Ab are present)
- Most important test for IHA

Microangiopathic hemolytic anemia


1. What is microangiopathic hemolytic anemia (hemolysis in small blood vessel)?
- It's hemolysis that occurs due to vascular pathology (usually something in blood vessel breaks the
RBC)

2. What are some etiologies?


- Presence of microthrombi
○ TTP- thrombotic thrombocytopenic purpura
○ HU- hemolytic uremic syndrome
○ DIC
○ HELLP - hemolysis elevated liver enzyme and low platelet
- Prosthetic heart valves - crush RBC
- Aortic stenosis - crush RBC

3. What is blood smear finding?


- Schistocytes (broken RBC) - aka helmet cells

Fig - schistocytes (helmet cells) - has mostly two acute angle and loss of about 50% of RBC;
contrast bite cells that have usually >2 acute angles and almost entire volume of RBC is present.

Malaria
1. How does malaria cause anemia?
- Plasmodium infects and replicates in RBC. RBC ruptures as merozoites (a stage in their lifecycle)

RBC disorders Page 9.3


- Plasmodium infects and replicates in RBC. RBC ruptures as merozoites (a stage in their lifecycle)
are released
- Spleen also consumes infected RBC causing some extravascular hemolysis

Erythroblastosis fetalis
Defn - Maternal IgG crossing placenta and attacking fetal RBC (ex - Rh -ve mother carrying
two consecutive Rh +ve babies)
Present - See extramedullary hematopoiesis (ex - in liver) because RBC are damaged

RBC disorders Page 9.4


5.7 Normocytic Anemia due to Underproduction

1. What is anemia due to underproduction?


- It's anemia caused due to low RBC production by bone marrow.
- Characterized by low corrected reticulocyte (<3%)

3. What are some etiologies of anemia due to underproduction?


- Renal failure - decreased erythropoietin production by peritubular interstitial cells
- Anything that causes microcytic and macrocytic anemia
- Damage to bone marrow precursor cells - ex parvovirus B19

4. Describe how parvovirus B19 infection leads to anemia.


- Parvovirus B19 infects progenitor RBC and temporarily halts erythropoiesis.
- It causes significant anemia in setting of preexisting marrow stress (ex - sickle cell)
- Treatment is supportive (infection is self-limited)

Aplastic anemia
1. What is aplastic anemia?
- Aplastic anemia is damage to hematopoietic stem cell resulting in pancytopenia (anemia,
leukopenia, thrombocytopenia)

2. What are etiologies of aplastic anemia?


- Etiologies:
○ Drugs or chemicals, radiation
○ Viral infection - parvo B19, HIV, EBV, HCV
○ Autoimmune damage
○ Fanconi anemia (inherited DNA repair defect that causes bone marrow failure) - high risk of
leukemia later

3. What are biopsy finding in aplastic anemia?


- Empty fatty marrow

Fig: Aplastic anemia (left) vs normal bone marrow on right. Note the depletion of marrow and
replacement by fat globules on left.

4. What's treatment for aplastic anemia?


- Immunosuppression for cases with abnormal T cell activation
- Stop causative drugs
- Blood transfusion and marrow stimulating factors (erythropoietin, GM-CSF, G-CSF)
- May need bone marrow transplant

Myelophthisic process
1. What are melophthisic process?
- Pathologic processes that replace bone marrow (ex - cancer)
- Hematopoiesis is impaired resulting in pancytopenia

RBC disorders Page 10.1


Lymphoid tissue anatomy

1. Spleen
- PALS (periarteriolar lymphatic sheath) - has T cells (drink tea with pals) - in white pulp
- Germinal center - has B cells - in white pulp

RBC disorders Page 11.1


Chapter 6: White Blood Cell Disorders
CMAP summary

WBC disorders Page 1.1


Chromosome with genes

1. Ch 8 - c-myc ( protein in Ras- MAPK pathway)


2. Ch 11 - B globulin of Hb (1 allele per chromosome), Cyclin D1
3. Ch 14 - Ig heavy chain
4. Ch 15 - retinoic acid receptor
5. Ch 16 - alpha globulin of Hb (2 alleles per chromosome)
6. Ch 18 - Bcl2 (anti-apoptotic protein)

WBC disorders Page 2.1


6.1 Leukopenia and Leukocytosis

1. Draw out the development of different blood cell lines.

2. What happens in leukopenia or leukocytosis?


- Leukopenia - low WBC (penia = lack of)
- Leukocytosis - high WBC
- Usually, increase or decrease is in one particular cell line

Leukopenia
1. List the differences between neutropenia and lymphopenia.
Neutropenia Lymphopenia
Decreased circulating neutrophils Decreased circulating lymphocytes
Causes: Causes:
- Drug toxicity- Chemotherapy - Damage - Immunodeficiency - HIV, DiGeorge syndrome
stem cell and decreased production of - High cortisol - Induces apoptosis of lymphocytes
WBC, specially neutrophil - Autoimmune destruction - Lupus
- Severe infection- Neutrophils move to - Whole body radiation - Lymphocytes are highly
tissue sensitive to radiation; lyphopenia is earliest change to
- Lupus emerge after whole body radiation

Treatment: GM-CSF or G-CSF


*Corticosteroids cause neutrophilia but lymphopenia and eosinopenia. They cause sequestering of eosinophils in
lymph nodes, induce apoptosis of lymphocytes, and impair adhesion of neutrophils to vessel wall.
Leukocytosis
1. What are the causes of leukocytosis.
- Leukocytosis is increased WBC count.
Causes
Neutrophilic - Bacterial infection or tissue necrosis (also see left shift which have decreased Fc receptors
leukocytosis (CD16)
- High cortisol state - impairs neutrophil adhesion; marginated pool is released to blood
(marginated pool - leukocytes that are attached to vessel wall and hang there like bats)
Monocytosis - Chronic inflammatory state (autoimmune and infectious)
- Malignancy
Eosinophila - Allergic reaction (type I hypersensitivity)
- Parasitic infections
- Hodgkin lymphoma (increased IL5)
Basophilia - Classically seen in chronic myeloid leukemia (CML) - (HY)

WBC disorders Page 3.1


Basophilia - Classically seen in chronic myeloid leukemia (CML) - (HY)
Lymphocytic - Viral infection
leukocytosis - Bordetella pertussis (whopping cough)- (exception because bacteria usually increase
neutrophil)

Infectious mononucleosis
1. What causes mono?
- EBV (most common), CMV (less common)

2. What's presentation of EBV infection?


- Mono has classic triad of lymphadenopathy, fever and exudative pharyngitis
- Other:
○ T cell hyperplasia
 Lymphadenopathy due to hyperplasia in paracortex (location IMP to know)
 Splenomegaly due to hyperplasia in periarterial lymphatic sheath (PALS)
○ Hepatitis

3. Describe the screening test for mono (monospot test)?


- Test detects haterophile antibodies (haterophile = loves others) - detects IgM Ab that cross-react with horse or
sheep RBC
- Test turns positive after 1 week of infection

4. What are two cases where person has mono but monospot test is negative?
- Mono is caused by CMV (only EBV mono will have positive monospot test)
- Test is done within 1 week of infection

5. What's definitive diagnosis for EBV infection?


- Serologic test for EBV viral capsid Ag.

6. What are complications of EBV infection?


- Splenic rupture (due to splenomegaly) - avoid contact sports for 1 year
- Rash is exposed to ampicillin (pharyngitis of group A strep is treated by penicillin)
- Virus dormant in B cell:
○ Risk for B cell lymphoma, esp in immunodeficiency
○ Recurrence of mono

WBC disorders Page 3.2


6.2 Acute Leukemia

1. What defines acute leukemia? What are its types?


- Acute leukemia is presence of >20% blasts in the bone marrow (normal is 1-2%). Blasts indicate
myeloid or lymphoid stem cell (undifferentiated cells).

- Types:
○ Acute lymphoblastic leukemia (ALL) - presence of >20% lymphoblast stem cell
○ Acute myeloid leukemia (LML) - presence of >20% myeloid stem cell

2. What is presentation and clinical findings in acute leukemia?


- Normal hematopoiesis is disturbed:
○ Anemia (tired)
○ Thrombocytopenia (bleeding)
○ Neutropenia (infection)
- Note that WBC count is high due to presence of lots of blasts in blood
- Blast cell appear as large immature cells with punched out nucleoli.

Fig: blast cells with punched out nucleolus (as if punched via punching machine) - notice lots of
them (gives high WBC count)

Acute lymphoblastic leukemia


1. How do you distinguish if acute leukemia is from myeloblast of lymphoblast?
Acute lymphoblatsic leukemia +ve nuclear staining for TdT (a DNA polymerase)
Acute myelogenous leukemia +ve cytoplasmic staining for myeloperoxidase (MPO)

2. What is epidemiology of ALL?


- Most commonly seen in children
- Associated with down syndrome (usually >5 years)

3. Distinguish the two types of acute lymphoblastic leukemia.

WBC disorders Page 4.1


3. Distinguish the two types of acute lymphoblastic leukemia.
B-ALL (B-acute lymphoblastic leukemia) T-ALL (acute lymphoblastic lymphoma)
Characterized by lymphoblast (TdT+) that Characterized by lymphoblasts (TdT+) that express
express CD10, CD19, and CD20 (markers of CD2-CD8; lack CD10
B cells)
Excellent response to chemotherapy (give Excellent response to chemotherapy (give
prophylaxis dose in scrotum and CSF) prophylaxis dose in scrotum and CSF)
- Usually present in teenagers as mediastinal
(thymic) mass - cancer called lymphoma because of
cells make a mass

4. What are two types of B-ALL?


- T (12;21) - good prognosis - usually seen in children
- T (9;22) (aka Philadelphia + ALL) - poor prognosis - usually seen in adults

Picmonic for ALL

Pt popn - Affects <15 years (quincenera scene)


Prognosis - t(12;21) has better prognosis (girl with 1 wand and tutu looking at her reflection)
- Excellent response to chemotherapy - chemo girl kissing on the last figure
- Give prophylactic chemo to testes and CNS - bottom left figure guy hitting enemy
on head and testes
Markers - TDT +ve (explode with TNT)
- CALLA +ve (villan with collar - bottom middle pic)
Presentatio - Thymic mass (medistinal arrow) for T-ALL
n

Acute Myeloid leukemia


1. What is acute myeloid leukemia?
- Presence of immature myeloid cells >20% in bone marrow
- Myeloid stem cell characterized by +ve cytoplasmic staining for myeloperoxidase (MPO)
- Crystal aggregate of MPO may be seen as Auer rods

WBC disorders Page 4.2


Fig: Aure rods inidicating MPO presence (diagnosis for AML)

2. What is epidemiology of AML?


- Mostly affects old people - (50-60 years)

3. What are different types of AML?

Acute Cause - t(15;17) translocates retinoic acid receptor (RAR) to


promyelocytic chromosome 17. Bad receptor blocks maturation of
leukemia WBC and promyelocytes (blasts) accumulate.
Treatmen - All trans retinoid acid (ATRA - vit A derivative) - it
t can bind to bad retinoic acid receptor. (this signaling
important for development of myeloblasts)
Complica - DIC (promyelocytes contain numerous primary
tion granules that increase risk for DIC).

Acute monocytic ○ Blasts characterstically infiltrate gums


leukemia (MPO -
ve)

Acute - Associated with Down syndrome (usually seen <5 years)


megakaryoblastic
leukemia (MPO -
ve)

Myelodysplastic syndrome
7. What is myelodysplastic syndrome?
- Dysplasia of myeloid stem cells. It increases risk of AML.
- Presents as
○ >3% but <20% blasts (if it's >20%, it's acute leukemia)
abnormal maturation of cells and increased blasts because cells can't fully differentiate-

WBC disorders Page 4.3


○ abnormal maturation of cells and increased blasts because cells can't fully differentiate-
that's why increased risk of AML
○ Most pt die from infection or bleeding (some progress to AML)
○ Cytopenia and hypercellular bone marrow (cells are stuck in bone marrow and not going
out)

- Ex - Pseudo-Pegler-Huet anomaly - neutrophils with bilobed nuclei typically seen after


chemotherapy

Fig: Pseudo-Pegler-Huet anomaly

WBC disorders Page 4.4


6.3 Chronic Leukemia

1. What is chronic leukemia? What are it's types?


- Chronic leukemia is proliferation of MATURE circulating lymphocytes.
- It's usually seen in adults and have insidious onset.
- Types:
Cancer name Cell type high Remarks
Chronic lymphocytic Naïve B cells (CD5 - Most common leukemia overall
leukemia (CLL) and CD20 +ve) - - See smudge cell (like someone smashed a cell in slide) -
naïve kids smudge - See generalized lymphadenopathy
their food - Small lymphocytic lymphoma is a type of CLL that affects tissue

Complicati • Transformation to diffuse large B-cell lymphoma (richter


ons transformation) - see enlarging lymph nodes or spleen
• Hypogammaglobulinemia - INFECTION MOST COMMON
CAUSE OF DEATH
Associatio • Autoimmune hemolytic anemia
n
Picmonic:

CaLL - CLL
Epid - Most common leukemia in adults (old people in pic)
Presentati - Often asymptomatic - both elderly are happy and dancing
on - Smudge cell - cell smudged on glass of grandpa
Associatio - Autoimmune hemolytic anemia - both IgM and IgG - cold
n and warm sea anemone

Hairy cell leukemia Mature B cells (TRAP - Lymphadenopathy usually absent


(rare leukemia) +ve (tartarate - Marrow fibrosis (dry bone marrow tap)
resistant acid

WBC disorders Page 5.1


resistant acid
phosphatase) and
hairy cytoplasm) -
hairs trap; mature
people are hairy

Picmonic:

Hairy guy with Basketball - hairy cell leukemia


Presentati - Splenomegaly (can be the only finding) - hairy cells
on accumulate in spleen - spleen baloon
- Lymphadenopathy usually absent
Treatment - Excelent response to 2-CDA (cladribine) - cloud ribbons

Adult T cell Mature CD4+ cells - Associated with HTLV-1; common in Japan and caribbean
leukemia/lymphoma - Skin rash, generalized lymphadenopathy, hepatosplenomegaly,
- lytic (punched out) bone lesion with hypercalcemia

- Excellent treatment to chemotherapy


Mycosis Fungoides Mature CD4+ cells - See sezary cells (cerebriform nuclei) in blood smear
(cutaneous T-cell that infiltrate skin - CD4 accumulation in skin produces rash, plaques and nodules called
lymphoma) pautrier microabscess

Fig - Sezary cells, pautrier microabscss - microcospic and gross

WBC disorders Page 5.2


6.4 Myeloproliferative Disorders

1. What is myeloproliferative disorders? What's its presentation?


- Myeloproliferative disorders is neoplastic proliferation of mature cells of myeloid lineage
- Disease presents in late adulthood (50-60 years)
- Presentation:
○ Hypercellular bone marrow with increased WBC
○ All cells in myeloid lineage are increased, classification is based on what cell is dominantly
present

2. What are types of myeloproliferative disorders? What is characteristic mutation?


Disorder Dominant cell Mutation
Chronic myeloid Granulocytes (neutrophil, t(9;22) aka philadelphia
leukemia (CML) eosinophil, basophil); increased chromosome - present in 95% of pt
basophil (HY)
Polycythemia vera RBC JAK2 kinase mutation
(PV)
Essential Platelets JAK2 kinase mutation
thrombocythemia
(ET)
Myelofibrosis Megakaryocytes JAK2 kinase mutation (50% of cases)

Chronic myelogenous leukemia (CML)


1. What is the mutation that causes CML? What's treatment?
- T(9;22) produces philadelphia chromosome. It creates fusion of BCR on ch 22 with ABL receptor
tyrosine kinase protein. It leads to increased tyrosine kinase activitiy.
- Treatment
○ Imatinab - blocks tyrosine kinase activity (HY)
○ Hydroxyurea - inhibits ribonucleotide reductase (DNA synthesis) - prevent cell dividion
○ IFN alpha - increased destruction of cells
○ Hydroxyurea and IFN alpha can be used in many disease where cell divides too much

2. What is presentation and complication?


- Splenomegaly is common
- Complication:
○ Transformation to AML (2/3rd of cases)
○ Transformation to ALL (1/3rd of cases)

3. How do you diagnose CML?


- Peripheral blood smear shows leukocytosis - too much WBC
- FISH can be used to see philadelphia chromosome (seen in 95% of cases); however gene
translocation is seen in 100% of cases.

WBC disorders Page 6.1


4. How do you distinguish CML from lukemoid reaction (reactive neutrophilic leukocytosis)?
CML Lukemoid reaction
Negative leukocyte alkaline phosphatase Positive leukocyte alkaline phosphatase (LAP) stain
(LAP) stain (LAP present in normal
neutrophils but absent in malignant
neutrophils)

INCREASED BASOPHILS (HY) No increase in basophil


t(9;22) present t(9;22) absent

5. Picmonic for CML

WBC disorders Page 6.2


CML Camel
Mutation - t(9;22) - ninja and double tutu
- Makes philadelphia chromosome (philadelphia cream cheese)
- BCR-ABL fusion protein created - VCR - Abraham lincoln
Treatment - Imatinib - eye-mat

Polycythemia vera (PV)


1. What is PV?
- Increase in RBC, WBC and platelets but main presentation is based on too much RBC.

1. What are presentation of PV?


- Presentation based on increased viscosity of blood:
○ Flushed face due to congestion (plethora)
○ Intense itching, especially after bathing (due to histamine release from increased mast cells
due to heat)
○ Erythromelalgia - severe burning pain, and red-blue discoloration due to blood clots in
vessels of extremities; melalgia = buring pain in limbs
○ Hepatosplenomegaly (splenomegaly because there's too much RBC so amount of bad RBC
will be high and spleen has to eat them; hepatomegaly because liver makes RBC due to
mutation in JAK2 kinase).
○ Gout - proerythroblasts (RBC witn nucleus) will leak out and be eaten by spleen. Metabolism
of nucleus increases uric acid.
○ Venous thrombosis
 No. 1 cause of budd-chiari syndrome
 Dural sinus thrombosis
 Retinal vein thrombosis

WBC disorders Page 6.3


Fig: polycythemia presentation - from left to right: dural sinus thrombosis, Erythromelalgia, facial
plethora, itching, hepatosplenomegaly, gout

2. How do you treat PV?


- Phlebotomy (first line)
- Hydroxyurea - second line (inhibits ribonucleotide reductase - DNA synthesis)
- Interferon alpha - increased destruction of proerythroblasts (can induce gout)

3. What's prognosis of PV?


- Death within a year without treatment

4. How do you distinguish PV from reactive polycythemia?


PV Reactive polycythemia
Normal SaO2, low erythropoietin - If due to lung disease or high altitude- Low SaO2, high
(EPO) EPO
- If due to ectopic EPO production, SaO2 normal, high EPO

Essential thrombocythemia vs Myelofibrosis


1. Differentiate essential thrombocythemia vs myelofibrosis
Essential thrombocythemia (ET) Myelofibrosis
Neoplastic proliferation of platelets - Neoplastic proliferation of megakaryocytes
megakaryocytes divide too much. (megakaryocyte = cell with big nucleus)
Thrombocyte = platelet
Associated with JAK2 kinase mutation Associated with JAK2 kinase mutation
Presentation Presentation:
- increased risk of thrombosis and/or - Increased risk of thrombosis and/or bleeding,
bleeding (low platelet due to too infection
much thrombosis) - Hepatosplenomegaly - they will produce blood
- Splenomegaly - spleen has to eat cells as marrow is fibrosed (MOST COMMON
too much platelets CAUSE OF SPLENOMEGALY IN PT >55 YEAR
- Rarely progress to marrow fibrosis OLD)
or acute lukemia - Marrow fibrosis (due to PDGF production from
- Rare risk for hyperuricemia or gout megakaryocytes) will impinge on bone and

WBC disorders Page 6.4


- Rare risk for hyperuricemia or gout megakaryocytes) will impinge on bone and
(only platelets are increased which cause bone pain
don't have nucleus) - Splenomegaly due to extramedullary
hematopoiesis (due to marrow fibrosis)
- Leukoerythroblastic smear (tear drop RBC - RBC
squeezing through fibrosed marrow, nucleated
RBC, immature granulocytes)
• Enlarged megakaryocytes in bone
marrow biopsy

Fig: leukoerythroblastic smear, marrow fibrosis


- Diagnosis: - Diagnosis:
• Diagnosis of exclusion • Tear drop RBC
• See platelets >450k for >2 months • Bone marrow tap will be dry; see fibrosis in
biopsy
- Treatment: - Treatment:
• Asprin for low risk pt to prevent • Stem cell transplant - replace fibrosis
clot • Transfusion to give blood cells that’s missing
• Hydroxyurea , interferon alpha to
reduce megakaryocyte formation
• Plateletpheresis - take excess
platelet from circulation in
emergency condition
- Leukoerythroblastic anemia is anemia due to space occupying lesion in marrow

WBC disorders Page 6.5


6.5 Lymphadenopathy (LAD)

1. When do you see painful vs painless lymphadenopathy?


- Lymphadenopathy is enlarged lymph nodes.
- Painful:
○ Lymph nodes draining a region of acute infection (acute lymphadenitis)
- Painless:
○ Chronic inflammation (chronic lymphadenitis)
○ Metastatic carcinoma
○ Lymphoma

2. What are indications depending on what part of lymph node is enlarged?


- Follicular enlargement (B cell region):
○ Rheumatoid arthritis
○ Early stages of HIV
- Paracortex enlargement (T cell region):
○ Viral infection (ex - EBV)
- Sinus histiocytes enlargement:
○ Lymph nodes draining with cancer

WBC disorders Page 7.1


6.6 Lymphoma, 6.7 Hodgkin Lymphoma

1. What is lymphoma?
- Neoplastic proliferaion of lymphoid cells that makes a mass; may arise in lymph node or extranodal tissue.

2. Distinguish Hodgkin lymphoma (HL) and non-Hodgkin lymphoma (NHL).


Hodgkin lymphoma Non-Hodgkin lymphoma
Frequency 40% 60%
Malignant cells Reed-Sternberg cells (owl eye cells) - B cells - Lymphoid cells - many subtypes - most are B cell
induce inflammation and fibrosis than T cells

Mass Inflammatory cells and fibrosis (reactive cells) Lymphoid cells


composition
Clinical - Painless lymphadenopathy - Painless lymphadenopathy
- B symptoms - Classically seen in late adults
- Bimodal: classically seen in young adult
and >55 yrs
Spread Contiguous (touching); rarely extranodal Diffuse; often extranodal
Staging Guides therapy Limited importance
Radiation mainstray treatment
Leukemic phase Doesn't occur Occurs
Prognosis Much better prognosis than non-Hodgkins

WBC disorders Page 8.1


Association EBV HIV and autoimmune diseases

Hodgkin lymphoma (HL)


1. What are Reed-sternberg cells?
- Large B cells (CD 15 and CD 30 +ve) with multilobed nuclei and prominent nucleoli that looks like Owl eye
- They produce cytokines which attract inflammatory cells and cause fibrosis (can be seen in biopsy)

2. What are presentation of Hodgkin lymphoma?


- Due to cytokine release:
○ B symptoms - fever, chills, wt loss, night cells
○ Presence of reactive inflammatory cells that make bulk of tumor
○ Fibrosis

3. What are different subtypes of Hodgkin lymhoma and what are their characters?
Subtype Characters
Nodular sclerosis - Most common type of HL
- Classic pt is young adult female
- Presents as enlarged cervical or mediastinal lymph nodes
- Biopsy:
• Lymph node is divided by band of fibrosis
• Reed sternberg cells are present in lake like spaces (lacunar cells)

Fig: band of fibrosis (left), lacunar cells (a variant of Reed sternberg cells - right)
Mixed cellularity - Associated with abundant eosinophils (reed sternberg cells produce IL-5)
Lymphocyte rich - Best prognosis of all types
Lymphocyte depleted - Most aggressive of all types
- Usually seen in HIV pt or old pt

Non-Hodgkin lymphoma
1. What are types of non-hodgkin lymphoma?
- Non-hodgkin lymphoma is classified based on cell size, cell type (B or T), pattern of cell growth, cell surface markers,
translocations.
- Types:
○ Small B cells
 Follicular lymphoma - enlarged and too many follicles
 Mantle cell lymphoma - cancer of mantle (area around follicle)
 Marginal zone lymphoma - cancer of marginal zone (area around mantle)
 Small lymphocytic lymphoma (CLL that involve tissue)
○ Intermediate B cell lymphoma
 Burkitt lymphoma
○ Large B cells
 Diffuse large B-cell lymphoma

2. What are the causes of the different NH lymphoma?


NH lymphoma Cause Remarks

WBC disorders Page 8.2


NH lymphoma Cause Remarks
Follicular t(14;18) - BCL2 on ch 18 moves to Ig heavy chain locus on ch14 - Can progress to diffuse large B
lymphoma (BCL2 inhibits apoptosis by blocking Bax and bac from making cell lymphoma (presents as
channel in mitrochondrial membrane so cytochrome C can leak enlarging lymph node)
out)

Mantle cell t(11;14) - Cyclin D1 on ch 11 goes to Ig heavy chain locus on ch14 - Often present in late
lymphoma adulthood with painless
lymphadenopathy

Marginal zone - Chronic inflammatory state - Hashimoto thyroiditis, Sjogren - Marginal zone made of post-
lymphoma syndrome, H pylori gastritis (causes MALToma - a type of germinal B cells
marginal zone lymphoma)
Small - Chronic lymphocytic leukemia (naïve B cells; smudge cell)
lymphocytic affecting tissue
lymphoma
Burkitt - t(8;14) - c-myc on ch 8 goes to Ig heavy chain locus on ch14 (myc - Often present in child or young
lymphoma is a protein in RAS-MAPK pathway - cellular proliferation and adult as extranodal mass:
increased cyclin D production) • African form - in jaws
- Associated with BV infection • Sporadic form - in
- Biopsy finding abdomen

Fig: starry sky appearance (night sky made of dark lymphocytes;

WBC disorders Page 8.3


Fig: starry sky appearance (night sky made of dark lymphocytes;
bright stars made up of mitotic figures)- left; right - african form
appearing on jaws

Diffuse large - Arise sporadically or transformation of low grade lymphoma (ex- - Most common NHL
B-cell follicular lymphoma, chronic lymphocytic leukemia) - High grade (aggressive)
lymphoma - Present in late adulthood as
extranodal mass or enlarging
lymph nodes

3. How do you distinguish follicular lymphoma from reactive follicular hyperplasia?


Follicular lymphoma Reactive follicular hyperplasia
- Cells are monoclonal - Cells are polyclonal
- Bcl2 expression in follicles (Bcl2 - anti-apoptotic - No Bcl2 expression in follicles
protein)
- Disruption of lymph node architecture - Preservation of lymph node architecture
- Lack of tingible body marcophage in germinal - Presence of tingible body marcophage (they eat bad B cells
center produced during somatic hypermutation)

Fig: bad lymph node architecture (follicles


present not only in cortex)

Fig: left (follulicular lymphoma lacking tingible body macohpage);


right (follicular hyperplasia with tingible body macrophage)

4. What's treatment of follicular lymphoma?


- Rituximab (anti-CD20 antibody) or low dose chemotherapy

WBC disorders Page 8.4


6.8 Plasma Cell Disorders (dyscrasias)

Multiple myeloma
1. What is multiple myeloma? What is molecular mechanism?
- MM is malignant proliferation of plasma cells in bone marrow. It's most common primary
malignancy of bone.
- Serum IL-6 (aka RANK-L) may be increased that induces plasma cell growth and Ig production.

2. What are clinical features of MM?


Bone pain with - Plasma cell produce IL-6 (RANK-L) that binds to RANK on
hypercalcemia osteoclasts (bone breaks down) - creates punched out lesions
mainly on skull and vertebra

Elevated serum protein (M - Classically due too much IgG (55% of cases) or IgA (25%)
spike present in serum production
protein electrophoresis -
SPEP)

Increased risk of infection - Even though too much Ig produced, lack of Ig specificity
Rouleaux formation on - Increased serum protein decreases charge between RBC and RBC
blood smear makes stacks
(rouleau = cylindrical
packet of coins)

Anemia • Plasma cells crowd out bone marrow so little RBC synthesis

WBC disorders Page 9.1


Primary AL amyloidosis - Due to too much Ig light chain (they deposit in tissue)
Proteinurea and renal - Free light chain protein excreted in urine as Bence Jones protein
failure - Light chain deposition in kidney can cause renal failure

Fig: Bence Jones crystal (see in Waldenstrom macroglobulinemia


too)
• Mnemonic CRAB
○ HyperCalcemia
○ Renal involvement / Roulex formatin
○ Anemia / Amylodosis
○ Lytic Bone lesions

Monoclonal Gammopathy of undetermined significance (MGUS)


1. What is MGUS? What's its significance?
- Increased serum protein with M spike on serum protein electrophoresis but no other feature of
MM (- no CRAB - no lytic bone lesion, no hypercalcemia, no rouleaux, no Bence Jones proteinura
etc)
- Significance:
○ Common in elderly (5% of >70 year old people)
○ 1% of pt with MGUS progress to multiple myeloma each year

Waldenstrom Macroglobulinemia
1. What is Waldenstrom macroglobulinemia? What are clinical features? What is treatment?
- B cell lymphoma with monoclonal IgM production (M for Macroblobulinemia)
- Clinical features:
○ M spike present - due to hyper IgM
○ Presentation based on hyperviscosicity of blood due to IgM pentamer (no CRAB findings
present)
▪ Retinal hemorrhage and stroke
▪ Bleeding - high blood viscosity leads to defective platelet aggregation
▪ Raynaud phenomena

WBC disorders Page 9.2


○ Generalized lymphadenopathy
- Treatment:
○ Plasmapheresis to remove IgM from serum

WBC disorders Page 9.3


6.9 Langerhans Cell Histiocytosis

1. What is langerhans cell histiocytosis? What is histology finding?


- It is neoplastic proliferation of Langerhans cells (dendritic cells in skin and mucus membrane derived from monocytes).
Histiocyte = tissue macrophage
- Histology finding:
○ Characteristic Birbeck (tennis racket) granules on electron microscopy
○ Cells are CD1a and S100 +ve by imunohistochemistry

Fig: Birbeck granules characteristic of Langerhans cell histiocytosis

2. Describe clinical features of different types of langerhans cell histiocytosis.


Disease Clinical presentations
Letterer-Swie disease (acute disseminated Malignant - Rapidly fatal due to multiple organ involvement
langerhans cell histiocytosis) proliferation of - Classic pt is <2 year old infant
Langerhans cells - Presentation:
• Skin rash
• Cystic skeletal defect

Eosinophilic granuloma Benign - Classic presentation is pathologic fracture in


proliferation of adolescents
Langerhans cells - Skin is not involved
in bone - Biopsy shows langerhans cells with mixed inflammatory
cells and lots of eosinophils
Hand-Schuller-Christian disease Malignant - Classic pt is >3 year old
proliferation of - Presentation:
Langerhans cells • Skull rash, Lytic skull defect
• Diabetes insipidus
• Exopthalmos

WBC disorders Page 10.1


• Exopthalmos

- Classic features:
○ If you see person's name, it's malignant and you see rash
○ if disease has 2 person's name, pt is <2 year old
○ If disease has 3 person's name, pt is >3 years

WBC disorders Page 10.2


Chapter 7: Vascular Pathology
Vasculitis

1. What is vasculitis? What's it's prognosis?


• It is inflammation of blood vessel wall - can be both artery or vein (we'll talk about artery in this chapter).
• Initially, the inflammation weakens the blood vessel and can cause aneurysm. Later on, it causes fibrosis
of vessel wall and reduces lumen size and can cause ischemia.
2. What are layers of blood vessel?
• Intima - endothelial sitting on basement membrane
• Media - smooth muscle
• Adventitia - connective tissue

3. What is etiology of vasculitis? Is it infectious?


• Etiology mostly unknown. Not infectious.

4. What are clinical features of vasculitis?


• Non-specific symptoms (only tells us that inflammation is occuring)- fever, fatigue, weight loss, myalgia
• Symptoms of organ ischemia due to -
○ Thrombus formation - damage of endothelium exposes tissue factor and collagen that stimulates
clot formation
○ Reduced lumen size - Injury and healing of vessel wall causes fibrosis that reduces lumen size of the
vessel
○ Aneurysm - inflammation and necrosis of vessel wall weakens it leading to aneurysm

4.5. What are the types of arteritis?


• Large vessel arteritis - affects aorta and its major branches
• Medium vessel arteritis - affects muscular arteries that supplies organs (ex - renal artery, coronary
arteries)
• Small vessel arteritis- affects arterioles, capillaries and venules

Large vessel vasculitis Medium vessel vasculitis Small vessel vasculitis


Temporal Arteritis Polyarteritis nodosa Wegner granulomatosis
Takayasu Arteritis (aka pulseless disease) Kawasaki disease Microscopic polyangiitis
Buerger disease Churg-Strauss
Henoch-Schonlein

Large vessel vasculitis


5. What are clinical features of temporal (giant cell) arteritis - a type of large vessel arteritis?
• Vasculitis of old people (avg age 72) - usually affect female
• Classic location is branches of carotid artery-
○ Temporal artery - old people with new onset headache (classic involvement)
○ Opthalmic artery - see visual disturbance
○ Arteries of jaw - jaw claudication
• Polymyalgia (joint and muscle pain) with flu like symptoms a common association
• ESR is elevated (>100) and CRP too

6. How do you diagnose temporal arteritis? What's a caution point?


• Do a vessel biopsy - see inflamed vessel wall with giant cells and intimal fibrosis
• Granulomatous vasculitis

Vascutis, HTN Page 1.1


• Granulomatous vasculitis
• Vessel is affected segmentally. Therefore, take a long segment out for biopsy. Negative biopsy doesn't
rule out disease.

Fig - temporal arteritis biopsy. M is the media, I is the intima, F is the area of fibrosis between media and
intima that has narrowed the lumen, circle above M shows giant multinucleated cells, circle left of F
shows inflammation.

7. What is treatment of temporal arteritis (HY)?


• Corticosteroid. Treat soon because of high risk of blindness due to involvement of ophthalmic artery.

8. Contrast symptoms of temporal arteritis with Takayasu arteritis (aka pulseless disease) - a type of large
vessel arteritis?
• Takayasu arteritis is same as temporal arteritis with a couple of exceptions-
○ Affects mainly adults <50. Classic pt is young asian female
○ Affects aortic arch at branch point instead of branches of carotid artery
▪ Visual and neurologic symptoms
▪ Weak or absent pulse in upper extremity (therefore disease called pulseless disease)
▪ ESR is elevated, vasculitis is granulomatous
▪ Treat with corticosteroids

Medium vessel arteritis


9. What is polyarteritis nodosa (a type of medium vessel arteritis)?
• Necrotizing vasculitis that affects vessels in many organs but spares the lungs

10. What is clinical presentation of polyarteritis nodosa? What chemical is it associated with?
• Classic pt is young adults
• Symptoms depend on vessel involved-
○ Renal artery involvement cause hypertension
○ Mesenteric arteries - abdominal pain with melena
○ CNS arteries - neurologic disturbances
○ ESkin lesions
• Associated with serum HBsAG (hepatitis B surface antigen)

11. Correlate pathophysiology of polyarteritis nodosa with imaging.

Vascutis, HTN Page 1.2


11. Correlate pathophysiology of polyarteritis nodosa with imaging.
• The disease has lesion of varying stage in same artery. Early character of disease is that it creates
fibrinoid necrosis that makes the vessel lumen wide (as dead vessel is weak, aneurysms occur). Late
character of disease is that it creates thick fibrosis that narrows the lumen. Therefore imaging gives a
"string-of-pearl" appearance with alternate thick and thin sections.

Fig - the red marking showing fibrinoid necrosis seen in polyarteritis nodosa.

Fig - 'pearl on string' appearance of polyarteritis nodosa

12. What is treatment of polyarteritis nodosa?


• Corticosteroids and cyclophosphamide
• Fatal if not treated

13. What are clinical presentation of Kawasaki disease- a type of medium vessel vasculitis?
• Classic pt is asian child under 4
• Presentation (very nonspecific - pt has like viral infection)
○ Fever>5 days
○ Shrieking irritability - extremely irritable kid
○ Conjuctivitis sparring area right next to iris
○ Erythematous rash of palms and soles and any mucocutaenous membrane (ex - strawberry tongue)
○ Enlarged cervical lymph nodes

14. What artery is preferentially involved in Kawasaki disease? (HY)


• Coronary artery

Vascutis, HTN Page 1.3


• Coronary artery
• Complication -
○ thrombosis with MI (3 year old kid with MI) - due to endothelial damage and exposure of collagen
and tissue factor
○ aneurysm with rupture
((imagine a kid driving kawasaki bike with his hands and legs, his HR goes up - so the disease affects heart
mainly)

13. What is treatment of Kawasaki disease?


• Asprin to prevent thrombosis (antiplatelet drug - prevents formation of thromboxane A2)
• IVIG (intravenous immunoglobulins)
• Disease is self-limited

14. Why don't you give asprin to kids with viral disease?
• Because they can get Rye syndrome - encephalopathy and massive liver necrosis.

15. What is Buerger disease (medium vessel vasculitis)? How do you treat it?
• It's necrotizing vasculitis involving digits. Presents with ulceration, gangrene, and autoamputation of
fingers and toes.
• Disease associated with Raynaud phenomenon - pale to blue to red color change in digits due to
exposure to cold. Pale and blue color are due to vasospasm; red is due to return of circulation.
• Disease is associated with smoking - treatment is smoking cessation (HY)

Small vessel vasculitis (wegner granulomatosis, microscopic polyangitis, chug-strauss, and Henoch Schonlein)
• The first 3 of these disease can lead to RPGN.
• Henoch Schonlein has IgA deposition and can lead to IgA nephropathy

Wegner Granulomatosis Microscopic polyangiitis


Necrotizing vasculitis Necrotizing vasculitis
Necrotizing granuloma present Granulomas absent
C-ANCA positive P-ANCA positive
Affects nasopharynx, lungs, and kidney Affects lungs and kidney and other organs
Treatment - cyclophosphamide, corticosteroids. Treatment - cyclophosphamide, corticosteroid.
Relapse common Relapse common

Churg-Strauss Henoch-Schonlein purpura


Necrotizing vasculitis Most common vasculitis in children
Necrotizing Granulomas present Palpable purpura in buttocks pathognomic
P-ANCA positive Vasculitis due to IgA immune deposition. Can lead to
IgA nephropathy
See asthma and eosinophila - pt Disease usually follows upper respiratory tract
misdiagnosed with allergies often infection
Affects multiple organs including lung and GI involvement can cause GI pain bleeding
heart
Treat by steroids if necessary. Disease self limited.

16. What is Wegner Granulomatosis (a type of small vessel vasculitis)?


• Necrotizing granulomatous vasculitis involving nasopharynx, lungs, and kidneys. (think weCner's disease-

Vascutis, HTN Page 1.4


• Necrotizing granulomatous vasculitis involving nasopharynx, lungs, and kidneys. (think weCner's disease-
and imagine a 'C' in a human body passing through nasopharynx, lungs and kidneys).

17. What is presentation of Wegner Granulomatosis?


• Usually affect middle-aged males
• Sinusitis or nasopharyngeal ulceration
• Hemoptysis with bilateral nodular lung infiltrates
• Hematuria due to RPGN (HY)
• Serum C-ANCA level correlate with disease activity

18. How do you diagnose Wegner Granulomatosis? How do you treat it?
• Pt are C-ANCA (cytoplasmic - anti neutrophil cytoplasmic antibodies) positive.
• Biopsy shows large necrotizing granuloma with adjacent necrotizing vasculitis.
• Treatment is Cyclophosphamide and corticosteroids. Relapse is common.

Fig - large necrotizing granuloma seen in Wegner granulomatosis. Granulomas are absent in microscopic
polyangiitis.

19. What is microscopic polyangiitis (type of small vessel vasculitis)? Contrast this disease with Wegner
granulomatosis (WG).
• Necrotizing vasculitis involving multiple organs (hence the name polyangiitis), especially lung and kidney
(similar to WG).
• It's very similar to WG but nasopharyngeal involvement and granulomas are absent.
• Serum P-ANCA (perinuclear - anti neutrophil cytoplasmic antibodis) correlate with disease activity instead
of C-ANCA.
• Treatment is same as WG - corticosteroids and cyclophosphamide. Relapses are common.

20. What are C-ANCA and P-ANCA?


• In C-ANCA, anticytoplasmic antibodies mainly target cytoplasm away from nucleus. In P-ANCA,
anticytoplasmic antibodies mainly target cytoplasm in perinuclear area (hence the name - perinuclear).

21. What is Churg-Strauss syndrome ( a type of small vessel vasculitis)?


• Necrotizing granulomatous vasculitis with eosinophils.
• Involves multiple organs, especially lungs and heart.
• Asthma and peripheral eosinophila.
• P-ANCA correlate with disease activity.

Vascutis, HTN Page 1.5


22. Differentiate Churg-Strauss and microscopic polyangiitis.
Churg-Strauss Microscopic Polyangiitis
• Necrotizing granulomas with eosinophils present • No granulomas
• Peripheral eosinophila and asthma present • No eosinophila or asthma
• Involves heart and lung • Involve lung and kidney
• P-ANCA correlate with disease activity • P-ANCA correlate with disease activity

23. What is Henoch-Schonlein purpura (type of small vessel vasculitis)?


• Vasculitis due to IgA immune complex deposition
• Most common vasculitis in children

24 What is presentation of Henoch-Schonlein purpura?


• Classic presentation is palpable purpura on buttocks and legs. Usually, purpura aren't palpable. It's
palpable in this case due to vasculitis
• GI involvement - GI pain and bleeding
• IgA nephropathy leads to hematuria (HY)
• Usually occurs following an upper respiratory tract infection - the infection generates IgA as IgA protects
mucosal site. Too much IgA cause deposition and disease.

25. How do you treat Henoch-Schonlein purpura?


• Disease is self-limited. Treat with steroids if severe. Disease may recur.

Vascutis, HTN Page 1.6


Hypertension

1. What are two types of hypertension?


• Pulmonary htn
• systemic HTN (systolic or diastolic BP >140/90) - 25% of US popn are hypertensive

2. What are primary and secondary hypertension?


• Primary HTN is when we don't know the etiology. It represents 95% cases of HTN.

3. What are risk factors for primary HTN?


• Age
• Race - high risk for African Americans, low risk for asians
• Obesity
• Stress
• High salt diet
• Lack of physical activity

4. How does high sodium cause HTN?


• Systolic BP is a function of stroke volume
• Diastolic BP is a function of total peripheral resistance
• Sodium increases blood volume and peripheral resistance so increases both systolic and diastolic
BP.

5. Describe renal artery stenosis as cause of secondary HTN. (HY)


• Low blood flow to justaglomerular apparatus causes release of renin. Renin converts
angiotensinogen to angiotensin I. Angiotensin I is converted to angiotensin II by ACE in lung
endothelium. Angiotension II increases BP by two ways -
○ Powerful vasoconstrictor - increases diastolic BP
○ Stimulate adrenal medulla to secrete aldosterone - it increases sodium reabsorption and
potassium excretion from DCT. Water follows sodium and body will have more water -
increases systolic BP.

6. What is clinical presentation of renal artery stenosis (RAS)?


• Atrophy of the stenosed kidney
• Increased blood renin

7. What are important causes of RAS?


• Artherosclerois (classically seen in old males)
• Fibromuscular dysplasia (classically seen in young females) - developmental defect of blood vessel
wall that results in irregular thickening of large and medium sized arteries, specially the renal
arteries.

8. Distinguish benign and malignant HTN.


Benign Malignant
Mild or moderate increase in BP Severe increase in BP (200/120 mm Hg)
Clinically silent May arise from preexisting benign HTN or de novo
Vessels and organs are damaged Pt presents with acute end organ damage - acute renal failure,
over time headache, papilledema
MEDICAL EMERGENCY - pt can have fibrinoid necrosis of blood

Vascutis, HTN Page 2.1


MEDICAL EMERGENCY - pt can have fibrinoid necrosis of blood
vessel wall

Vascutis, HTN Page 2.2


Arteriosclerosis

1. What is arteriosclerosis? What are it's 3 patterns?


- Arteriosclerosis is hardening of arterial wall.
- Patterns-
○ Atherosclerosis - thickening of intima due to plaque(occurs in medium and large sized
vessels)
○ Arteriolosclerosis - thickening of small vessels. Wall thickens due to protein deposition
(hyaline arteriolosclerosis) or hyperplasia of smooth muscle (hyperplastic arteriolosclerosis)
○ Monckeberg medial sclerosis - calcification of media. Not very clinically significant

Artherosclerosis
2. Describe histology of artherosclerosis.
- Artherosclerosis is an intimal plaque that obstructs blood flow. It contains necrotic lipid core with
a fibromuscular wall. The lipid core can undergo dystropic calcification.

Fig - artherosclerosis. M is the media, I is the intima, the circle represents the necrotic lipid core
with cholesterol crystals. The necrotic core is surrounded by a fibromuscular wall.

3. What are 4 most common sites for artherosclerosis?


- Note artherosclerosis occurs in large and medium sized vessels -
Abdominal aorta

Vascutis, HTN Page 3.1


○ Abdominal aorta
○ Coronary arteries
○ Popliteal artery
○ Internal carotid artery

4. What are key risk factors for artherosclerosis? (HY)


Modifiable Non-modifiable
HTN Age
Hypercholesteremia (LDL Gender (male and postmenopausal females at high risk.
increases risk and HDL reduces) Estrogen has protective effect on pre-menopausal females)
Smoking Genetics (positive family history)
Diabetes

5. What is pathogenesis of artherosclerosis?


- Intima is damaged --> lipids gets between intima and media -->lipid is oxidized and then
phagocytozed by macrophage --> this makes fatty streak. Fatty streak is seen as early as
teenagers.
- Inflammation, healing and increased deposition of lipid occurs. A fibromuscular cap occurs due to
inflammation and healing.

6. What are some complication of artherosclerosis?


- It's complications account for >50% of disease in western world
- Artherosclerosis is asymptomatic until occlusion is >70% (HY)
a) Stenosis of medium sized vessels (vessels that feed organs)
○ Peripheral vascular disease (popliteal A)
○ Angina (coronary A)
○ Ischemia bowel disease (mesenteric A)
b) Plaque rupture with thrombosis
○ MI (coronary A)
○ Stroke (MCA)
c) Plaque rupture with embolization
○ Results in artherosclerotic emboli (hallmark is cholesterol cleft in emboli)

Fig - presence of cholesterol cleft in embolus is hallmark of artherosclerotic embolus.


d) Aneurysm due to weak wall - MOA - usually media and adventitia receives nutrient by diffusion
from lumen. As artherosclerotic plaque increases, it creates diffusion barrier resulting in atrophy

Vascutis, HTN Page 3.2


from lumen. As artherosclerotic plaque increases, it creates diffusion barrier resulting in atrophy
of media and adventitia.

Arteriolosclerosis
7. What is arteriolosclerosis?
- Narrowing of small arterioles.
- Causes - deposition of hyaline or hyperplastic media.

8. What is hyaline arteriolosclerosis?


- Proteins leaking into vessel wall causes vascular thickening
- Seen as pink hyaline on microscopy

Fig - pink hyaline in vessel wall characterstic of hyaline arteriolosclerosis

9. What are 2 causes of hyaline arteriolosclerosis? (VHY)


- Benign HTN - high pressure forces protein into the vessel wall
- Diabetes - non-enzymatic glycosilation of basement membrane makes vessel wall leaky and
protein leaks in. This is how diabetes can cause nephrotic syndrome (diabetic nephropathy).

10. What is key consequence of hyaline arteriolosclerosis? (HY)


- Reduced vessel diameter leads to end organ ischemia
- Classically produces glomerular scarring (arteriolonephrosclerosis) which progress to chronic renal
failure (that's how diabetic pt with long standing HTN die

Vascutis, HTN Page 3.3


Fig - glomerular scarring seen in arteriolonephrosclerosis

11. What is hyperplastic arteriolosclerosis?


- Vessel wall thickens due to hyperplasia of smooth muscle
- 'Onion-skin' like appearance

Fig - onion skin like appearance of blood vessel in hyperplastic arteriolosclerosis

12. What causes hyperplastic arteriosloclerosis?


- Malignant HTN - this leads to acute renal failure (ARF) and fibrinoid necrosis of vessel

13. What are consequences of hyperplastic arteriosclerosis?


- Reduced vessel diameter leads to end organ ischemia
- Malignant HTN may lead to fibrinoid necrosis of vessel wall (another cause of fibrinoid necrosis is
vasculitis)
- Classically causes ARF with 'flea-bitten' appearance - flea-bitten necrosis is due to pin point
hemorrhages (HY)

Vascutis, HTN Page 3.4


Fig - flea bitten kidney classic of hyperplastic arteriolosclerosis due to pin point hemorrhage -
leads to ARF. Contrast glomerular scarring in chronic renal failure due to arterionephrosclerosis

Monckeberg medial calcific sclerosis


14. Describe Monckeberg medial sclerosis.
- Calcification of media
- Does not obstruct lumen, therefore not clinically significant
- Seen as incidental finding in X-ray or mammography

Fig- Enclosure shows monckeberg medial sclerosis. Note how it's limited to media and doesn't
have much effect on lumen diameter.

Vascutis, HTN Page 3.5


Fig - mammography showing Monckelberg medial calcification of blood vessel

Vascutis, HTN Page 3.6


Aortic Dissection and Aneurysm

1. What is aortic dissection?


- Blood leaks into the media through a break in intima and it rips the media in half

Fig - aortic dissection

2. What are two things required for aortic dissection?


a. A ton of stress - therefore it mainly happens in proximal 10 cm of aorta
b. Pre-existing weakness of media - caused by HTN or inherited defect of connective tissue

3. How does HTN weaken media and lead to aortic dissection?


- HTN leads to hyaline arteriolosclerosis of vaso vasorum (blood vessel that supplies media and
adventitia of aorta). Media weakens due to low supply of blood.

4. What is the most common cause of aortic dissection?


- HTN

5. What are some connective tissue disorders that can lead to dissection?
- Marfan syndrome - mutation in fibrillin-1 protein. Fibrillin is the base on which elastic fibers are
laid down. Media has lots of elastic fibers
- Ehlers-Danlos syndrome - mutation in collagen or proteins that interact with collagen.

6. What is clinical picture of aortic dissection?


- Presents with tearing chest pain that radiates to the back
- Most common cause of death is cardiac tamponade. Other causes include aorta bursting open and
obstruction of arteries that branch off aorta.

Aneurysms
7. What are two classic location of aneurysm?
- Thoracic aneurysm
- Abdominal aneurysm

8. What is a classic cause of thoracic aneurysm?


- Tertiary syphilis- syphilis is end arteritis disease. It affects vaso vasorum and poor blood supply
weakens aortic wall. Weak vessel wall leads to aneurysm
- Tree bark appearance of aorta is classic finding - as vaso vasorum is affected, poor blood supply
leads to fibrosis and scarring

Vascutis, HTN Page 4.1


Fig - normal aorta (left); tree bark like aorta in tertiary syphilis (right)

9. What are complications of thoracic aneurysm?


- Aortic insufficiency (HY) - as aorta dilates, the valves are pulled apart and can't close well
- Compression of mediastinal structures
- Thrombosis in wall of aneurysm due to disruption in laminar blood flow. It can embolize.

10. What is the most common site of abdominal aortic aneurysm (AAA)? Why is this the most
common site?
- Below the renal arteries and above aortic bifurcation
- Vasa vasorum stops at the level of renal artery bifurcation. Therefore, artherosclerosis of aorta
below this level leads to high level of media atrophy due to poor diffusion of nutrient and gases.
The weak aortic wall leads to aneurysm. (note - most common location of artherosclerosis in
human is abdominal aorta).

Fig: normal abdominal aorta (left); AAA (right) - note calcification of vessel wall and thrombosis
11. What is presentation of AAA?
- >60 year old smoker male with HTN
- Pulsatile abdominal mass that grows with time

12. What is complication of AAA?


- Rupture, especially when >5cm in diameter
- Presents with triad of hypotension, pulsatile abdominal mass and flank pain.

Vascutis, HTN Page 4.2


Chapter 8: Cardiac Pathology
8.1 Ischemic Heart Disease

- Infarction - tissue necrosis due to lack of oxygen


- Ischemia - inadequate blood supply to an organ
- Hypoxemia - (PaO2 < 60mm Hg)
- Early MI - <55 year for men and <45 for women
- 3 most common cause of chest pain in outpatient setting: GERD, anxiety, costochondral tenderness
- Pain with MI is usually described as pressure - elephant sitting on lung. Pleuritic chest pain is more sharp
- ST depression in AVR lead is more specific for pericarditis

1. What is most common cause of ischemic heart disease?


- Artherosclerosis of coronary arteries
- Risk factors same as risk factor for artherosclerosis - age, sex, race, smoking, HTN, diabetes

2. How long does a ischemia last before irreversible injury to cardiac myocytes occur?
- 20 minutes
Angina (reversible injury to cardiac myocytes)
3. Describe Stable angina. What type of damage is suffered by the cells?
- Chest pain develops with physical or emotional stress
- Caused due to >70% stenosis of coronary arteries
- Myocytes undergo reversible injury during stable angina (HY)

4. What is presentation of stable angina? What is seen on EKG?


- Chest pain <20 mins that radiates to left arm or jaw. If > 20 minutes, it causes myocardial infarction
(irreversible damage to myocytes)
- Diaphoresis, SOB
- Pain relieved by rest or nitroglycerin
- EKG shows ST segment depression - because subendocardial ischemia is seen as ST depression. Stable
angina causes subendocardial ischemia because blood vessels travel in epicardium and endocardium is last
part to receive blood.

5. Describe unstable angina.


- Chest pain occurs at rest (that's why called unstable)
- It's due to rupture of artherosclerotic plaque with thrombosis and incomplete occlusion of coronary artery
(rupture usually occurs at neck of plaque)
- Myocytes undergo reversible injury

6. What is presentation of unstable angina? What is seen on EKG?


- Relieved by nitroglycerin (venodialation reduces the work heart has to do)
- High risk of progression to MI because the thrombus can grow.
- ST depression on EKG - same reason as stable angina

Cardiac Page .1


Fig - unstable angina in coronary artery. Note the dark thrombus. The thrombus has high chance of
growing and leading to MI.

7. What is prinzmetal angina?


- Vasospasm that completely clamps coronary artery - leads to transmural ischemia
- Chest pain irrespective of physical/emotional exertion
- Myocytes undergo reversible injury

8. What is presentation of prinzmetal angina?


- ST segment elevation - as coronary artery completely clamps down, we get transmural ischemia.
Transmural ischemia presents at ST elevation.
- Relieved by NG or calcium channel blockers

Myocardial infarction (irreversible injury to cardiac myocytes)


9. What is MI? What are it's causes?
- Necrosis of cardiac myocytes (irreversible injury)
- Main cause - rupture of artherosclerotic plaque with thrombosis and complete occlusion of coronary
artery
- Other causes - coronary artery vasospasm, emboli, vasculitis (ex - kawasaki disease)

10. What are clinical features of MI?


- Severe crushing chest pain (>20 minutes - cells die after this time)
- Diaphoresis, SOB
- Symptoms not relieved by nitroglycerin
- Mostly involve left ventricle. Right atria and ventricles are usually spared

11. What are key arteries involved in MI?


- LAD (most common) - leads to infraction of anterior wall of LV and anterior interventriclular septum
- Right coronary artery (2nd common) - infraction of posterior wall of LV and posterior interventricular
septum
- Left circumflex artery - infraction of lateral wall of LV

12. Describe initial phase of MI.


- Subendocardial necrosis involving <50% of myocardial thickness
- ST depression (recall subendocardial infraction leads to ST depression and transmrural infraction leads to
ST elevation)

13. What are lab enzyme tests of MI?


- Hallmark of irreversible damage to cell is membrane leak. So, cardiac enzymes will leak.

Cardiac Page .2


- Hallmark of irreversible damage to cell is membrane leak. So, cardiac enzymes will leak.
- Troponin I - most sensitive and specific marker
○ Rises 2-4hrs post infraction
○ Peaks at 24 hrs
○ Returns to normal 7-10 days
- CK-MB - useful for detecting reinfarction
○ Rises 4-6 hrs after infraction
○ Peaks at 24 hrs
○ Returns to normal by 72 hours

14. What is treatment of MI?


- MONA (morphine, oxygen, nitrates, asprin)
- ACEi (decreases blood volume due to low aldo, and reduces peripheral vasoconstriction (afterload))
- Beta blocker (slow heart rate and reduce risk of arrhythmia)
- Definitive treatment:
○ Fibrinolysis or angioplasty
 Complications: contraction band necrosis and repurfusion injury (reperfusion injury occurs due
to free radical damage by neutorphils and oxygen.

Fig - the box shows necrotic myocytes (no nuclei) and the circles show contraction band necrosis

15. Describe time frame of MI (HY)


Time Microscopic Gross change Complication
change
<4 hrs none none - Cardiogenic shock
- CHF
- Arrhythmia
4-24 hrs Coagulative Dark discoloration - Arrhythmia (it doesn't happen post 24 hrs
necrosis because the conduction system is already
damaged and necrosis occurs < 1 day. So if
arrhythmia don't happen by then, it won't
happen.

1-3 days Inflammatio Yellow pallor - Fibrinous pericarditis (chest pain with friction

Cardiac Page .3


1-3 days Inflammatio Yellow pallor - Fibrinous pericarditis (chest pain with friction
n rub)- only occurs with transmural infraction
(Neutrophil)

3 day - 1 Macrophage Yellow pallor - Rupture of ventricular free wall (cardiac


week s tamponade), papillary muscle (mitral regurg) or
interventricular septum (shock)
1-3 Granulation Red border (blood vessels)
weeks tissue with emerging from edge of infract
fibroblasts,
collagen and
blood
vessels

Months Fibrosis White scar - Aneurysm


- Mural thrombosis
- Dressler syndrome (autoimmune pericarditis)
(HY)

• Arrhythmia doesn't happen post 24 hrs because the conduction system is already damaged and necrosis
occurs < 1 day.
• Fibrinous pericarditis only occur with transmural infaract
• Papillary muscles are fed by right coronary artery
• Dressler syndrome - inflammation of pericardium and exposure of pericardial antigen can cause
autoimmune attack to pericardium

<1 day 1 day - 1 week 1 week- 1 month >1 month


Coagulative necrosis First neutrophils and then Granulation tissue Scar tissue
macrophage
• Dark discoloration of • Yellow pallor • Red border surrounding yellow • White
heart pallor scar

Cardiac Page .4


Fig - coagulative necrosis of heart showing dark discoloration (<1 day)

Fig- yellow pallor of heart post MI (1 day - 1 week)

Fig - fibrinous pericarditis (fibrin exudate during neutrophil rich stage (day 1-3 post MI) rubs when hear
contracts producing and characteristic friction rub). Only see during transmural infraction

Cardiac Page .5


Fig - 1-3 weeks post MI. The yellow pallor is central area of necrosis (granulation tissue?) surrounded by
emerging blood vessels from edge of infaract

Fig - Months after MI (white scar tissue)

Cardiac Page .6


Fig - layers of heart muscle

16. What is sudden cardiac death? What is it's etiology


- Unexpected death due to cardiac disease with no symptom or death <1 hr after symptom arise
- Cause : ventricular arrhythmia
- Etiology:
○ Severe artherosclerosis (90% of cases)
○ Less common: cardiomyopathy, mitral valve prolapse, cocaine abuse

17. What is chronic ischemic heart disease?


- Poor myocardial function due to chronic ischemic damage (with or without infraction)
- Can progress to congestive heart failure

Cardiac Page .7


8.2 Congestive Heart Failure (CHF)

1. What is division of CHF?


- Left sided failure
- Right sided failure

2. Differentiate right and left sided CHF.


Left sided failure Right sided failure
Causes - HTN - Most common cause is left
- Dilated cardiomyopathy sided failure
- MI - Left-to-right shunt
- Restrictive cardiomyopathy - Chronic lung disease (cor
pulmonale)
Presentat - Presentation based on pulmonary congestion and - Jugular venous distension
ion pulmonary edema - Painful hepatosplenomegaly
- Dyspnea, paroxysmal nocturnal dyspnea (increased with characterstic 'nutmeg
venous return while lying flat), orthopnea (SOB while liver'
flat), crackles in lung - Cardiac cirrhosis
- Hemosiderin laden macrophage in lung (aka heart - Pitting edema
failure cells) - pulmonary capillaries burst due to
backing up
- Decreased flow to kidney activates RAS which leads
to fluid retention and worsens CHF
Treatmen ACE inhibitors
t

Cardiac Page .1


8.3 Congenital Defects

1. What is epidemiology of congenital heart defect?


- Occur during week 3-8 (organogenesis)
- Seen in 1% of live births
- Most diseases are sporadic
- Most common congenital heart defect is VSD

2. If there's a left-right shunt in heart, which direction does it initially present? Why does it reverse?
- Early shunt is left to right due to reduced resistance in pulmonary circulation.
- Shunt later becomes right to left - pulmonary circulation increases resulting in pulmonary HTN and
hypertrophy of pulmonary vessels

3. What is Eisenmenger syndrome (aka tardive cyanosis)?


- Serious hypoxemia caused by reversal of left-to-right shunt is called Eisenmenger syndrome.
- It can be present during atrial septal defect, VSD or patent ductous arteriosus

4. What are presentation of Eisenmenger syndrome?


- Right ventricular hypertrophy
- Polycythemia vera (increased RBC to combat hypoxemia)
- Clubbing, cyanosis

VSD
1. What's most common congenital heart defect? What is it associated with (HY)?
- Ventricular septal defect
- It's associated with fetal alcohol syndrome

2. How does it present?


- Early on, the shunt is left to right, later on it becomes right to left.

3. How is it treated?
- Small defects close spontaneously. Surgery for large defects.

Cardiac Page .1


- Small defects close spontaneously. Surgery for large defects.

ASD
1. What are types of atrial septal defects?
- Ostium secundum (most common) - hole between atria. Ostium is primitive membrane that
divides heart to left and right sides.
- Ostium primum (aka endocardial cushion defect) -
○ ASD + valve defect + defect in intraventricular wall.
○ Endocardial cushion - junction of atrial septum, ventricular septum, tricuspid and mitral
valves.
○ Most associated heart defect with Down's syndrome
- Sinus venosus

Fig: osteum primum (left), osteum secundum (middle and right)

2. What's ASD presentation? What's an important complication?


- Split S2 sound: due to increased blood in right heart, pulmonic valve closes late.
- Paradoxical emboli are important complication - ex - DVT emboli will go to brain instead of lungs

PDA
1. What is patent ductous arteriosus? What's it associated with?
- Failure of ductous arteriorus to close after birth.
- Associated with congenital rubella
- Presents with left to right shunt between aorta and pulmonary artery. Later on, the shunt
becomes right to left due to pulmonary HTN and hypertrophy of pulmonary vessels

2. What's it's presentation?


- Asymptomatic at birth but has 'machine-like murmur'
- May lead to Eisenmenger syndrome results in lower extremity cyanosis (because ductus arteriosus
happen after upper extremity branching)

Cardiac Page .2


happen after upper extremity branching)

3. What's it's treatment?


- Indomethacin - decreased PGE. PGE kEEEps PDA open.

Tetralogy of fallot
1. What is tetralogy of fallot?
- Stenosis of right ventricular outflow tract
- Right ventricular hypertrophy
- VSD
- Aorta that overrides the VSD
- Right to left shunt. Almost all others are left to right in the beginning.

2. What's it's presentation?


- Early cyanosis due to right-to-left shunt (more stenting = more cyanosis)
- Usually after exercise, pt squat to increase pulmonary blood flow (squatting increases systemic
resistance)
- Boot shaped heart on X-ray

Fig: boot shaped heart indicating tetralogy of falot

Transposition of great vessels


1. What is transposition of great vessels?
- Aorta arises from right ventricle and pulmonary artery arises from left ventricle. Right side does

Cardiac Page .3


- Aorta arises from right ventricle and pulmonary artery arises from left ventricle. Right side does
systemic circulation and left side does pulmonary curculation
- Left and right sided blood never mix (early cyanosis)

2. What's it's presentation? What's it associated with? How do you treat?


- Presentation:
○ Early cyanosis (right and left sided blood don't mix)
○ Right ventricle hypertrophy and atrophy of left ventricle
- Treatment:
○ Create a shunt after birth is required for survival
○ Administer PGE (PGE kEEEps PDA open)
- Association:
○ Maternal diabetes

Truncus arteriosus
1. What is truncus arteriosus?
- It's when a single large vessel arises from both ventricles. (truncus fails to divide to aorta and
pulmonary artery)
- Presentation:
○ Early cyanosis

Tricuspid atresia (atresia means fail to form a tube)


1. What's tricuspid atresia? How does it present?
- It's failure of development of orifice of tricuspid valve.
- Presentation:
○ Hypoplastic right ventricle (ASD and VSD often present)
○ Early cyanosis
- Association:

Cardiac Page .4


- Association:
○ ASD alone or ASD + VSD

Coarctation of aorta
1. What's coarctation of aorta? What are two types?
- Coarctation of aorta is narrowing of aorta
Infantile type Adult type
Anatomy Narrowing is after aortic arc but before PDA Narrowing is after aortic arch (not
associated with PDA; if PDA
present, it's infantile type)
Associati Associated with PDA and Turner syndrome (one Associated with bicuspid aortic
on X, no Y) (HY) valve (HY)
Presenta Presents as lower extremity cyanosis in infants, - Presents as HTN in upper
tion often at birth - due to coarctation, lower extremitties and hypotension with
extremity isn't supplied by LV but by RV. Upper weak pulse in lower extremities;
extremities are fine because LV supplies there often discovered in adulthood
- Collateral circulation across
intercostal arteries causes
engorged artieries and notching on
ribs on X-ray (HY)

Cardiac Page .5


Cardiac Page .6
8.4 Valvular Disorders

Acute rheumatic fever


1. What is pathogenesis of actue rheumatic fever?
- It's a systemic complication of group A strep which presents 2-3 weeks after streptococcal
pharyngitis
- Bacterial M protein mimics human protein and autoantibodies are generated.

2. How is acute rheumatic fever diagnosed?


- Diagnosis is based on evidence of group A strep infection (elevated ASO or anti-DNAse B titer) +
major (JONES) or minor criterea
- Minor criteria
○ Fever and elevated ESR (non-specific)
- Major criteria (JONES)
○ Joint (migratory polyarthritis) - swelling in pain in large joints (wrist, knee, ankle) that
resolve in days and move to another large joint
○ O (pancarditis)
 Endocarditis - Mitral valve is most commonly affected. See small vegetations along
line of closure that lead to regurgitation
 Myocarditis - Aschoff bodies seen (focal area of chronic inflammation). Presence of
Anitschkow cells (reactive histiocytes with slender, wavy nucleus), fibrinoid material
and giant cells. (myocarditis most common cause of death)
 Pericarditis - friction rub and chest pain
○ Subcutaneous Nodules
○ Erythema marginatum - nonpruritic rash with erythematous border commonly on trunk and
limbs
○ Sydenham chorea

3. What is most common cause of death in acute rheumatic fever?


- Myocarditis

4. What is prognosis of acute rheumatic fever?


- Acute attack usually resolvs but may progress to chronic rheumatic heart disease
- Repeat exposure with group A strep increases chance of rheumatic heat disease

Chronic rheumatic fever


1. What's presentation of chronic rheumatic fever?
- Mitral valve is most commonly affected, aortic valve is no. 2; other valves rarely affected
- Mitral valve - classic fish mouth appearancedue to stenosis (valve can't open well)
- Aortic valve - fusion of commissures

2. What's a complication of chronic rheumatic fever?

Cardiac Page 4.1


2. What's a complication of chronic rheumatic fever?
- Infective endocarditis

Aortic stenosis
1. What are some causes of aortic stenosis?
- Normal wear and tear of valve
- Bicuspid aortic valve (speeds up wear and tear)
- Chronic rheumatic fever

2. What is its presentation?


- Presents in late adulthood (>60 years)
- Crescendo-decrescendo murmur with systolic ejection click (click is when the valve opens?)

3. How do you distinguish stenosis from chronic rheumatic fever vs normal wear and tear?
- In chronic rheumatic fever, there is fusion of commissures of aortic valves. Also, we see mitral
stenosis.

4. What are complications of aortic stenosis?


- Concentric left ventricular hypertrophy - may progress to cardiac failure
- Angina with syncope with exercise (decreased perfusion of heart and brain)
- Microangiopathic hemolytic anemia (see schistocytes)

5. How do you treat aortic stenosis?


- Valve replacement

Aortic regurgitation
1. What are causes of aortic regurgitation?
- Isolated aortic root dilation (most common cause)
- Aortic dissection, Syphillis (causes aortic root dilation)
- Valve damage, ex - infective endocarditis

2. What are clinical features of aortic regurgitation?


- Increased pulse pressure (water-hammer pulse) - diastolic pressure is low due to regurgitation,
systolic pressure increases due to increased stroke volume (pulse pressure is difference between
systolic and diastolic pressures)
- LV dilation and eccentric hypertrophy due to volume overload

3. What is treatment of aortic regurgitation?


- Valve replacement once LV dysfunction develops

Cardiac Page 4.2


Mitral valve prolapse
1. What is mitral valve prolapse? What are some etiologies?
- Mitral valve prolapse is ballooning of mitral valve into left atrium during systole
- It occurs due to myxoid degenration of valve making it floppy
- Etiologies:
○ Marfan syndrome
○ Ehlers-Danlos syndrome

2. What is presentation of mitral valve prolapse?


- Mostly asymptomatic
- Mid-systolic click followed by regurgitation murmur
- Murmur is softer with squatting (increased systemic resistance decreases left ventricular
emptying)

3. What are complications?


- Rare but infective endocarditis, arrhythmia and severe mitral regurg

4. What is treatment of mitral valve prolapse?


- Valve replacement

Mitral regurgitation
1. What are some causes of mitral regurgitation?
- Complication of mitral prolapse
- LV dilation
- Infective endocarditis
- Acute rheumatic fever - vegetation on valve edge prevent smooth closing
- Papillary muscle rupture after MI

2. What is presentation of mitral regurgitation?


- Holosystolic "blowing" murmur; lower when squatting (increased systemic resistance decreases
LV emptying) and expiration (increases blood return to LV)
- Volume overload and left sided failure

Mitral stenosis
1. What are some causes of mitral stenosis?
- Chronic rheumatic valve disease most common cause

2. What are presentations of mitral stenosis?


- Opening snap followed by diastolic rumble
- Volume overload with dilation of left atrium:
○ Pulmonary congestion with edema and alveolar hemorrhage
○ Pulmonary HTN and eventual right sided heart failure
○ A-fib with mural thrombus

Cardiac Page 4.3


8.5 Endocarditis

1. What is endocarditis?
- Inflammation of endocardium (mostly valves) usually due to bacterial infection.

Pathogen
1 S. viridans Most common cause of Subacute endocarditis (small
endocarditis; dental vegetations)
procedure
2 S. aureus Most common cause of Acute endocarditis (large
endocarditis in IV drug vegetations)
users
3 S. epidermidis Key organism to cause
endocarditis of prosthetic
valves
4 Strep bovis Cause endocarditis in pt
with colorectal carcinoma
(HY)
5 HAECK organism Hemophilus, Cause endocarditis with negative
Actinobacillus, blood culture (because
Cardiobacterium, pathogens difficult to grow)
Eikenella, Kingella
6 Nonbacterial Occurs if pt is
thrombotic hypercoagulable or has
endocarditis adenocarcinoma
7 Libman-Sacks Sterile vegetations on Associated with lupus
endocarditis both side of mitral valve
8 Coxiella burnetti Most common cause of
culture -ve endocarditis
1. Describe endocarditis due to S. viridans.
- Most common cause of endocarditis.
- Low virulence pthogen; therefore mainly infects previously damaged valves
- Results in small vegetations that don't destroy valves (therefore called subacute endocarditis)

3. Describe pathogenesis of endocarditis due to S. viridans.


- Damaged endocardial surface develops thrombotic vegetations
- During transient bacteremia (ex - dental procedure), bacteria can be trapped in these vegetations

4. Describe endocarditis due to S. aureus.


- Most common cause of endocarditis in IV drug abusers
- High virulence organism; can infect normal valves (mainly tricuspid)
- Vegetations are large and destroy valve (called acute endocarditis)

Cardiac Page 5.1


Fig - staph aureus endocarditis

5. Describe clinical presentation of endocarditis. (FOR JANE)


- Fever
- Murmur
- Janeway lesion - mainly on palm and sole (painless)
- Osler nodules (hurts - ouch!) - mainly on finger and toes (painful)
- Roth nodules
- Hemolytic anemia (usually microcytic)
- Nailbed splinter hemorrhage
- Emboli

Fig - roth nodules (retinal hemorrhage); osler and roth nodules are immunologic.

Endocarditis may lead to glomerulonephritis and positive Rheumatic factor.

6. How does endocarditis lead to low blood iron.


- Acute phase reactant proteins are made (hepsidin being one major one). Hepsidin traps iron in
storage site. This leads to high ferritin. Also, bone marrow takes iron from blood because hepsidin
is trapping iron in storage site. That’s how serum iron decreases.

7. Describe nonbacterial thrombotic endocarditis.


- It is sterile vegetation seen on valves during hypercoagulable state or underlying adenocarcinoma.

Cardiac Page 5.2


- It is sterile vegetation seen on valves during hypercoagulable state or underlying adenocarcinoma.
- These vegetations occur on bicuspid valves along lines of closure and lead to mitral regurg

8. Describe Libman-Sacks endocarditis.


- Sterile vegetations on both side of mitral valve - leads to mitral regurg
- Associated with lupus (HY)

9. Describe diagnosis of endocarditis.


- Surface Echo - 60% sensitive
- Transesophageal echo - 90% sensitive

- Endocarditis leads more often regurg of valves rather than stenosis


- If valve vegetation is >1cm, consider surgery of valves

Cardiac Page 5.3


8.6 Cardiomyopathy

1. Describe dialated cardiomyopathy.


- Most common type of cardiomyopathy
- Leads to systolic dysfunction (heart can't contract very well)
- Complications:
○ mitral and tricuspid regurg
○ Arrhythmia (heart's conduction system is stretched up)

2. What are some causes of dialated cardiomyopathy?


- Idiopathic in most cases
- Mutation - they are autosomal dominant
- Myocarditis - coxcakie virus most common pathogen
- EtOH abuse (HY)
- Drugs - doxorubicin, cocaine
- Pregnancy (HY) - occurs in 3rd trimester or soon after birth

Fig - myocarditis; notice the presence of lymphocytes. Most common cause is coxcakie virus;
acutely, it can cause death; in chronic cases, it can cause dialated cardiomyopathy

3. What is treatment for dialated cardiomyopathy?


- Nothing; pt need transplant

Hypertrophic cardiomyopathy
4. Describe hypertrophic cardiomyopathy.
- Massive hypertrophy of left ventricle
- Most common cause (HY): due to autosomal dominant mutations in sarcomere proteins

5. What are its clinical presentation?


- Diastolic dysfunction (heart doesn't fill well)
- Sudden death in young athletes due to ventricular arrhythmias.
- Syncope with exercise
- Biopsy (HY):
○ Myofiber hypertrophy with disarray

Cardiac Page 6.1


Fig - myofiber hypertrophy and disarray (fibers oriented in different directions)common in
hypertrophic cardiomyopathy

Restrictive cardiomyopathy
6. What is restrictive cardiomyopathy and it's causes?
- Diastolic dysfunction
- Causes
○ Amyloidosis
○ Sarcoidosis
○ Hemochromatosis
○ Endocardial fibroelastosis (in kids) - there's fibrosis and elastosis in endocardium
○ Loeffler syndrome - eosinophilic inflammation of endocardium and myocardium

7. What is EKG finding of restrictive cardiomyopathy?


- Low voltage EKG
- Diminished QRS amplitudes

Cardiac Page 6.2


8.7 Cardiac Tumors

Myxoma Rhabdomyoma Metastasis


Most common cardiac tumor in Most common cardiac Most common cardiac tumor
adults tumor in children
Benign mesenchymal tumor with Benign hamartoma of
abundant ground substance in cardiac muscle
histology
Pedunculated mass in left atrium Usually seen in ventricle Most commonly affects pericardium
and seen as pericardial effusion

Associated with tuberous Breast, lung carcinoma, melanoma and


sclerosis lymphoma common source of
metastasis
Myxoma
1. What is most common primary cardiac tumor in adults?
- Myxoma

2. What type of tumor is myxoma?


- Benign mesenchymal tumor with abundant ground substance in histology

2. What is its presentation?


- Pedunculated mass in left atrium that can obstruct mitral valve (syncope)

Fig: biopsy (abundant ground substance) and autopsy of myxoma

Rhabomyoma
1. What is most common primary cardiac tumor in children?
- Rhabdomyoma

Cardiac Page 7.1


2. What kind of tumor is rhabdomyoma?
- Benign hamartoma of cardiac muscle
- Usually seen in ventricle

2. What is it associated with?


- Tuberous sclerosis (a genetic disorder that causes non-malignant tumors in many different organs,
primarily in the brain, eyes, heart, kidney, skin and lungs. Common presentation: seizures,
developmental delay, intellectual disability and autism)

Fig: rhabdomyoma

Metastasis
1. What is most common type of cardiac tumor?
- Metastasis (more common than primary tumors)

2. What are common sites of metastasis?


- Breast, lung carcinoma, melanoma and lymphoma

3. What is its most common presentation?


- Most commonly affects pericardium and seen as pericardial effusion

Cardiac Page 7.2


Chapter 9: Respiratory Tract
Pathology
9.1 Nasopharynx, 9.2 Larynx

Nasopharynx
1. Differentiate angiofibroma and nasopharyngeal carcinoma.
Angiofibroma Nasopharyngeal carcinoma
Defn Benign tumor of nasal mucosa Malignant tumor of nasopharyngeal epithelium
made up of large blood vessels
and fibrous tissue
Demograp Classically seen in adolescent - Classically seen in Chinese young adults and
hics males - very rare in females african kids

Presentati Presents with profuse epistaxis - Associated with EBV


on (nose bleed) - HY - Often involves cervical lymph nodes
Biopsy Biopsy - pleomorphic keratin positive epithelial
cells (poorly differentiated SCC) in background
of lymphocytes (HY)

2. Differentiate Rhinitis and Nasal polyp.


Rhinitis Nasal polyp
Cause Rhinovirus no 1 cause - Secondary to repeated rhinitis (HY)
- CF (if you see child with nasal polyp, suspect CF) -
HY
- Asprin intolerant asthma (HY)

Presentatio Runny nose, sneezing, Protrusion of edematous, inflamed nasal mucosa


n congestion

3. What is allergic rhinitis? What's it's associated with?


- A type of rhinitis caused due to type 1 hypersensitivity reaction (ex - pollen)
- Association:
○ Asthma
○ Eczema
- Presentation:
○ Eosinophilic infiltrate

4. What is aspirin intolerant asthma?


- Presents as triad of asthma, aspirin induced bronchospasm and nasal polyps.

Pulmonary Page 1.1


- Presents as triad of asthma, aspirin induced bronchospasm and nasal polyps.
- Seen in 10% of asthma patients

Larynx
1. Differentiate laryngeal papilloma and laryngeal carcinoma
Laryngeal papilloma Laryngeal carcinoma
Defn Benign papillary tumor of vocal cord SCC of epithelial lining of vocal cord
Cause HPV 6 and 11; EtOH and smoking (papilloma rarely
progress to carcinoma)
Presentatio - Hoarseness of voice - Hoarseness of voice
n - Usually single in adults and multiple - Cough and stridor
in children (HY)

2. Differentiate acute epiglottitis and laryngotracheobronchitis (croup).


Acute epiglottitis laryngotracheobronchitis (croup)
Cause H. Influenzae type b most common cause (in Parainfluenza virus most common
immunized or non-immunized kids. cause
Present Too much drooling, dysphagia, sore throat, Barking cough and inspiratory
ation fever, muffled voice, inspiratory stridor stridor
xray Thumb sign on xray Steeple sign on Xray

Risk of actue airway obstruction (medical


emergency)

3. What is vocal cord nodule (singer's nodule)? What's its cause?


- Nodule on true vocal cord

Pulmonary Page 1.2


- Nodule on true vocal cord
- Caused due to excessive use of vocal cord; usually bilateral (wear and tear issue
- Composed on degenerative myxoid connective tissue
- Treat with rest

Fig: vocal cord nodule (usually bilateral and seen on true vocal cord)

Pulmonary Page 1.3


9.3 Pulmonary Infections

1. What is pneumonia? What causes it?


- It's infection of lung parenchyma.
- Causes:
○ Lack of cough reflex
○ Damage to mucociliary escalator
○ Mucus plugging

2. What are presentation of pneumonia?


- Fevers and chills (organism usually leak out to blood)
- Cough with yellow-green (pus) or rusty (blood) sputum
- Tachypnea with pleuritic chest pain (inflammation produces bradykinin and PGE2 which causes pain)
- Decreased breath sounds and dullness to percussion (loss of air volume due to exudates will result in dullness
to percussion)
- Elevated WBC count (due to infection)

3. How do you diagnose pneumonia?


- X-ray
- Blood culture/ sputum stain and culture

4. What are patterns of pneumonia seen on Xray?


Lobar pneumonia Bronchopneumonia Interstitial pneumonia (aka atypical pneumonia)
- Affects whole lobe - Affects area around - Inflammation of interstitium without consolidation of air
bronchioles (mostly sacks (see increased lung markings in X-ray)
multifocal)
- Cause is mostly - Cause is mostly - Called atypical because need special media to grow the
bacterial bacterial bacteria. Viruses also cause it
- Most common Bacterial causes:
cause: - Mycoplasm pneumoniae
• Strep. - Chlamydophilla
Pneumoniae - Legionella (pathoma puts legionella in broncho)
(95%)
• Klebsiella
pneumoniae
• H influenziae
Treat: ceftriaxone Treatment:

Fig - lobar pneumonia (right), bronchopneumonia (middle), interstitial pneumonia (left). Note lack of alveolar
exudate in interstitial pneumonia

Pulmonary Page 2.1


exudate in interstitial pneumonia

Lobar pneumonia Bacteria Association


Strep pneumo • Most common cause of community acquired pneumonia
Klebsiella pneumoniae • enteric flora that's aspirated.
• Common seen in alcoholics, nursing home pt.
• Bacteria has thick mucoid capsule, so see currant jelly sputum.
• Often complicated by lung abscess

Broncho pneumonia Association


bacteria
Staph aureus • Most common cause of secondary pneumonia (pneumonia superimposed on viral
upper respiratory tract infection)
• Often complicated by abscess or emphyema (free pus in pleural space)
Haemophilus • Common cause of secondary pneumonia
influenzae • Associated with COPD
Pseduomonas • Associated with cystic fibrosis patients
aeruginosa
Moxarella catarrhalis • Associated with COPD
• Community acquired
Legionella • Transmitted from water source
pneumophila • Associated with COPD and immunocompromised states
• Intracellular organism visualized by silver stain
• Pt. Presents with pneumonia, diarrhea, and hyponatremia

Atypical pne Association


organism
Mycoplasma • Most common cause of atypical pneumonia
pneumoniae • Affects young adult in close quarters (military recruits, dorm students)
• Complications: autoimmune hemolytic anemia (IgM against I antigen on RBC),
erythema multiforme
Chlamydia • 2nd most common cause of atypical pneumonia in young adults
pneumoniae
Respiratory • Most common cause of pneumonia in infants
syncytial virus (RSV)
Cytomegalovirus • Associated with posttransplant immunosuppressive therapy
(CMV)
Influenza virus • Commonly seen in elderly, immunocompromised, or people with preexisting lung
disease
• Increases risk for S aureus or H influenza secondary pneumonia
Coxiella burnetti • Pneumonia with high fever (Q fever; generally, pneumonia has low fever)
• Associated with farmers and veterinarians
• Coxiella is different from rickettsia in 3 ways: causes pneumonia, does not require
arthropod for transmission (transmitted as spores), does not produce rash
5. What are four classic phases of lobar pneumonia?
- Congestion - due to edema
- Red hepatization - due to neutrophil and RBC exudate. Hepatization because previously spongy lung is now
tough due to fluid

Pulmonary Page 2.2


tough due to fluid
- Grey hepatization - breaking down of RBC makes lung gray.
- Resolution - lung tissue is regenerated by type II pneumocytes

6. What is aspiration pneumonia? What are it's causes? What's it's presentation?
• Seen in patients at risk for aspiration (ex - comatose, alcoholics)
• Causes: anaerobic bacteria of oropharynx:
○ Bacteroides
○ Fusobacterium
○ Peptococcus
○ Kliebsiella (is it anaerobic?)
• Classic presentation:
○ Right lower lobe abscess

Tuberculosis
1. Describe presentation of primary TB.
• Caused by inhalation of bacteria.
• Presentation:
○ Focal caseating necrosis classically in lower lobe and hilar lymph nodes
○ The foci undergo fibrosis and calcification resulting in Ghon complex
○ Mostly asymptomatic
○ Leads to positive PPD

Fig: Ghon complex (calcified and fibrosed lung): classic location is subpleural region near hylar nodes

1. Describe presentation of secondary TB (aka reactivated TB).


• Commonly seen due to immunosuppression, AIDS, or old age
• Presentation:
○ Usually affects upper lobes
○ Forms many focal caseating necrosis, or miliary TB or TB bronchopneumonia
○ Symptoms:
▪ Fevers and night sweats
▪ Cough with hemoptysis
▪ Weight loss

9. What are classic locations for spread of miliary TB?


• Kidney - most common organ to be involved - gives sterile pyura
• Meninges (classic location is base of brain)
• Cervical lymph nodes
• Lumbar vertebrae (pott disease)

Pulmonary Page 2.3


9.4 Chronic Obstructive Pulmonary Disease

1. What is COPD? What are some findings?


• COPD is obstruction to getting air out of lungs.
• Findings:
○ Low FEV1:FVC ratio - Decreased FVC, even more low FEV1
○ Normal FEV1:FVC is 80%. Normal TLC = 7 L.
○ Increased TLC due to air trapping

2. What are some causes of COPD?


- Chronic bronchitis (blue bloaters)
- Emphysema (pink puffers)
- Asthma
- Bronchieatasis

Chronic bronchitis
3. What is chronic bronchitis? What is the mechanism?
- chronic productive cough lasting at least 3 months over a minimum of 2 years
- hypertrophy of bronchial mucinous glands--patients cough up cups of mucous

4. Describe pathophysiology of chronic bronchitis and histology of airway?


- Below lamina propria are serous glands (secrete water to humidify air) and mucous glands
(secrete mucus to trap pollutants).

Pulmonary Page .1


(secrete mucus to trap pollutants).
- With chronic smoking, mucus glands undergo hypertrophy and hyperplasia. This causes tons of
mucus production, and the mucus can plug airways causing hypoxemia.
- Other:
○ Epithelium is pseudostratified columnar
○ Lamina propria has venules that warm the cold air coming from outside

Fig: Cross section of chronic bronchitis. Top right portion has respiratory epithelium, bottom left
has cartilage. It's clearly visible that mucus glands make >50% thickness of airway.

5. With what is chronic bronchitis highly associated?


- Smoking

6. What are the clinical features of chronic bronchitis?


- productive cough due to excessive mucous production,
- cyanosis ('blue bloaters') - mucus plugs trap CO2;
- increased risk of infection (anytime you plug a tube, it increases risk of infection behind the block)
- Reid index Increases to >50 from <40%;
- Cor pulmonale (pulm HTN) - due to globally low PAO2 in lungs (low PAO2 induces
vasoconstriction)

7. What is Reid index?


- It's the ratio of thickness of airway mucous gland to total thickness of airways. Normally, it's <40%.

Emphysema
8. What is emphysema?
• Destruction of alveolar air sac and multiple sacs combine to become one.
• Due to loss of elasticity of air sacs, lung becomes like a shopping bag, not effectively expelling air
out.
• Also, elasticity of air sacs attached to bronchioles keep the bronchiole open during expiration.
When the sacs are gone, then bronchioles collapse during expiration.

Pulmonary Page .2


Fig: Histology of emphysema showing multiple air sacs combining to make one.

9. What is pathophysiology of emphysema?


• Imbalance between protease and antiprotease. Inflammation induces high protease activity. So,
smoking leads to inflammation which leads to emphysema.

10. Differentiate two classic causes of emphysema (destruction of alveolar air sacs).
Smoking (no. 1 cause of emphysema) A1AT deficiency (alpha 1 antitrypsin) -
Pollutants cause inflammation that induce A1AT is an important antiprotease that
protease mediated damage to alveoli inhibits protease damage to alveoli
Centriacinar emphysema seen mainly in upper Panacinar emphysema seen mainly in lower
lobes (upper lobes have more air) lobes
Can cause liver cirrhosis too
Complications: hypoxemia and cor pulmonale Complications: hypoxemia and cor pulmonale
(pulm HTN) (pulm HTN)

11. What is acinus?


• Functional unit of lung (a terminal broncheole and alveoli associated with it)

12. Why does A1AT deficiency cause cirrhosis?


- A1AT deficiency is due to misfolding of mutated protein, mutant A1AT accumulates in ER of
hepatocytes which results in liver damage.
- A1AT is doesn't go to blood and lung because it's not exported by liver.

13. What does liver biopsy in A1AT deficiency show? (HY)


- Pink-purple, PAS-positive (a stain) globules in hepatocytes. Note - mucin and tropheryma whipplei
are also PAS +ve

Pulmonary Page .3


14. Describe the genetics of A1AT transmission.
- PiM - normal allele
- PiZ - mutant allele
PIMM • Norman healthy person
PiMZ - Haterogytoes; usually asymptomatic (low circulating A1AT)
- Significant emphysema risk with smoking
PiZZ - Homozygous mutant
- Significant risk for panacinar emphysema and cirrhosis

15. What are clinical presentation of emphysema?


- Dyspnea and cough
- Minimum sputum - contrast to chronic bronchitis
- Prolonged expiration with pursed lips (pink puffer) - pursed lips create back pressure to prevent
airway collapse (pt are not cyanotic because they are oxygenated; in chronic bronchits,
broncheoles are plugged up)
- Barrel chest - increased anterior-posterior diameter of lung
- Weight loss - use muscles to breathe
- Late complication:
○ Cor pulmonale
○ Hypoxemia in late stage due to loss of capillaries
Asthma
16. What is asthma?
- Reversible airway bronchoconstriction, most commonly seen due to allergic stimuli (atopic
asthma)
- Associated with allergic rhinits, eczema (atopic dermatitis) and family history of atopy (tendency
of type I hypersensitivity reactions)
- Commonly seen in kids

17. What is pathogenesis of asthma? (HY)


- Allergens induce TH2 phenotype in genetically susceptible patient. TH2 secretes:
IL-4 Induces class switching to IgE
IL-5 Attracts eosinophils
IL-10 Stimulates TH2 and inhibits TH1
- IgE coats mast cells and next time same allergen is encountered, massive mast cell degranulation
occurs.
- Histamines (arteriolar vasodilation and increase vascular permeability) and leukotriene
(vasoconstriction, increased vascular permeability (by constricting pericytes), and
bronchoconstriction) are released by mast cells.

Pulmonary Page .4


bronchoconstriction) are released by mast cells.

18. What is late phase reaction in asthma? (HY)


- Eosinophils release MAJOR BASIC PROTEIN that damages cells and induces bronchoconstriction

19. What are clinical features of asthma?


- Productive cough, dyspnea, wheezing in response to allergen exposure (episodic).

20. What are biopsy findings in asthma?


- Curschmann spirals (spiral shaped mucus plug)
- Charcot-leyden crystals (HY) (eosinophil derived MAJOR BASIC PROTEIN that indicate eosinophilic
inflammation)

Fig: Carcot leyden crystal on left and curschmann spiral on right

21. What are nonallergic causes of asthma?


- Exercise
- Viral infection
- Asprin (HY)
- Occupational exposure

22. What is presentation of asprin intolerant asthma?


- Nasal polyps (nasal polyp in kids highly associated with CF)
- Bronchospasm with aspirin

Bronchiectasis
1. What is bronchieatasis?
- Necrotizing damage to airway walls lead to permanent dilation of bronchioles
- Imagine if you blow air into a big tube, the air will just move randomly inside the tube and might
not come out

Pulmonary Page .5


Fig: large dilated structures are airway, not coeleaced alveoli

2. What is pathophys of bronchiectasis?


- Loss of muco-ciliary clearance system is the main problem. Mucus accumulation followed by
bacterial overgrowth leads to pus filled infection and permanent dilation of airways.

3. What are come cauases of bronchiectasis?


Causes:
○ CF (classic pt)
○ Allergic bronchopulmonary aspergillosis - classically seen in asthamits and CF pt.
○ Kartagener syndrome (mutation of dyenin arm cilia)
○ Tumor or foreign body that blocks airway (infection behind block can cause necrosis).

3. What is presentation and complication of bronchiectasis?


- Cough, dyspnea and foul smelling sputum
- Complication:
○ secondary systemic amyloidosis (HY) - systemic increase in SAA (an acute phase reactant)
produced chronically due to chronic inflammation. SAA is converted to AA that's deposited.
○ Hypoxemia and cor pulmonale

4. What is presentation of Kartagener syndrome?


- Sinusitis (cilia in nasal sinus not working well)
- Infertility
- Inversion of body organs (ex - heart on right)
- Bronchiectasis

Pulmonary Page .6


9.5 Restrictive Diseases

1. What are 4 examples of restrictive lung disease?


- Idiopathic (10), 20 pulmonary fibrosis
- Pneumoconioses
- Sarcoidosis
- Hypersensitivity pneumonitis (pigeon breeder's lung)

2. What are etiologies of 10 and 20 pulmonary fibrosis?


Primary Increased TGF beta --> induce fibrosis
Secondary Amiodorone, bliomycin, radiation
You need lung transplant for pulmonary fibrosis

3. What is pneumoconiosis? What's its pathophysiology?


- Interstitial lung disease caused due to chronic occupational exposure with fibrogenic material.
- Pathophys - macrophages ingest fibrogenic material and induce fibrosis

4. What are some examples of pneumoconiosis?


Example Risk group Complication Presentation
Silicosis Sand blasters - High risk of TB (inhibits - Silicotic nodule (lots of collagen with
phagolysosome formation) minimum inflammation)
- High risk for lung
carcinoma
Berylliosi Beryllium High risk for lung cancer Non caseating granuloma in hylar nodes
s miners and other organs
Aerospace
workers
Asbestos Construction High risk of lung carcinoma - Asbestos body (ferruginous body) in
is workers (more) and mesothelioma biopsy - ferritin and hemosiderin coat
Plumbers asbestos
Shipyard - Pleural plaques
workers

Pulmonary Page .1


Fig: A: Asbestos body (ferritin and hemosiderin coated asbestos particle); B: pleural plaque on
diaphragmatic pleura; C: Silicotic nodule, notice lots of collagen with minimum inflammation

5. What is presentation of sarcoidosis?


- Sarcoidosis is non-caesating granuloma in multiple organs (most commonly localized in lungs and
hyalar lymph nodes)
- Presentation: non specific - fatigue, wt loss, joint pain and arthritis
- Others: uveitis, erythema nodosum, cardiac sarcoidosis, neurosarcoidosis (affects CN often)etc
- Hypercalcemia (HY) -
○ 1 alpha hydroxylase activity of epithelial histiocytes convert vit D to active form
- Asteroid bodies are seen in biopsy (not-specific; can be seen in giant cells of any granulomas)

Fig: asteroid bodies seen in giant cells of granulomas (present in sarcoidosis)

6. What is hypersensitivity pneumonitis (aka extrinsic allergic alveolitis)?


- It is granuloma and interstitial inflammation caused due to inhaled organic or non-organic matter
(aka pigeon breeder's lung).
- Pathophys:
○ Ab-Ag complex forms in lung that activates neutrophils and eventually lymphocytes.
Lymphocytes mediate most damages
- Cause:
○ many organic and non-organic matters
- Presentation:
○ fever/cough hours after exposure
○ Chronic exposure leads to interstitial lung diseaese

7. How is iron stored in body?


- Free Fe produces ROS by Fenten reaction so cells store free Fe in ferretin protein or hemosiderin
Ferretin Hemosiderin
Intracellular protein that acts as buffer against Fe overload or Intracellular complex made of
shortage (protein can be secreted too) Ferretin and other stuff
Fe in Ferretin can be given out when needed Fe here is poor source to supply
to body

Pulmonary Page .2


to body
Plasma Ferretin correlates well with total Fe in body; so serum
ferretin a common test to access anemia.

Pulmonary Page .3


9.6 Pulmonary Hypertension

1. What is pulmonary hypertension?


- Mean arterial pressure in pulmonary circulation >25 mm Hg (normal is 10 mm Hg) is pulmonary
hypertension

2. What are causes of pulmonary HTN?


- Causes can be divided to primary and secondary.
Primary pulmonary HTN Secondary pulmonary HTN
- Etiology unknown (some familial forms - Hypoxemia (COPD, interstital lung disease)
related to BMPR2 mutation (HY) - see - Increased volume in pulm circuit (congenital heart
proliferation of vascular smooth muscle) disease)
- Recurrent pulmonary embolism (HY)
- Classically seen in young adult females

3. What are clinical characters of pulmonary hypertension?


- Characterized by atherosclerosis of pulmonary trunk (HTN leads to artherosclerosis) and smooth
muscle hypertrophy of pulmonary arteries, intimal fibrosis
- See plexiform lesions with severe long standing disease
- Right ventricular hypertrophy with cor pulmonale
- Exertional dyspnea and right sided heart failure - if you see a young woman with exertional
dyspnea, think pulmonary hypertension

Fig: plexiform changes - tuft of capillaries that arise as a complication of HTN

Pulmonary Page 5.1


9.7 Respiratory Distress Syndrome (RDS)

Acute respiratory distress syndrome


1. What is ARDS?
- Respiratory distress due to damage to alveolar-capillary interface (diffuse alveolar damage).
Protein rich fluid leaks into air sac. Protein rich fluid and epithelial cells are reorganized to make
hyaline membrane.
- Membrane blocks air exchange; membranes are sticky and promote alveoli collapse.

Fig: hyaline membrane seen on alveoli represents ARDS (right); diffuse whiteout seen in Xray in
ARDS (left)

2. What are causes of ARDS?


- Mechanism is activation of neutrophils that induces protease and free radical damage of type I
and II pneumocytes.
- It's occurs secondary to (100's of causes)
○ Sepsis
○ Infection
○ Shock
○ Trauma
○ Aspiration
○ Pancreatitis
○ DIC
○ Hypersensitivity reactions
○ Drugs

3. What's presentation of ARDS?


- Hypoxemia and cyanosis (due to thickened diffusion barrier and collapse of air sacs)
- White out on chest x-ray
- Hyaline membrane in alveoli on biopsy

4. How do you treat ARDS? What makes recovery complicated?


- Ventilation with positive end-expiratory pressure (PEEP) - every time patient breathes out, the
lung collapses a little more due to hyaline membrane. Positive pressure at end of respiration
prevents this.
- Treat underlying cause
- Recovery may be complicated due to interstitial fibrosis. Type II pneumocytes (stem cells) are
damaged and new cells can't be generated. (HY)

Pulmonary Page 6.1


Neonatal respiratory distress syndrome
1. What is neonatal respiratory distress syndrome?
- Respiratory distress in newborn due to inadequate surfactant production. Alveoli collapse and
hyaline membranes are seen.

2. How is enough surfactant production determined in fetus?


- Surfactant (phosphatidylcholine aka lecithin) is produced by type 2 pneumocytes from 28 weeks;
enough levels are not reached until 34 weeks (childbirth occurs in 40th week)
- Amniotic fluid ratio of lechitin to sphingomyelin >2 indicates enough surfactant production

3. What are some causes of neonatal respiratory distress syndrome?


- Premature birth
- Caesarian section delivery - child lacks stress induced steroids; steroids increase surfactant
production and release from pneumocytes
- Maternal diabetes - insulin decreases surfactant production (high blood sugar from mom causes
high insulin production in baby)

4. What is presentation of neonatal respiratory distress syndrome?


- Hypoxemia and cyanosis
- Ground glass appearance on X-ray
- Increased use of respiratory muscle; use of accessory muscle, grunting

5. What are it's complication?


- High risk of patent ductus arteriosus and necrotizing enterocolitis (decreased
- Supplemental oxygen used for treatment increases risk of free radical injury
○ retinal injury leads to blindness (O2 treatment for ARDS imp cause of blindness in neonates)
○ lung damage leads to bronchopulmonary dysplasia

Pulmonary Page 6.2


9.8 Lung Cancer

1. What are key risk factors for lung cancer?


- Cigarette smoking (85% of lung cancer)
○ Risk directly linked to duration and amount of smoking (pack years)
- Radon (2nd most common cause) - most common ionizing radiation exposure in USA
○ Colorless, odorless gas
○ Decay product of uranium
○ Found in soil, accumulates in closed space (basement)
- Asbestos

2. What is carcinogenicity of cigarette smoking?


- Contains > 60 carcinogens
- Polycyclic aromatic hydrocarbons and arsenic are particularly carcinogenic
- Cancer risk directly increases with duration and amount of smoking (pack years)

2. What is presentation of lung cancer?


- Average age at presentation is 60.
- Most common cause of cancer death in USA
- Nonspecific presentation -
○ Cough, wt loss, hemoptysis, post obstructive pneumonia

3. How do you diagnose lung cancer?


- Diagnosis requires biopsy
- Imaging reveals solitary nodule (coin-lesion) - growing lesion concerning
- Coin lesions also seen in (HY):
○ Granulomas - TB, fungus (ex - histoplasma in midwest)
○ Bronchial hamartoma - benign tumor of lung tissue + cartilage; often calcified in imaging
○ Harmartoma - disorganized mass that grows at same rate as surrounding tissue; made of same cells that
makes the tissue

4. Describe the TNM staging of lung cancer.


- T - tumor size
i. Pleural involvement classically seen in adenocarcinoma (adenocarcinoma is peripheral)
ii. Obstruction of SVC (superior vena cava syndrome) - distended head and neck veins with edema and blue
discoloration of arms and face
iii. Involvement of recurrent laryngeal nerve (hoarseness) or phrenic nerve (diaphragmatic paralysis)
iv. Horner's - compression of sympathetic chain (ptosis, anhydrosis - in skin, miosis) - especially if tumor is at
apex of lung (pancoast tumor)
- N-
○ Spread to hilar and mediastinal lymph nodes
- M-
○ Unique site of distant metastasis is adrenals (HY)
○ Others - brain, bone, liver

5. What's prognosis of lung cancer?


- Poor (no effective screening method) - 5 year survival rate is 15%

6. What are two categories of lung cancer?


Small cell carcinoma (15% of all lung carcinoma) Non-small cell carcinoma (85% of all lung carcinoma)

Pulmonary Page 7.1


- Adenocarcinoma (40%) - glands or mucus production
- Squamous cell carcinoma (30%) - keratin pearls or intercellular
bridges
- Large cell carcinoma (10%) - none of above features seen
- Carcinoid tumor (5%)
- Usually no amenable to surgery (treat with - Treat upfront with surgery (doesn't respond well to
chemotherapy and radiation) chemotherapy)

7. What are different types of lung cancer?


S.N Cancer type Remarks
1 Small Cell carcinoma - Treat with chemotherapy
2 Non-small cell carcinoma Subtype
Squamous cell carcinoma
Adenocarcinoma
Broncheoalveolar carcinoma (Adenocarcinoma in situ)
Large cell carcinoma
(Bronchial) carcinoid tumor

3 Mesothelioma - related to asbestos


4 Metastasis - Common origin of metastasis - breast, colon

8. Classify the different types of lung cancer.


Neuroendocrine Adenocarcinoma Related to smoking - small, Paraneoplastic Undifferentiat Excellent
(NE) tumor large, squamous, adeno syndrome ed and poor prognosis
prognosis
- Small cell - Adenocarcinom - Squamous cell carcinoma - Squamous cell - Small cell - Bronchoal
carcinoma a (most common in male carcinoma carcinoma veolar
(poorly smokers (PTHrp) carcinoma
differentiated
NE cells)
- (Bronchial) - Bronchioalveola - Small cell carcinoma (male - Small cell - Large cell
carcinoid tumor r carcinoma smokers) carcinoma carcinoma
(well (adenocarcinom (ADH, ACTH, Ab
differentiated a in situ) for Ca channel)
NE cells)
- Adenocarcinoma (most - Large cell
common in female smokers carcinoma (B-
and non-smokers) - HCG)
broncheoalveolar carcinoma
not associated with smoking
- Large cell carcinoma
9. Describe the following types of cancer.
Cancer Histology Association Location Remarks
Small cell - Poorly differentiated small cell - Male smokers Central - Associated with 5A and 1B
carcinoma and very aggressive (99% of small cell - Produces ACTH
- Arise from neuroendocrine carcinoma pt are - Produces ADH
cells (Kulchitsky cells) smokers) - Produces Ab for Eaton-
- Chromogranin +ve (less +ve Lambert syndrome

Pulmonary Page 7.2


- Chromogranin +ve (less +ve Lambert syndrome
than carcinoid tumor) (presynaptic Ca channel Ab)
(paraneoplastic syndromes)
- Anti-neuronal antibody
syndrome (limbic encephalitis,
cerebellar degeneration,
opsoclonus, GI dysmotility,
poly radiculopathy)
- Amplification of myc
oncogene
- LOVES TO GO TO BRAIN - give
prophylactic cranial
irraditation
- MOST AGGRESSIVE TYPE OF
LUNG CANCER
Adenocar - Glands or mucin - Most common Peripheral
cinoma tumor in
nonsmokers and
female smokers

Fig: glandular structure in


adenocarcinoma
Squamous - Keratin pearls or intercellular - Most common Central - May produce PTHrp
cell bridges (by definition) tumor in male (paraneoplastic syndrome)
carcinoma smokers - Hilar mass from bronchus
- Associated with double C
• HyperCalcemia- due to
PTHrp
• Cavitation

Fig-
intracellular
Fig: keratin pearl bridge
Large cell - Poorly differentiated and highly - Smoking Central or - Poor prognosis
carcinoma anaplastic cells (no keratin peripheral - Poor response to
pearls, intercellular bridges, chemotherapy; remove
glands or mucin) surgically
- Paraneoplastic - may secrete
B-HCG
(Bronchial - Well differentiated - Not related to Central or - MOST COMMON PRIMARY
) neuroendocrine cells; smoking peripheral LUNG CANCER IN CHILDREN
Carcinoid chromogranin positive (when central, - Low grade malignancy; rarely,
tumor makes polyp can cause carcinoid
like mass in syndrome - caused due to
bronchus) release of vasoactive
substance (mainly serotonin) -
flushing, diarrhea, restrictive
cardiomyopathy due to

Pulmonary Page 7.3


Carcinoid (when central, Low grade malignancy; rarely,
tumor makes polyp can cause carcinoid
like mass in syndrome - caused due to
bronchus) release of vasoactive
substance (mainly serotonin) -
flushing, diarrhea, restrictive
cardiomyopathy due to
endocardial fibrosis
Fig: chromogranin positivity

Bronchiol - Columnar cells that grow along - Not related to Peripheral - Excellent prognosis
oalveolar preexisting bronchioles and smoking - Pneumonia like consolidation
carcinoma alveoli; arise from clara cells on imaging
(adenocar
cinoma in
situ)

Fig: normal alveoli top right;


columnar cells on rest
Metastasi - Most common source are Canon-ball - More common than primary
s breast and colon carcinoma nodules on tumors
imaging

Mesotheli - See psammoma bodies in - Highly associated - Malignant tumor of


oma biopsy (concentric with asbestos mesothelial cells
calcifications - other HY exposure (lung (mesothelium is a membrane
cancer - papillary thyroid, cancer more of simple squamous cells that
meningioma, papillary serous common in lines body cavities: pleura ,
ovarian) asbestos peritoneum, mediastinum and
exposure) pericardium)
- Tumor encases the lung

- Small cell carcinoma is poorly differentiated neuroendocrine tumor; carcinoid tumor is well differentiated
neuroendocrine tumor.
- Neuroendocrine cells have neurosecretory granules; chromogranin stains positive for neurosecretory granule.

Pulmonary Page 7.4


- Neuroendocrine cells have neurosecretory granules; chromogranin stains positive for neurosecretory granule.

Pulmonary Page 7.5


9.9 Pleura

1. Describe anatomy of pleura?


- It's lined by mesothelial cells
- It produces pleural fluid

2. What are differences between spontaneous and tension pneumothorax?


Spontaneous pneumothorax Tension pneumothorax
- Often due to rupture of - Often due to penetrating chest wall injury
emphysematous bleb
- Often seen in young adults
- Xray - trachea deviates to side of - X-ray - trachea pushed to opposite side of injury; medical
collapse emergency; put chest tube

Fig: spontaneous pneumothorax (no tracheal shift)

3. Describe mesothelioma (malignant neoplasm of mesothelial cells).


- Presentation:
○ Recurrent pleural effusion (mesothelial cells make pleural fluid)
○ Tumor encases the lung
- Risk factor:
Asbestos (lung cancer far more likely)

Pulmonary Page 8.1


○ Asbestos (lung cancer far more likely)

Fig: mesothelioma (tumor encasing the lung)

Pulmonary Page 8.2


Chapter 10: Gastrointestinal
Pathology
10.1 Oral Cavity

1. Describe the following oral cavity pathologies


Presentation Cause
Cleft palate - Cleft lip and palate usually occur Due to failure of facial prominence to fuse -
and lip together one superior, two from sides and two from
- inferior - in early pregnancy
Apthous - Painful superficial ulceration of oral - Arise in relation to stress but resolves
ulcer mucosa - gray base (granuloma) with spontaneously
surrounding erythema
Behcet - Recurrent apthous ulcer, genital - Can be seen in viral infection
syndrome ulcer and uveitis - Unknown etiology
Oral herpes - Dew drops in petal appearing painful - HSV 1 - usually occur in childhood
ulcer - Virus remains dormant in trigeminal
ganglia; stress and sunlight can activate
virus and cause ulcers again

2. Describe the squamous cell carcinoma of oral cavity.


Description - Malignant neoplasm of squamous epithelia of oral cavity
Risk factor - EtOH and tobacco
Common site - Floor of mouth
Precursor - Leukoplakia
lesions • White plaque that can't be removed (unlike candida) - represents
dysplasia
- Erythroplakia (red plaque)
• Vascularized leukoplakia and highly suggestive of squamous cell
dysplasia

3. Differentiate leukoplakia from hairy leukoplakia


- Both can't be scraped off.
- Hairy leukoplakia is caused by EBV, usually seen in immunocompromized patients on side of
tongue and is NOT precancerous
- Leukoplakia is cancerous.

Fig: erythroplakia (right), leukoplakia (middle), hairy leukoplakia (left - not precancerous)

GI Page 1.1
10.2 Salivary Gland

1. Describe the following pathologies of the salivary gland.


Presentation Cause Labs Risk
Mumps - Bilateral inflammation Mumps virus - Increased amylase Sterility,
of salivary glands (can be from especially in
- Orchitis pancreas too) teenagers
- Pancreatitis
- Aseptic meningitis
Sialaden - Mostly unilateral - Mostly due to obstructive
tis inflammation of stone leading to staph
salivary gland Aureus infection

2. Describe the following cancers of salivary gland (SG)


Cancer Epidemiology Location Composition Presentation
Pleomorphic Most common Mostly in Stromal (cartilage) and - Mobile painless circumscribed
adenoma tumor of SG parotids epithelial cells - mass in angle jaw
biphasic tumor - High recurrence; irregular
margins and extension across
capsule makes resection hard
- Rarely becomes carcinoma - if it
does, it will involve CN 7
Warthin 2nd most Mostly in Cystic tumor with
tumor common tumor parotids germinal center and
of SG lots of lymphocytes
Mucoepiderm Most common Mostly in Mucinous and - Involves CN 7
oid carcinoma malignant tumor parotids squamous cell
of SG

GI Page 2.1
10.3 Esophagus

1. Talk about features of following pathologies.


Pathology Description Presentation
Tracheoesop Most common variant has proximal esophageal atresia with - Vomiting,
hageal fistula distal esophagus arising from trachea - polyhydroamnio
s,
- aspiration,
- abdominal
distension

Esophageal Protrusion of esophageal mucosa, most often in upper - Dysphagia for


web esophagus poorly chewed
food

Plummer ○ Severe iron deficiency anemia


venson ○ Esophageal web
syndrome ○ Beefy red tongue due to atrophic glossitis (inadequate cell
turnover)
○ Usually seen in post-menopausal woman
Zenker - (false diverticulum) - only mucosa protrudes in upper - Halitosis (rotting
diverticulum esophagus at junction of esophagus and pharynx food in
diverticulum)
- Dysphagia
- Obstruction

Mallory - Longitudinal laceration of mucosa at gastroesophageal - Painful

GI Page 3.1
Mallory - Longitudinal laceration of mucosa at gastroesophageal - Painful
Weiss junction hematemesis
syndrome - Cause - vomiting - usually alcoholism of bulimia
- Risk for Boerhaave syndrome

Boerhaave - Esophageal rupture leading to air in mediastinum and


syndrome subcutaneous emphysema (crepitis)

Esophageal varices
Definition - Dilated submucosal veins in lower esophagus due to portal hypertension

Anatomy - Esogpageal vein is connected to portal vein via left gastric vein
Epidemiology - Most common cause of death in cirrosis pt
Presentation - Asymptomatic or present as painless hematemesis

Achalasia
Definition - Disordered esophageal motility with inability to relax lower esophageal sphincter
(LES) due to damage of ganglion cells in myenteric plexus
Etiologies - Chagas disease (HY)
- Idiopathic
Clinical - Dysphagia of solid and liquid food
features - Putrid breath (rotting)
- High LES pressure on esophageal manometry
- Bird beak sign on X-ray

GI Page 3.2
Complicatio - Increased risk for squamous cell carcinoma
n

Gastroesophageal reflux disease (GERD)


Definition - Reflux of stomach acid due to reduced LES tone
Risk factor - Hiatal hernia
- EtOH, tobacco
- Obesity, fat rick diet
- Caffeine
Clinical features - Heartburn (mimics cardiac chest pain)
- Asthma (adult-onset) and cough
- Damage to enamel of teeth
Complication - Barret esophagus (in 10% of pt with GERD) and strictures

Barret's esohagus
Definition - Metaplasia in lower esophageal mucosa from stratified squamous epithelium to
columnar epithelium with goblet cells
Complicatio - May progress to adenocarcinoma
n
Histology

Fig: squamous epithelia on left, columnar epithelia with goblet cells (right) - barret's
esophagus

Esophageal carcinoma
Adenocarcinoma Squamous cell carcinoma
Epidemiolo - Most common esophageal cancer in western - Most common esophageal cancer
gy world (classically seen in lower 1/3rd of worldwide
esophagus)
Risk factor - Barret esophagus - EtOH and tobacco (most common
cause)
- Achalasia
- Esophageal web (ex- Plummer-
Vinson synd)

GI Page 3.3
Vinson synd)
- Esophageal injury (ex - lye
ingestion)
- Very hot tea
Prognosis - Poor (both type presents late)
Presentation - Progressive dysphagia of solid and liquid food
- Weight loss, pain
- Hematemesis
- SCC may have hoarse voice (recurrent laryngeal nerve involvement) and
cough (tracheal involvement) (HY)
Lymph node If cancer in upper 1/3rd of Cervical nodes
involvement esophagus
Middle 1/3rd Mediastinal or tracheobronchial
nodes
Lower 1/3rd Celiac and gastric nodes

GI Page 3.4
10.4 Stomach

Sections of stomach

Gastroschisis and Omphalocele (schisis = break; omphalo = navel; cele = swelling)


Gastroschisis - Absence of anterior abdominal wall and exposure of abdominal contents outside
Omphalocele - Persistent herniation of bowel into umbilical cord
- Due to failure of herniated intestines to return to body cavity, contents are covered by peritoneum and amnion
of umbilical cord

Fig: omphalocele (middle), gastroschisis (right)

Pyloric stenosis
Definition - Congenital hypertrophy of pyloric smooth muscle
Epidemology - M>F
- Presents after 2 weeks of birth
Presentation - Visible peristalsis and projectile non-bilious (non-green) vomiting after eating
- Olive like mass on abdomen

Fig: visible peristalsis after eating in a kid with pyloric stenosis


Treatment - Myotomy (cut open pyloric sphincter

Acute and chronic gastritis

GI Page 4.1
Acute and chronic gastritis
Acute gastritis Chronic gastritis
Definition - Acidic damage to stomach mucosa - Chronic inflammation of stomach mucosa that leads to intestinal
metaplasia
Defense - Mucin produced by foveolar cells
against - Bicarb secretion
acid - Normal blood supply (provides nutrition
and picks up acid

Fig: intestinal metaplasia seen in chronic gastritis


Etiologies - Severe burn (aka Curling ulcer) - due to - Chronic autoimmune gastritis
loss of blood supply due to hypovolemia - Chronic H. Pylori gastritis
- Increased intracranial pressure (aka
Cushing ulcer) - causes increased
stimulation of vagus nerve and increased
acid production
- NSAIDs - decrease PGE2 (PEG2 decreases
acid production, stimulate mucus, bicarb
production and increases blood flow)
- Chemotherapy
- Shock
- Heavy alcohol consumption

Differentiate chronic autoimmune gastritis and chronic H pylori gastritis.


Chronic autoimmune gastritis Chornic H pylori gastritis
Defn - T cell mediated damage to gastric parietal cells (type 4 - H pylori produces urease to create basic environment; and
hypersensitivity) protease damages mucosal defense - acidic damage to
stomach
Epi. - Most common cause of gastritis (90%)
- Antrum is most common site
Cli. - Atrophy of mucosa and intestinal metaplasia (stomach - Intestinal metaplasia - high risk of intestinal type
Feat. biopsy looks like intestine) - high risk of stomach adenocarcinoma of stomach
adenocarcinoma - Epigastric abdominal pain
- Achlorhydria (low acid) with increased gastrin levels - Increased risk of ulceration
and antral G cell hyperplasia
- Megaloblastic anemia (pernicious anemia) due to lack
of IF
Lab - see Ab against parietal cells and/or intrinsic factor (IF) - +ve urea breath test
- Stool antigen seen
Treat. - Triple therapy (PPI + marcolide (clarithromycin) + amoxicillin
- Intestinal metaplasia, gastritis and ulcer goes away after
treatment
Comp. Increased risk of gastric adenocarcinoma (intestinal Increased risk of adenocarcinoma (intestinal type) and
type) MALToma

GI Page 4.2
type) MALToma

Peptic ulcer disease


Definition - Solitary mucosal ulcer involving proximal duodenum (90% of cases) or distal stomach (10%)
Differenc Duodenal ulcer Gastric ulcer
es
Cause - 95% due to H. pylori - 75% due to H. pylori
- Rarely due to Zolinger Ellision (ZE) syndrome - NSAID use
- Bile reflux
Present. - Epigastric pain that improves with meal - Epigastric ulcer that worsens with
meal
Location - Mostly in anterior duodenum. - Mostly in lesser curvature of
antrum
Risk - If ulcer in posterior duodenum, rupture may lead to bleeding - Risk of bleeding from left gastric
from gastroduodenal artery or pancreatitis artery
Biopsy - Ulcer with hypertrophy of Brunner glands
Risk of - Duodenal ulcers almost never malignant (duodenal carcinoma - May be caused by gastric
carcinoma extremely rare) carcinoma (intestinal subtype)

How do you differentiate if gastric ulcer is due to gastric carcinoma or not?


Benign gastric peptic ulcer Malignant gastric peptic ulcers (intestinal type)
- Usually small (<3cm), sharply demarcated (punched out), and - Usually large and irregular with heaped up margins
surrounded by radiating folds of mucosa (biopsy is needed for
definitive diagnosis)

Gastric carcinoma (adenocarcinoma)


Classes - Intestinal type (more common)
- Diffuse type
Presentatio - Presents late with weight loss, abdominal pain, anemia and early satiety
n - Rarely presents as acanthosis nigricans (hyperplasia and darkening of epidermis) and Leser-Trelat sign (lots of
seborrheic keratosis)
Metastasis - Liver (most common)
- Periumbilical region (sister mary joseph nodule) Intestinal type

GI Page 4.3
- Bilateral ovary (Kukenberg tumor) Diffuse type

Differentiate intestinal and diffuse type of gastric carcinoma


Intestinal type Diffuse type
Epid More common
Presentati - Large irregular ulcer with - Prominent desmoplasia and thick stomach wall (linitis plastica)
on heaped up margin

Location - Lesser curvature of gastric


antrum (similar to gastric ulcer)
Risk - Intestinal metaplasia ( H. pylori - Not associated with any of the previous
factor and autoimmune gastritis)
- Nitrosamine (present in
smoked food- japan has high
rates)
- Blood type A
Histology - Signet ring cells that diffusely infiltrate the gastric mucosa - nucleus pushed to
edge as tumor cell produces lots of mucin - looks like ring

GI Page 4.4
Mets - Periumbilical region (sister - Bilateral ovary (Kukenberg tumor)
mary joseph nodule)

GI Page 4.5
10.5 Small Bowel, 10.6 Appendix

Duodenal atresia
Defn - Failure of duodenum to canalize
Association - Down syndrome
Clinical features - Bilious vomiting
- Polyhydramnios
- Distension of stomach and blind loop of duodenum (double bubble sign)

Fig: bilious vomit (left), double bubble sign (right)

Merkel diverticulum (MOST COMMON CONGENITAL GI ANOMALY)


Defn - Outpouching of all three layers of bowel wall (true diverticulum) due to failure of vitelline
duct to involute (partial involution)
Anatomy - Developing midgut receives nutrients through vitelline duct from yolk sack
- Vitelline duct forms in 4th week and involutes in 7th week
Partial involution of vitelline duct Merkel diverticulum
No involution Meconium comes out through umbilicus

Rule of 2 - Seen in 2% of population


- 2 inches long
- 2 feet away from ileocecal valve
- Presents within first 2 year of life
Presentation - Most cases are asymptomatic
- Bleeding due to heterotopic gastric mucosa
- Volvulus, intussuception (telescoping) or obstruction (CAN MIMIC APPENDICITIS)

Volvulus and intussuception


Volvulus Intussusception
Defn - Twisting of bowel along its - Telescoping of proximal bowel forward into distal segment
mesentery - Results in obstruction and infraction
- Results in obstruction and
ischemia

GI Page 5.1
ischemia
Cause - Associated with leading edge (focus of traction)
• In kids, most common cause is rotavirus related
lymphoid hyperplasia - mostly occurs in terminal ileum
leading to intussuception into cecum
• In adults most common cause is tumor
Pres - Most common location are - See currant jelly stool
sigmoid colon (elderly),
cecum (young adult)

Small bowel infraction


Feature - Small bowel highly susceptible to infraction
- Abdominal pain, bloody diarrhea and decreased bowel sounds
Types Cause
Transmural - Thrombosis/embolism of SMA (a-fib, vasculitis) or thrombosis of
infraction mesenteric vein (polycythemia vera, lupus)
Mucosal - Hypotension
infraction

Lactose intolerance
Defn - Hypofunction of lactase enzyme found in brush-border of enterocytes. Enzyme breaks
lactose into glucose and galactose
Presentation - Lactose osmotically active (abdominal distension and diarrhea after eating milk products)
Cause - Congenital - autosomal recessive
- Acquired - develops in late childhood
- Temporary - after infection (lactase is highly susceptible to injury)

Celiac disease
Defn - Immune mediated damage to small bowel villi due to gluten exposure
- Damage most prominent in duodenum (Fe deficiency)
Genetics - Related to HLA DQ2 and HLA DQ8
Pathophys - Gliadin pathogenic component in gluten. After absorption, its deamidated by tissue
transglutaminase (tTG). Deamidated gliadin is presented by APC. CD4 T cells mediate tissue
damage.
Presentati - Chlidren present with failure to thrive, abdominal distention diarrhea
on - Adults present with chronic diarrhea and bloating
- Dermatitis herpetiformis - small, herpes like vesicles on skin due to IgA deposition on
dermal papillae

GI Page 5.2
dermal papillae
- IgA deficiency may be seen
- All symptoms resolve with gluten free diet
Lab - IgA against endomysium, tTG, or gliadin
findings - IgG against those antigens if pt have IgA deficiency
Biopsy - Flattening of villi, hyperplasia of crypts, increased intraepithelial lymphocytes

Fig - Flattened villi with crypt hyperplasia in Celiac (left) and normal villi (right)
Complicati - Small bowel carcinoma and T cell lymphoma despite good dietary control
on

Tropical sprue
Defn - Damage to small bowel villi due to unknown organism (disease responds to
antibiotics) resulting in malabsorption
- Biopsy finding similar to Celiac?
Difference from - Damage most prominent in jejunum and ileum (Vit B12 and folate deficiency - I fuck
celiac bitches in France) - In Celiac, damage is mostly in duodenum
- Arises after infectious diarrhea and responds to antibiotics
- Occurs in tropical regions - ex-Caribbean

Whipple disease
Defn - Systemic tissue damage due to macrophages loaded with Tropheryma whippelii
Presentati - Classic damage is done in small intestine lamina propria
on • Macrophages compress lacteal and interfere with chylomicron transport from
enterocytes - fat malabsorption and steatorrhea
- Other sites include joints (arthritis), cardiac valves, lymph nodes and CNS
Biopsy - Foamy macrophages in lamina propria of small intestine
- Tropheryma whipplii stain PAS +ve inside the lysosome of macrophage (mucin also PAS
+ve)

GI Page 5.3
Fig - PAS stain in whipple disease (mucin in goblet cells and bacteria inside macrophages
are staining pink)

Abetalipoproteinemia
Defn - Genetic lack of apolipoprotein B-48 and B-100 (autosomal recessive)
Presenta - Absent plasma VLDL and LDL (require B-100)
- Fat malabsorption due to defective chylomicron formation (requires B-100)

Carcinoid tumor
Defn - Malignant proliferation of neuroendocrine cells; low grade malignancy
Clinical - Can arise anywhere in gut; small bowel most common site
features - Grows as submucosal polyp-like nodule
- Often secretes serotonin (carcinoid syndrome) - asymptomatic as MAO in liver
metabolizes it to 5-HIAA (see 5-HIAA in urine)

Carcinoid - Seen after tumor metastasizes to liver (bypass hepatic circulation) and
syndrome (HY) secretes serotonin there
Trigger - EtOH or emotional stress that triggers serotonin release
Presentati - Bronchospasm
on - Facial flushing
- Diarrhea
- Carcinoid heart disease - right sided endocardial fibrosis, especially in
tricuspid valve and pulmonary valve (not seen in left heart due to
presence of MAO in lungs)

GI Page 5.4
presence of MAO in lungs)

APPENDIX
Acute appendicitis (MOST COMMON CAUSE OF ACUTE ABDOMEN)
Defn - Acute inflammation of appendix
Cause - Obstruction of appendix by lymphoid hyperplasia (common in kids) and fecolith (common
in adults)
Presen - Periumbilical pain, fever, nausea - pain eventually localizes to right lower quadrant
(McBurney point)
- Rupture results in peritonitis that presents with guarding and rebound tenderness
Complicatio - Periappendicular abscess common complication
n

GI Page 5.5
10.7 Inflammatory Bowel Disease (IBD)

Ulcerative colitis Chorn disease


Wall - Mucosal and submucosal layers - Full thickness with knife like fissures
involve
ment
Locatio - Begins at rectum and - Anywhere from mouth to anus with skip lesions;
n continuously moves back - Terminal ileum most common - Vit B12 deficiency
anywhere up to cecum - Rectum is least common

Fig: Chorn's (left); UC (right)


Sympto - LLQ pain (rectum) with bloody - RLQ pain (ileum) with non-bloody diarrhea
ms diarrhea
Inflam - Crypt abscess with neutrophils - Lymphoid aggregated with noncaseating granuloma in 50% of
mation cases

Gross - Psudopolyps - Cobblestone mucosa


appear - Los of haustra lead to lead pipe - Creeping fat
ance sign on imaging - Strictures

GI Page 6.1
Complic - Toxic megacolon - Malabsorption (terminal ileum mostly affected)
ation - Carcinoma (depends on extend - Calcium oxalate nephrolithiasis
and duration of disease - no - Fistula formation
problem until >10 years) - Carcinoma if colonic disease present
Smokin - Protects against UC - Increases chance of chron
g
Associa - 10 sclerosing cholangitis (P- - Ankylosing spondylitis, sacroilitis
tion ANCA +ve) - Migratory polyarthritis
- Erythema nodosum

GI Page 6.2
10.8 Colon

Hirschprung disease
Definition Failure of Myenteric (auerbach) plexus and submucosal (Meissner) plexus migration
to distal sigmoid colon and rectum
Association Down's syndrome
Presentati - Presentation based on defective peristalsis/obstruction
on • Failure to pass meconium
• Empty rectal vault on digital rectal exam (DRE)
• Megacolon with risk for rupture

Fig: normally, rectum is bigger than colon


Diagnosis Suction biopsy reveals lack of ganglion cells
Treatment Take the involved bowel out - surgery

Colonic diverticula
Definition Outpouching of colonic mucosa and submucosa through muscularis propria (false
diverticulum)

Cause - Increased wall stress


• Constipation, straining, low fiber diet (increased risk with old age)
• Arise where vasa recta transverses muscularis propria (weak point in colonic
wall)
• Most often found in sigmoid colon
Presentatio - Mostly asymptomatic
n - Rectal bleeding (hematochezia)
- Diverticulitis - due to obstruction of fecal material (appendicitis like symptom on
left lower quadrant)
- Fistula - inflamed diverticulum attaches to local structure
• Colovesicular fistula - presents as air or stool in urine

GI Page 7.1
Angiodysplasia and Hereditary hemorrhagic telangiectasia
1. Differentiate the above two.
Angiodysplasia Hereditary hemorrhagic telangiectasia
Acquired malformation of mucosal and Autosomal dominant disorder resulting thin walled
submucosal capillary beds blood vessels, especially in mouth and GI tract
Mostly seen in cecum and right colon
due to high wall tension
Rupture of capillary beds present as Rupture of blood vessels present as bleeding
hematochezia in old adults

Ischemic colitis (think like MI of colon)


Definition Ischemic damage to colon, mostly at splenic flecture - watershed area of SMA
Cause Artherosclerosis of SMA most common cause
Presentation - Postprandial pain and wt loss
- Infraction results in pain and bloody diarrhea

IBS (Irritable bowel syndrome)


Presentation - Relapsing abdominal pain with bloating, flatulence and change in bowel habits
- IMPROVES WITH DEFECATION
- Increased dietary fiber may help
Prevalence - Classic pt is middle aged women
Cause - Disturbed abdominal motility (no identifiable pathologic change)

GI Page 7.2
10.9 Colorectal Carcinoma

Colonic polyps
1. Differentiate hyperplastic and adenomatous polyps.
Hyperplastic polyp Adenomatous polyp
Most common colon polyp; usually in rectosigmoid Second most common colon polyp
section
Occur due to hyperplasia of glands (polyclonal) - Occur due to neoplastic proliferation of
serrated appearance on microscopy - both are glands (monoclonal)
identical
Benign with no malignant potential Benign but may progress to
adenocarcinoma via adenoma-
carcinoma sequence

2. What is adenoma-carcinoma sequence and what are the steps in it?


- Adenoma-carcinoma sequence describes molecular progression from colonic mucosa to
adenomatous polyp to carcinoma.
- Steps:
Mutation Presentation Gene function
APC (adenomatous increases risk of APC inactivates B-catenin (b-catenin
polyposis coli gene) - polyp formation increases Cyclin D transcription and
sporadic or germline progresses cell cycle)
K-ras leads to formation K-ras works in RAS pathway. One function
of polyp in increase Cyclin D transcription
P53 mutation and increased Progression of P53 transcription factor of p21; prolonged
COX expression adenoma to activation causes transcription of Bax and
carcinoma lead to apoptosis
- Asprin can be given to stop progression of colon polyp to carcinoma

GI Page 8.1
3. How are screening of polyps done?
- Colonoscopy - both hyperplastic and adenomatous polyp look the same; distinguished by
microscopy
- Fecal occult blood - some polyps bleed

4. What types of polyps have greatest risk of progression to cancer?


- Sessile growth
- Size >2 cm
- Villous histology

Familial adenomatous polyposis (FAP)


Presentation - Presence of 100 or 1000's of adenomatous colonic polyps
- Will progress to cancer by 40 year (remove colon prophylactically)
Mutation - Autosomal dominant
- APC mutation (chromosome 5)

1. Describe the following syndromes associated with FAP.


Gardner - FAP + fibromatosis + osteoma
syndrome - Fibromatosis - usually presents as desmoplasia in retroperitoneum and
destroy local tissue (non-neoplastic proliferation of fibroblasts)
- Osteoma - benign tumor of bone; usually presents in skull

Turcot FAP + CNS tumor (medulloblastoma and glial tumor) - you put turban on head
syndrome

Juvenile polyp, juvenile polyposis and Peutz-Jeghers syndrome


1. Differentiate between the above three.
Juvenile polyp Juvenile polyposis Peutz-Jeghers syndrome
(aka heriditary intestinal

GI Page 8.2
(aka heriditary intestinal
polyposis)
Sporadic, in children <5 years Sporadic or AD (mutation in Inherited - autosomal
SMAD - don’t need to know) dominant
Usually presents as solitary Multiple juvenile polyp in - Multiple hamartomatous
rectal hamartomatous polyp stomach and colon (juvenile polyp in GI tract
that prolapses and bleeds refers to type of polyp, not age - Hyperpigmentation (freckle
of pt) like spots) on lips, oral
mucosa and genial skin

Large number increase risk of Increased risk of colorectal,


progression to carcinoma breast and gynecologic
cancer

Colorectal cancer
1. What is colorectal cancer and what's its epidemiology?
- 3rd most common cause of cancer related death
- Peak incidence in 60-70 year olds (both men and women)

2. What are the two molecular pathways that cause CRC?


- Adenoma-carcinoma sequence (most common) Familial adenomatous polyposis (FAP) -
inherited mutation in APC
- Mutation of mismatch repair (MMR)enzymes - Lynch syndrome (inherited mutation in
see microsatellite instability MMR)

3. Describe microsatellite instability's role in CRC. What disorder is associated with microsatellite
instability?
- Microsatellites are repeating sequences of noncoding DNA; their number is maintained during
cell division (i.e. they're stable)
- Instability indicates defect in DNA mismatch repair enzymes
- Disorder associated:
○ Heriditary non-polyposis colorectal carcinoma (HNPCC) - aka Lynch syndrome
 CRC is de novo (not from adenomatous polyps) at early age and usually right sided.
Adenoma-carcinoma sequence HNPCC
80% of sporadic colon cancer
1st mutation - APC - increases chance of polyp Mutation is in DNA mismatch repair
formation enzyme
2nd mutation - K-ras - polyps formed
3rd mutation - COX overexpression and p53
mutation - polyps turn to carcinoma
Cancer progresses from adenomatous polyps Cancer is de-novo (no need to see
adenomatous polyps)
Common age 60-70 Seen early

GI Page 8.3
Common age 60-70 Seen early
Cancer is usually right sided

4. Describe the following features of colorectal cancer.


Location - Right sided - usually see raised lesion; presents as iron deficiency anemia due to
bleeding and vague pain. Older adult with Fe deficiency anemia has colorectal
cancer until proven otherwise
- Left sided -
• napkin ring lesion:
• decreased stool caliber,
• left lower quadrant pain,
• blood streaked stool
Staging - T - depth of invasion (limited to mucosa wont spread away due to lack of
lymphatics)
- N - spread to regional lymph nodes
- M - most common metastasis site is liver
Serum - CEA - used to assess assessing treatment response and detecting recurrence; not
marker useful for screening

GI Page 8.4
Glycogen storage disease

Von Gierke's (Van jerky) Pompe (pump) Cori (coral) Mcardle (muscle glider)
- Type 1 glycogen storage - Type 2 - Type 3 - Type 5
disease
- Glucose-6-phosphatase - Lysosomal alpha 1, 4, - Alpha 1, 6, glucosidase - Muscle glycogen
deficiency (last enzyme in glucosidase aka acid maltase deficiency (glycogen phosphorylase deficiency
glycogenolysis) deficiency debranching enzyme)
- Presentation: - Presentation: - Presentation: - Presentation:
• High glycogen in liver • Cardiomegaly • Stunted growth • Severe muscle cramp
• Enlarged kidney during exercise
• Myoglobinuria
• Increased glycogen in
muscle
• Hepatomegaly - Hepatomegaly • Hepatomegaly
- Severe hypoglycemia - Normal blood glucose - Hypoglycemia
- Lactic acidosis - Hypotonia - Normal lactic acid - Normal lactic acid

1. Von Gierke's disease (Van Jerky)

- Type 1 glycogen storage disease - #1 near Von Jerky


MOA - Glucose-6-phosphatase deficiency - glue with 6P cigar - last enzyme in glycogenolysis reaction
Presentation - High glycogen in liver (glider liver with up arrow)
- Severe hypoglycemia with fasting - liver can't pump out glucose in fasting
- Hepatomegaly (balloon liver)
- Big Kidney (balloon kidney) - glycogen is also stored in kidney
- Lactic acidosis (lake acid with up arrow)

GI Page 9.1
- Lactic acidosis (lake acid with up arrow)

2. Pompe disease (pump)

- Type 2 glycogen storage disease - pump with toto


MOA - Deficiency in lysosomal alpha 1,4 glucosidase (lysol bottle with wand (1), fork (4), glue and daisy bottle) aka acid
maltase (maltese with lemon)
Presentatio - Normal blood glucose - lysosomal breakdown pathway not major source to maintain blood glucose
n - Hepatomegaly
- Cardiomegaly
- Hypotonia - hippo with tonic

3. Cori's disease (coral)

- Type 3 glycogen storage disorder - 3 tree

GI Page 9.2
- Type 3 glycogen storage disorder - 3 tree
MOA - Deficiency in glycogen debranching enzyme alpha 1, 6 glucosidase (saw cutting branch, and afro, wand, saxaphone,
glue with daisy)
Presentatio - Normal blood lactic acid level (body can go gluconeogenesis and some glucose can be taken off glycogn) - lake acid
- n with no up arrow
- Hepatomegaly - glycogen accumulates in liver due to lack of debranching enzyme
- Stunted growth - because body can't mobilize energy source - C clamp
- Hypoglycemia - due to lack of debranching enzyme (hippo-glue bottle)

4. Mcardles disease

- Type 5 glycogen storage disease - hi 5 hand


MOA - Deficiency in muscle glycogen phosphorylase. Enzyme essential for first step in glycogen breakdown. (muscle glider
with phosphate p)
Presentatio - Increased glycogen in muscle (up arrow in glycogen muscle) - glycogen is trapped in muscle
n - Painful muscle cramps with exercise (clamps on muscle man) - due to insufficient glucose to muscle
- Myoglobinuria (mayo globe urinal) - dark, brown urine - without enough glucose, muscle breaks down and releases
myoglobin
- Normal lactic acid (lime with bell curve) - Disease localized only to skeletal muscle and body's overall metabolism
isn't severely affected

GI Page 9.3
Lysosomal storage disorder

1. Hunter and Hurler syndrome


Hunters syndrome Hurlers syndrome
- X-linked recessive
- Iduronate sulfatase deficiency - Alpha-L-iduronidase deficiency
- Heparan sulfate and dermatan sulfate - Heparan sulfate and dermatan sulfate
accumulates in lysosome accumulates in lysosome
Presentation: Presentation:
• No corneal clouding • Corneal clouding
• Aggressive behavior • Developmental delay
• Gargoylism
• Airway obstruction
• Hepatosplenomegaly (due to
accumulation of molecules)

a. Hunters syndrome

Genetics - X linked recessive (X on shirt)


MOA - Iduronate sulfatase deficiency - results in abnormal carbohydrate breakdown in
lysosome (Iditarod on sulfur match)
- As a result, Dermatan sulfate accumulates on lysosome (deer tan man)
- Heparan sulfate also accumulates (hippie run)
Presentatio - No corneal clouding (no corn cloud flag)
n - Aggressive behavior (hair pulling)

GI Page 10.1
b. Hurlers syndrome

- Mucopolysaccharidoses disease - a set of disease where body can't break down long
sugar and they accumulate in various tissue and cause disease
MOA - Alpha -L- Iduronidase defeciency - (alpha 1 afro on Iditarod)
- As a result, Dermatan sulfate and heparan sulfate accumulates on lysosome (deer
tan man and hippie run)
Presentati - Gargoylism - flat face, depressed nasal bridge, bulging forehead - gargoyl
on - Airway obstruction - due to craniofacial abnormalities and glycosaminoglycan
deposition in neck tissue
- Corneal clouding
- Developmental delay
- Hepatosplenomegaly

c. Tay Sachs disease (tie with saxophone)

GI Page 10.2
Genetics - Mutation in hexosaminidase A (hexagon tires with mini A) which breaks down GM2
ganglioside (GM2 car)
- Seen more in Ashkenazi jewish population - jew gangster
MOA - GM2 ganglioside accumulates (GM2 car with gangster on side)
Presentatio - Onion skin lysosomes (onion bulb)
n - Cherry red macula (due to accumulation of GM2 ganglioside in retina and
transparency of macula) - cherry on onion
- Progressive degeneration of CNS due to accumulation of GM2 ganglioside (neuron
hands of onion)
- Developmental delay due to GM2 ganglioside accumulation in CNS (developmental
delay bus)

d. Niemann-pick disease (pick in man's knee)

MOA - Deficiency of sphingomyelinase (sphinx on man's leg) leads to accumulation of


sphingomyelin in CNS, liver, spleen.
- More common in Ashkenazi jewish population - jew guy
Presentatio - Foam cells in histology - foam in mouth
n - Hepatosplenomegaly - liver and spleen baloon
- Neurodegeneration
- Cherry red macula - cherry on eyes

e. Metachromatic leukodystrophy

GI Page 10.3
MOA - Deficiency of Arylsulfatase A (arrow sulfur match A). Cerebroside sulfate (cerebro
guy shooting arrow) builds and affect growth of myelin
Presentati - Presentation based on myelin damage
on - Ataxia (taxi)
- Dementia (d man)

f. Gauchers disease (gaucho)

GI Page 10.4
Epidemiolo - Most common lysosomal storage disease
gy
MOA - Deficiency of beta glucocerebrosidase (beta fish, glue, cerebrow guy) leads to
accumulation of glucocerebroside in cell and organs
Presentatio - Gaucher cells - macrophage with glucocerebroside that look like crumpled tissue
n paper - crumpled paper
- Hepatosplenomegaly due to glucocerebroside accumulation.
• Anemia due to RBC destruction in spleen
• Leukopenia and thrombocytopenia
- Bone crisis (crying bone) - severe pain in joints and bone due to glucocerebroside
accumulation
- Erlenmeyer flask deformity - aseptic necrosis of femur that makes it look like
Erlenmeyer flask (necrosis crow with femur)

Fig: Gaucher cell (left), Erlenmeyer flask deformity (middle), normal femur (right)

GI Page 10.5
g. Krabbes disease (Crab)

MOA - Deficiency of galactocerebrosidase (galaxy sign, guy with cerebro) that causes
accumulation of galactocerebroside which damages myelin
Presentati - Globoid cells on histology - enlarged macrophage with multiple nuclei - globe
on - Developmental delay - due to myelin damage- DD bus
- Peripheral neuropathy - damaged nerves
- Optic atrophy - damage to optic nerve fibers - optic atrophy glasses

GI Page 10.6
h. Fabry's disease (febreeze bottle, fairy carrot, fairy wedding)

Inheritance X linked
MOA - Deficiency of alpha galactosidase A (afro galaxy with side A) leads to accumulation

GI Page 10.7
MOA - Deficiency of alpha galactosidase A (afro galaxy with side A) leads to accumulation
of ceramide trihexaside
Presentatio - Globoid cells on histology - enlarged macrophage with multiple nuclei - globe
n - Developmental delay - due to myelin damage- DD bus
- Peripheral neuropathy - damaged nerves
- Optic atrophy - damage to optic nerve fibers - optic atrophy glasses

GI Page 10.8
Chapter 11: Exocrine Pancreas,
Gallbladder, and Liver Pathology
11.1 Exocrine Pancreas

0. What is annular pancreas? (HY)


- Developmental malformation where pancreas forms ring around duodenum.
- Risk of duodenal obstruction

Acute pancreatitis
1. Describe acute pancreatitis, it's etiology, pathophysiology and presentation.
- Acute pancreatitis is reversible enzymatic digestion of pancreas with hemorrhagic damage
(pancreas has lots of blood supply).
- Etiology
○ EtOH and gallstones cause >75% of cases
○ EtOh causes contraction of sphincter of oddi which slows drainage of pancreatic
enzymes --> risking premature activation of enzymes in pancreas
○ Other
 Trauma (ex - automobile accident)
 Hypercalcemia (Ca is activator of enzymes)
 Hyperlipidemia (
 Cystic fibrosis (In CF, low bicarb and water drainage to pancreatic ducts; slow flow
and thick enzyme --> enzyme activation)
 Drugs, scorpion venom
 Mumps
 Rupture of duodenal ulcer
- Pathophysiology
○ Activation of trypsinogen is the first step that cause activation of other enzymes
- Presentation
○ Epigastric pain radiating to back (pancreas is secondary retroperitoneal organ)
○ Nausa, vomiting
○ Periumbilical and flank hemorrhage

Fig - liquifactive necrosis of pancreas with fat necrosis of surrounding fat in pancreatitis

2. How do you diagnose acute pancreatitis?


- Elevated lipase (3 times normal) - most commonly used in diagnosis
- Elevated amylase
- Hypocalcemia - fat necrosis of peripancreatic fat uses up Ca in blood - poor prognotic indicator

3. What is complication of acute pancreatitis?


- Shock - pancreas is very vascular so damage can lead to tons of hemorrhage
- Pancreatic pseudocyst -

Liver and pancreas Page 1.1


- Pancreatic pseudocyst -
○ Pt has abdominal mass with persistent increased in amylase after acute pancreatitis.
Pseudocyst rupture cause release of enzyme and digestion and hemorrhage of gut.
- Pancreatic abscess
○ Usually due to E.Coli. Pt presents with persistently increased amylase.
- DIC - enzymes get on blood, digest coagulation factors
- ARDS - enzymes get to blood, go to lungs and damage alveoli-capillary interface

Chronic pancreatitis
4. What causes chronic pancreatitis?
- Chronic pancreatitis is fibrosis of pancreatic parenchyma which occurs after multiple bouts of
acute pancreatitis
- Etiology
○ EtOH (most common in adults)
○ Cystic fibrosis (most common in kids)
○ Idiopathic
- These etiologies cause repeated attack to pancreas. Other etiology of acute pancreatitis (ex -
hypercalcemia, scorpion bite etc happen only once or so.

5. What are features of chronic pancreatitis?


- Epigastric pain radiating to back
- Pancreatic insufficiency (small fibrosed pancreas)
○ Leads to malabsorption with steatorrhea
○ Fat soluble vitamin deficiency
- Dystrophic calcification of pancreas
- 20 DM in late phase - due to damage of islet cells
- High risk of pancreatic carcinoma

6. Why are serum lipase and amylase not good marker for chronic pancreatitis?
- Because in chronic pancreatitis, pancreas don't produce much enzyme so enzymes aren't
elevated in blood.

Pancreatic adenocarcinoma
7. Describe pancreatic adenocarcinoma, it's risk factor and patient population.
- It's adenocarcinoma arising from pancreatic ducts
- Most commonly seen in elderly
- Major risk factor (HY) -
○ Smoking and chronic pancreatitis

8. What is presentation of pancreatic adenocarcinoma?


- If tumor in head of pancreas - obstructive jaundice with pale stool, palpable gallbladder (blocking
of common bile duct)
- If tumor in body or tail or pancreas - 20 DM
- Acute pancreatitis - due to obstruction of duct
- Migratory thrombophelebitis -
○ Swelling, erythema and tenderness of extremities
- Serum tumor marker is CA 19-9

If you see thin old pt with sudden onset diabetis, suspect pancreatic adenocarcinoma. Because DM is
usually disease of fat people with onset during middle age, and pancreatic adenocarcinoma is disease of
old people.

9. What is prognosis and treatment of pancreatic adenocarcinoma?


- Very poor prognosis (1 year survival <10%)

Liver and pancreas Page 1.2


- Very poor prognosis (1 year survival <10%)
- Treatment is whipple procedure - take out head & neck of pancreas, duodenum and gall bladder

Liver and pancreas Page 1.3


11.2 Gallbladder and Biliary Tract

1. What is biliary atresia? What is its presentation


- Failure to form or early destruction of extrahepatic biliary tree (atresia means
lack of lumen)
- Presentation-
○ Presents with neonatal jaundice (conjugated bilirubinemia) and
progresses to cirrhosis

Cholelithiasis
2. Describe causes of gall bladder stones.
Cholesterol - Very high cholesterol or
stones - low bile acids and phospholipids (bile acid and
phospholipids dissolve cholesterol) (some drugs lower bile
acids)
- Stasis
Pigment stones Pigment stones form due to breakdown of conjugated
- Black bilirubin or too much bilirubin.
stones - Black stones - non-bacterial, non-enzymatic
- Brown breakdown of conjugated bilirubin. One cause -
stones hemolysis
- Brown stones - bacterial enzymatic breakdown of
bilirubin (stasis can cause bacterial growth in
gallbladder)

3. Describe presentation of cholesterol stones.


- Woman:men = 3:1 Most common stone in western world
- Radiolucent
- 70% of cases are asymptomatic; treat only if symptomatic. 10% of normal popn
have it.

4. What are risk factors for cholesterol stones?


- Age
- Estrogen -
○ Estrogen increases activity of HMG CoA reductase which increases
cholesterol production.
○ Estrogen increases stasis of gall bladder
- Fibrates (Clofibrate) -
○ lipid lowering drug that increases HMG coA reductase.
○ Also decreases conversion of cholesterol to bile acids
- Native American ethnicity
- Chron disease -
○ Most common site for Chron is terminal ileum; that's the site of reuptake
of bile salts and bile acid
- Cirrhosis

5. What's presentation and risk factor for pigment stones (aka bilirubin stones)?
- Presentation
○ Radioopaque
- Risk factors

Liver and pancreas Page 2.1


- Risk factors
○ Extravascular hemolysis (increased bilirubin)
○ Biliary tract infection (bacteria deconjugates conjugated bilirubin)

6. What are some complications of gallstones? (each are described with questions
below)
- Biliary colic
○ Pt has waxing and waning RUQ pain (pain starts when gall bladder
contracts against stone; pain goes away when bladder stops contraction)
- Acute pancreatitis and obstructive jaundice
○ Due to blockage of biliary tree
- Acute and chronic cholecystitis
- Ascending cholangitis
- Gallstone ileus - stone obstructing the intestine lumen
- Gallbladder cancer

Complications of gallstones
7. Describe acute cholecystisis and its presentation?
- Acute cholecystis occurs due to pressure build up in gall bladder due to biliary
tree blockage by stone. Pressure causes squeezing of vessels in gall bladder
wall --> ischemia and bacterial growth
- Presentation
○ RUQ pain that radiates to scapula (HY)
○ Fever with elevated WBC count
○ Increased serum alkaline phosphatase (cell lining biliary tree and gall
bladder contain alk phosphatase)
○ Risk of rupture if untreated

8. Describe chronic cholecystisis.


- Chronic inflammation of gall bladder due to chemical irritation from long
standing cholelithiasis.
- Hallmark - formation of Rokitansky-Aschoff sinus. (HY)

Fig - M resembles gall bladder mucosa. The red circles on bottom


indicates mucosa inside muscular layer (Rokitansky-Aschoff sinus) and it's
not supposed to be there.

9. What is presentation and complication of chronic cholecystisis?


- Presentation
Vague RUQ pain, especially after eating

Liver and pancreas Page 2.2


○ Vague RUQ pain, especially after eating
- Complication
○ Porcelian gallbladder (due to dystrophic calcification of necrotic
gallbladder wall)

Fig - procelian gallbladder.

10. How do you treat chronic cholecystisis?


- Treatment is cholecystectomy
- Treat especially if procelain gallbladder seen (high risk for cancer)

11. Describe clinical features of ascending cholangitis.


- Its ascending bacterial infection of biliary tree often by enteric gram negative
bacteria
- Association
○ Choledocolithiasis (>80% of cases)
- Presentation
○ Charcod's triad (biliary colic (RQU pain), fever, jaundice)
○ Reynaud's pentard (Charcod's triad + hypotension + altered mental
status) - indicates sepsis as opposed to cholangitis only

12. Describe gallstone ileus.


- Due to cholecystitis and fistula formation between gall bladder and small bowel,
stone might enter small bowel and obstruct it.

13. Describe gallbladder carcinoma, it's risk factor, presentation and prognosis.
- It's adenocarcinoma of glandular epithelium of gallbladder wall
- Gallstones are major risk factors, esp with procelain gallbladder
- Presentation
○ Cholecystitis in old woman (cholecystitis is a disease of 40-50 year olds)
- Poor prognosis.

Fig - gallbladder adenocarcinoma

Liver and pancreas Page 2.3


Fig - gallbladder adenocarcinoma

Choledocolithiasis
14. Describe clinical features of choledocolithiasis.
- Stones within the biliary tree
- 90-95% of pt with choledocolithiasis have cholelithiasis. But only 15% of pt with
cholelithiasis have choledocolithiasis.
- Lab finding -
○ Contrast to cholelithiasis, we see sign of biliary tree blockage
 Increased alk phosphatase
 Increased bilirubin

Liver and pancreas Page 2.4


11.3 Liver: Jaundice

1. Explain the pathway of Bilirubin metabolism.

Note: conjugated bilirubin and direct bilirubin are same thing; non -conjugated and indirect are same.
UGT is Uridine glucuronyl transferase

2. What is significance of bile?


- Bile is the only way of cholesterol excretion
- Bile is the main way of copper and iron excretion.

3. What is presentation of jaundice?


- First sign in scleral icterus (yellow sclera). Later see yellow skin
- Occurs due to serum bilirubin >2.5mg/dl

4. Describe the following diseases of hyperbilirubinemia.


Disease Lab values Etiology Clinical features
Extravascular hemolysis High UCB Too much UCB overwhelm liver - Dark urine (increased urobilinogen)
(aka ineffective - High risk for pigmented bilirubin gallstones
hemolysis)
Physiologic jaundice of High UCB Newborn has transiently low UGT - UCB is fat soluble and can deposit in basal
newborn ganglia (kernicterus) leading to neuro defect
and death
- Treatment - phototherapy - makes UCB
water soluble and can be excreted by kidney
(HY)
Gilbert syndrome High UCB Mildly low UGT activity (autosomal - Jaundice during stress (ex - infection);

Liver and pancreas Page 3.1


Gilbert syndrome High UCB Mildly low UGT activity (autosomal - Jaundice during stress (ex - infection);
recessive) otherwise clinically silent
Crigler-Najjar syndrome High UCB Absent UGT - Kernicterus, usually fatal

Dubin-Johnson High CB Low bilirubin canalicular transport - Dark liver (HY) (otherwise clinically silent)
syndrome protein (autosomal recessive)
Rotor syndrome High CB Low bilirubin canalicular transport - Lacks liver discoloration; otherwise same as
protein (autosomal recessive) Dubin-Johnson
Biliary tract obstruction High CB Seen with gallstones, pancreatic - Dark urine and pale stool
(obstructive jaundice) High alk carcinoma, cholangiocarcinoma, - Pruritus due to increased plasma bile acids
phosphatas parasites, liver fluke - Hypercholesterolemia with Xanthoma (bile
e is major way of cholesterol excretion)
Low urine - Steatorrhea with malabsorption of fat
urobilin soluble vitamin
Viral hepatitis High CB and Inflammation disrupts inflammation - Dark urine (due to CB)
UCB and small bile ductules - Normal or low urine urobilinogen (CB is
leaking out to blood and not going to
duodenum)

Notes
- Unconjugated bilirubin can't be excreted by urine as it's fat soluble. It floats in blood until it finds exit.
- Conjugated bilirubin is water soluble, can leak out to blood and will go to urine and make it dark

Liver and pancreas Page 3.2


11.3 Liver: Hepatitis

1. What are true liver function tests (LFT)?


- True liver function test tests for synthetic ability of liver
○ PT (INR) - most sensitive LFT
○ Albumin
○ Bilirubin

2. What are other LFT?


- Aminotransferases
○ Aspartate aminotransferase (AST) - found in mitochondria. (also increases in muscle injury
and MI)
○ Alanine aminotransferase (ALT) - found in cytosol (better test for hepatocyte injury than AST)
- Alkaline phosphatase - most sensitive test for cholestasis (also present in bone)

3. Some notes on LFT-


- If aminotransferase>alk phos --> hepatocyte injury
- If alk phos > aminotransferase --> cholestatic injury
- If AST>ALT by >2:1 and if both are <300 (IU/L?), suspect EtOH hepatitis
- If ALT>AST, usually viral or NAFLD

4. What does ALT>1000 IU/ml indicate?


- It indicates severe acute hepatitis and has limited etiologies-
○ Shock liver (hepatic ischemia)
○ Drug induced liver injury
○ Acute viral injury

5. What does PT measure?


- Prothrombin time (PT) measures time to convert prothrombin to thrombin. Therefore, it measures
activity of liver derived factors 2, 7, 9 and 10. If pt's PT is low despite enough vit K intake, it's
synthetic dysfunction.

6. What does the following findings indicate?


Findings Indication
Murphy's sign (inspiratory arrest during palpation of RUQ) Cholecystisis
Charcot's triad (RUQ pain, fever, jaundice) Choleangitis
Pruritus Cholestasis

5. What are most common cause of ESLD?


• Hep C
• NASH
• EtOH

6. What are signs of ESLD?


• Increased estrogen (gynecomastia, testicular atrophy) - liver converts estrogen to androgens
• Ascites, jaundice
• Hepatic encephalopathy- asterexis, clonus
• Palmar erythema, spider angiomata

Liver and pancreas Page 4.1


Viral hepatitis
1. What are some causes of viral hepatitis?
- Hepatitis virus A-E (most hep C leads to chronic hepatitis; Hep A,E mainly acute only)
• Acute hepatitis can progress to chronic cases
• Other causes include CMV and EBV

2. Describe presentation of viral hepatitis.


- Presentation-
○ Jaundice (mixed CB and UCB) with dark urine (due to CB)
○ Fever, malaise and nausea
○ Elevated liver enzyme (ALT>AST)
○ Symptoms last < 6 months (if >6 months, it's called chronic hepatitis)

3. Describe histology and pathophysiology of acute viral hepatitis.


- Histology-
○ Viral damage occurs both in between hepatocytes and in portal tracts.
- Pathophysiology-
○ For Hep virus, it's not the virus that causes the damage; it's the CD8 cells that kill
hepatocytes.

4. What is a common complication and histology of chronic hepatitis?


- Complication - Cirrhosis
- Histology - Inflammation predominantly involves portal tract; not much damage to liver
parenchyma.

Fig - L is liver parenchyma, P is portal tract. Note inflammation is mainly in portal tract only.

5. Describe Hep virus A and E.


Hep A Hep E
Transmission - Feco-oral - Feco-oral
- Common in - Commonly acquired from uncooked seafood or contaminated
travelers water
Acute/chronic - Mainly acute - Mainly acute only
only
Active - See Hep E - Presence of IgM marks active infection; presence of IgG marks
infection prior infection that's resolved or immunization (immunization
marker for Hep A only)

Liver and pancreas Page 4.2


marker for Hep A only)
- Hep E in pregnant woman associated with fulminant hepatitis
(liver failure with massive liver necrosis) (HY)

6. How are Hep B and C virus transmitted?


- Parenterally (unprotected sex, childbirth, IV drug, blood transfusion)

7. Describe serologic information about Hep B virus. (HY)


- HBsAG (hep B surface antigen)
○ It is the key and first marker of infection. If it's present disease is present (either active or
chronic).
○ It's the first serologic marker to rise.

- HBeAG and HBV DNA


○ Envelope antigen indicates infectivity of virus.
○ If it's present, pt is highly infectious (can transmit virus)

- HBcAB (hep B core antibody)


○ IgM-
 Indicator of battle between body and hepatitis virus in acute and window stage.
 Major Ab used to defeat the virus
○ IgG-
 If present in absence of HbsAG, it indicates resolved disease
 If present together with HBsAG for >6 months, it indicates chronic hepatitis

- HBsAB (hep B surface antibody)


○ IgG -
 If it's present, then person has won the battle against virus (either it has resolved, or
patient is immunized against it)
 If it's present, then HBsAG will be absent.

8. Which Hep viruses have immunization?


- A, B

9. Describe features of Hep C.


- About 50% of acute cases progress to chronic case with cirrhosis
- Both B and C are leading cause of liver cancer

Liver and pancreas Page 4.3


10. Describe serology of Hep C.
- Presence of HCV-RNA confirms infection; persistance of HCV-RNA indicates chronic disease

11. Describe Hep D.


- It can't infect by itself and depends on HBV for infection.
- Coinfection - if HBV and HDV infect at same time
- Superinfection - if HBV infects pt with preexisting HDV (more severe than coinfection)

Liver and pancreas Page 4.4


11.3 Liver: Cirrhosis and Tumors

1. Describe cirrhosis.
- It's end stage liver damage characterized by bridging fibrosis (fibrosis between portal tracts) and
regenerative nodules of hepatocytes

Fig - note the regenerative nodules with fibrosis in between - characterstic of cirrhosis.
(normal liver surface is smooth)

2. What mediates fibrosis?


- Stellate cells (that lie below endothelial cells) secrete TGF-beta which mediates the fibrosis.

3. What is presentation of cirrhosis?


- Portal HTN
- Decreased detoxification
- Decreased protein synthesis

4. What are presentation of portal HTN?


○ Ascites
 if SAAG (serum ascites albumin gradient - difference between serum albumin and
ascites albumin) > 1.1 g/dl, 97% accurate for diagnosing ascites due to portal HTN.
 If SAAG <1.1, suspect infection, cancer or inflammation
○ Congestive splenomegaly/hypersplenism
 Spleen eats up RBC because blood is backed up a lot
○ Protosystemic shunts (varices in rectum, esophagus, caput medusa)
○ Hepatorenal syndrome
 Rapidly developing renal failure due to liver failure

5. What are presentation of decreased detoxification?


○ Increased ammonia leads to mental status, asterixis and coma (reversible)
○ Increased estrogen leads to gynecomastia, spider angiomata and palmar erythema (liver
removes estrogen from blood)
○ Jaundice - low metabolism of unconjugated bilirubin

6. What are presentation of decreased protein synthesis?


○ Hypoalbuminemia (leads to edema)
○ Coagulopathy (high PT - due to low function of vit k dependent epoxide reductase)

7. What is the progression of alcohol related liver disease?


- Fatty liver
Fat accumulation in hepatocytes (reversible)

Liver and pancreas Page 5.1


○ Fat accumulation in hepatocytes (reversible)

Fig - white blob of fat in hepatocytes in fatty liver disease.

- Alcoholic hepatitis
○ Direct chemical injury to hepatocytes mediated by acetaldehyde (HY)
○ Generally seen with binge drinking
○ Histology
 Balooning of hepatocytes
 Mallory bodies (characterstic) - damaged intermediate filaments in hepatocytes (HY)
 Inflammation

Fig - circles showing balooning of hepatocyes and mallory bodies characterstic of


alcoholic hepatitis
○ Presentation-
 Painful hepatomegaly with AST>ALT (AST is located in mitochondria and alcohol is
mitochondrial toxin) (HY)
 May result in death
- Cirrhosis
○ Present with signs and symptoms of regular cirrhosis

8. Describe non-alcohol fatty liver disease (NAFLD).


- Can progress from fatty changes to hepatitis to cirrhosis
- Diagnosis of exclusion; ALT>AST
- Associated with obesity

Hemochromatosis and Wilson's disease

Hemochromatosis Wilson's disease


Due to high Fe in body Due to high Cu in body

Liver and pancreas Page 5.2


High risk of cirrhosis and hepatocellular High risk of cirrhosis and hepatocellular carcinoma
carcinoma
Seen in old people Seen in young people
Mutation in gene that regulates enterocyte Mutation in ability to excrete Cu into bile and ability to
Fe metabolism load Cu in ceruloplasmin

9. Describe hemochromatosis.
- Excess iron deposition (hemosiderosis) leads to tissue damage (hemochromatosis)
- Damage due to generation of free radical by Fenten reaction (HY).

10. Describe the MOA of primary hemochromatosis.


- Almost all Fe from diet is absorbed and stored by enterocytes. They regulate how much Iron to
supply to body.
- There's no real way to get rid of Iron (bile and bleeding excretes little bit)
- In hemochromatotis, the regulatory mechanism of enterocytes is lost and all Fe is absorbed.
- Mutation in HFE gene (most commonly c282y) leading to disregulation of enterocytes leads to 1 0
hemochromatosis.

11. What are some secondary causes of hemochromatosis?


- Receiving blood transfusion

12. What is presentation of hemochromatotis?


- Seen mainly in late adulthood because it takes time for Fe to accumulate.
- Classic presentation is bronze skin, cirrhosis and 20 DM (aka bronze diabetes).
- Other presentations-
○ Increased ferretin, high serum iron
○ Low TIBC (total iron binding capacity) - whenever ferretin is high, TIBC will be low
○ Cardiac arrhythmia, gonadal dysfunction - due to Fe deposition

13. What is liver biopsy finding of hemochromatosis?


- Brown pigments in hepatocytes (looks similar to lipofuscin - normal sign of old cells)
- Prussian blue stain will turn Fe blue, but won't mark lipofuscin

Fig - brown pigments in hepatocytes (maybe hemochromatosis or lipofuscin)

Fig - prussian blue stain showing Fe deposition in hepatocytes

14. How do you treat hemochromatosis? What's its prognosis?

Liver and pancreas Page 5.3


14. How do you treat hemochromatosis? What's its prognosis?
- Blood donation
- Increased risk of hepatocellular carcinoma (due to progression to cirrhosis)

Wilson disease
15. What is wilson disease?
- Autosomal recessive defect (ATP7B gene) that damages hepatocyte Cu transport and decreased
ability to bind Cu to ceruloplasmin (bile main way to excrete Cu; ceruloplasmin Cu carrier in blood)
leads to increased Cu in body.
- Free radical production leads to damage.

16. What's presentation of Wilson's disese? Labs?


- Presents in childhood (contrast to hemochromatosis that presents in adulthood)
○ Cirrhosis
○ Neuro - behavior changes, dementia, chorea, parkinsonism
○ Kayser Fleisher rings in cornea - due to Cu deposition
○ Increased risk of hepatocellular carcinoma
- Treatment - chelating agent
- Labs -
○ High urine copper
○ Decreased serum ceruloplasmin
○ Increased Cu in liver biopsy

Primary biliary cirrhosis and Primary sclerosing cholangitis


Primary biliary cirrhosis Primary sclerosing cholangitis (PSC)
- Autoimmune granulamotous destruction of - Inflammation and fibrosis of intrahepatic and
intrahepatic bile ducts extrahepatic bile ducts
- Associated with other autoimmune disease - Histology shows periductal fibrosis with 'onion
(classic pt women in child bearing age) skinning'
- String and pearl appearance of bile ducts (due to
interspersed fibrosis (string) and inflammation
(pearl)
- Anti-mitochondrial antibody present. (HY) - p-ANCA positive and associated with UC (UC is
also p-ANCA positive) and cholangiocarcinoma.
- Pt present with obstructive jaundice; cirrhosis - Pt present with obstructive jaundice; cirrhosis late
late complication; can cause hepatocellular complication; can cause hepatocellular carcinoma
carcinoma

Fig - preiductal fibrosis with onion skinning seen in PSC

Liver and pancreas Page 5.4


Fig - string and pearl appearance of biliary tree in PSC

Reye syndrome
17. What is reye syndrome?
- Fulminant (severe and sudden onset) liver failure with encephalopathy in children with viral
syndrome that take asprin
- Related to mitochondrial damage of hepatocytes
- Kawasaki disease (vasculitis) affects mainly coronary artery and presents like viral illness in kids.
But asprin is good to use in that case (HY)

18. What's presentation of reye syndrome?


- Hypoglycemia
- Elevated liver enzyme
- Nausea, vomiting
- May progress to coma and death

Hepatic cancer
19. Describe hepatic adenoma.
- Benign tumor of hepatocytes
- Associated with oral contraceptive use; tumor goes down with cessation of drug (HY)
- Risk of rupture because tumor is right below liver capsule - lead to hemorrhage, especially during
pregnancy

20. What are risk factors for hepatocellular carcinoma?


- Chronic hepatitis (usually caused by hep B and C)
- Anything that leads to cirrhosis - hemochromatosis, wilson's, NAFLD, EtOH FLD, alpha-1 antitrypsin
deficiency
- Aflatoxins derived from Aspergillus (HY) - countries that store grains can have Aspergillus grow on
them and they have high incidence of hepatocellular carcinoma
○ Aflatoxin causes mutation in P53 mutation

21. What's relationship between hepatocellular carcinoma and Budd-chiari syndrome?


- Hepatocellular carcinoma loves to invade hepatic vein and cause Budd-chiari
- Presents with painful hepatomegaly and ascites (SAAG <1.1 g/dl)

22. What is prognosis of hepatocellular carcinoma?


- Poor; tumor are detected late as symptoms are masked by cirrhosis

23. What's a serum marker for hepatocellular carcinoma?


- Alpha-fetoprotein

Liver and pancreas Page 5.5


24. Describe cancer metastasis to liver.
- More common than primary liver cancer
- Common source: colon, pancreas, lung, breast
- Presentation:
○ multinodular liver
○ Can feel nodules in free edge of liver in physical exam

Fig - multinodular liver is classic presentation of liver metastasis and can be felt on physical exam

Liver and pancreas Page 5.6


Chapter 12: Kidney and Urinary Tract
Pathology
12.1 Congenital

Horseshoe kidney (no. 1 congenital renal anomaly)


1. What is a finding in horseshoe kidney?
- Kidney abnormally located in lower abdomen as it gets caught in IMA root during its ascent from
pelvis to abdomen.

Fig: horseshoe kidney getting stuck during ascent by IMA

Renal agenesis (failure of kidney to form)


1. Differentiate presentation of unilateral and bilateral renal agenesis.
Unilateral renal agenesis Bilateral renal agenesis (no kidney present) -
incompatible with life
- Hypertrophy of existing kidney; risk of - Oligohydramnios (low amniotic fluid; amniotic fluid
renal failure later in life due to is urine) - presentation called Potter syndrome
hyperfiltration • Lung hypoplasia (lung develops by stretching
as baby breathes amniotic fluid in and out
• Flat face, low ears, extremity defects (baby
not floating, pressed against mother)

Dysplastic kidney
1. What is presentation of dysplastic kidney? Is it inherited?
- Mostly unilateral and non-inherited malformation of renal parenchyma with cysts and abnormal
tissue (ex-cartilage).
- Congenital but non-inherited
- If bilateral, must be distinguished from polycystic kidney disease

Fig: dysplastic kidney. Circle on left is cyst; circle on right shows growing cartilage

Polycystic kidney disease (PKD)


1. What is presentation of polycystic kidney disease?

Renal Page 1.1


1. What is presentation of polycystic kidney disease?
- Bilateral enlargement of kidney with cysts in renal cortex and medulla
- Inherited

Fig: polycystic kidney disease (note it's always bilateral)

2. Differentiate between autosomal dominant vs autosomal recessive type of polycystic kidney


disease.
Autosomal recessive (aka juvenile Autosomal Dominant (ADult)
form)
- Presents in infants as worsening renal - Presents in young adults with HTN (due to increased
failure and hypertension renin), hematuria and worsening renal failure
- Newborns may have potter sequence
- Associated with (HY): - Associated with:
• congenital hepatic fibrosis (see • berry aneurysm in brain - family history of renal
signs of portal HTN) and disease and brain related death
• hepatic cysts • hepatic cysts, and
• mitral valve prolapse
- Think as cyst in kidney, liver and brain.
- Mutation in APKD1 (adult polycystic kidney disease 1)
or APKD2; cysts develop over time

Medullary cystic kidney disease


1. What is medullary cystic kidney disease?
- Autosomal dominant cyst formation in medullary collecting ducts (PKD has both in cortex and
medulla)
- Parenchymal fibrosis leads to shrunken kidneys (PKD has big disease) and worsening renal failure

Renal Page 1.2


12.2 Acute Renal Failure

a) Classify the types of acute renal failure (azotemia) and its presentation.
- Pre-renal azotemia
- Intra-renal azotemia
○ Acute tubular necrosis (most common cause of ARF)
○ Acute interstitial nephritis
- Post-renal azotemia
- Presentation
○ Azotemia - hallmark (increase BUN in blood; uremia is symptoms due to increased BUN)
○ Oliguria (if we make <500ml urine/day, we can't excrete solid waste)

b) What are come causes of increased serum BUN?


- Decreased kidney function
- Use of steroids
- Upper GI bleed (blood has proteins and their metabolism can lead to nitrogenous waste build up)
- Hypercatabolic state (sepsis, critical illness)
- Dehydration, volume depletion.

Pre-renal azotemia (ARF)


b) Describe pre-renal azotemia and lab findings.
- Azotemia caused due to decreased renal perfusion.
- Causes - hypotension, decreased effective circulating volume, renal ischemia
- Lab findings -
○ FENa <1% (amount of excreted sodium in the urine)
○ Bun/Cr >15 (high)
○ High urine osmolality (>500) and low urine sodium

c) What is normal Bun/Cr ratio and how does it change during azotemia?
- Normally, Cr is excreted a little in PCT, and Bun is reabsorbed in PCT. Therefore, BUN is higher in
blood than Cr which makes normal BUN:Cr = 15
- In pre-renal azotemia, RAAS is activated due to decreased renal perfusion. More water is
reabsorbed, and more BUN is reabsorved with it. Therefore, BUN/Cr>15.
- In intra renal and post renal azotemia, Cr isn't excreted well. Build up of creatinine makes
Bun:Cr<15. (is this reasoning right)

d) What does FENa tell us?


- FENa = (urine Na/serum Na) / (urine Cr/serum Cr). It tells if kidney is dead or holding onto every
bit of salt it can. Lower it is, the better. If <1%, indicated pre-renal ARF.

Post-renal azotemia
e) Describe post-renal azotemia.
- Due to obstruction of urinary tract downstream, GFR decreases and results in azotemia, oliguria.
- Presentation
○ Early stage - FENa<1%, and BUN:Cr>15 (because increased backflow pushes BUN back to
blood.
○ Late stage - FENa>2%, BUN:Cr<15, and urine Osm<500 due to tubular damage and kidney
not working well.

Intra-renal azotemia (ARF)

Renal Page 2.1


Intra-renal azotemia (ARF)
a) Describe causes of intra-renal azotemia.
- Acute tubular necrosis
- Acute interstitial nephritis

b) Describe features of acute tubular necrosis (most common cause of ARF)?


- Necrotic tubular cells clog tubules, decrease GFR
- Hallmark - brown granular casts seen in urine
- Don't see inflammatory cells. If you see it, suspect acute interstitial nephritis

Fig- ATN - T are tubules that are clogged up by dead cells. G is glomerulus.

c) What are lab values for acute tubular necrosis?


- Bun:Cr <15 because BUN can't be reabsorbed
- FENa >2% because kidney not working well
- Urine osmolality <500 because kidney can't concentrate urine
- Azotemia, oliguria
- Hyperkalemia with metabolic acidosis (increased anion gap)

i) What are etiologies of acute tubular necrosis?


- Ischemic etiology - often preceded by prerenal azotemia. PCT and thick ascending limb are
particularly susceptible to ischemia
- Nephrotoxic etiology - PCT particularly susceptible
○ Heavy metal (lead
○ Myoglobinuria (crush injury
○ Antifreeze (ethylene glycol) - pt have oxalate crystal in their urine (HY)
○ Radiocontrast dye (iodine)
○ Aminoglycosides
○ Urate (chemotherapy drug if kills lots of tumor cells at once, damage of nucleic acids make
uric acid which can cause ATN (HY))

j) What is prognosis of ATN?


- Tubular cells are stable (can regenerate if needed). So support by dialysis because electrolyte
imbalance can kill. Otherwise, you can expect full recovery.

k) Describe acute interstitial nephritis. What are some drugs that can cause it?
- An intra-renal cause of ARF.
- Hallmark is presence of inflammatory cells in kidney interstitium.

Renal Page 2.2


- Hallmark is presence of inflammatory cells in kidney interstitium.
- Causes -
○ most common is infection or allergy
○ Others- toxin, disease
○ Drugs - commonly penicillin, diuretics, NSAIDs
- Eosinophils and monocytes are seen in drug induced AIN and polymorphonuclear nucleocytes are
seen in infectious AIN.

l) What is presentation of acute interstitial necrosis?


- Oliguria, fever, and rash days-weeks after infection/drug start
- Resolves with cessation of drug/infection
- May progress to renal papillary necrosis

m) What is renal papillary necrosis?


- Necrosis of renal papillae
- Presents with gross hematuira and flank pain
- Causes -
○ Analgesic abuse
○ DM
○ Sicke cell
○ Severe acute pyelonephritis

4. Describe autosomal dominant posycystic kidney disease.


• Third most common cause of renal failure
• Mutation in PDK1 is most common cause
• Presentation-
○ Hematuria, HTN, circle of Wills berry aneurysm
○ Azotemia
○ Dull loin pain (kidneys are super huge and can be easily palpated)

Renal Page 2.3


12.3 Nephrotic Syndrome

Nephrotic syndrome
1. What are presentation of nephrotic syndrome?
 Hallmark is >3.5g protein/day in urine
 Hypoalbuminurea
 Hypogammaglobulinemia
 Hypercoagulable state - loss of antithrombin 3
 Hyperlipidemia and hypercholesteremia - liver dumps fat to blood to make it thick (compensate
for loss of disease)

2. Describe minimal change disease.


• So common in kids that kids with nephrotic syndrome are assumed to have it.
• Cause - mostly idiopathic or may associate with hodkin's lymphoma (HY) - due to massive
production of cytokines.
• Sudden onset of symptoms, normal creatinine, normal renal function, normal BP. Not much
symptom other than nephrotic syndrome (that's why called minimal change).
• Change is so minimum that mainly albumin is lost, no immunoglobulin loss.
• Diagnosis - diagnosis of exclusion
• Histology -
○ glomerulus normal in light microscope, immunofluorescence.
○ EM shows microvillus transformation and effacement of foot process (a nonspecific finding
seen in all nephrotic diseases)
• Treatment- excellent response to steroids because disease caused by cytokines from active T cells.
Steroids reduce cytokine production
• Prognosis - often recur

Fig - effacement of foot process. Bottom part is capillary; black blobs are RBC.

3. Describe presentation of FSGS.


• Most common cause of nephrotic syndome in hispanics and blacks.
• Risk factors - most common in people with AIDS, heroin and sickle cell disease (HY). However,
most disease are idiopathic.
• Diagnosis - No serology, do biopsy. See focally segmental glomerulus.
• Treatment - not great but treat with steroids, immunosuppresants, control BP

4. Describe biopsy finding in FSGS


• Glomerulus is sclerosed (hardened) by hyalinosis (proteins and lipid deposition) in focal (only
some glomerulus affected - <50%) and segmental (only a part of glomerulus is affected) pattern.

Renal Page 3.1


Fig - Only bottom right glomerulus is affected segmentally.
• Sclerosis seen in H&E and hard to see in PAS or silver stain

5. Describe membranous nephropathy.


• Most common primary nephrotic syndrome. Often seen in old people
• Presentation:
○ Very severe nephrotic syndrome
• Treatment - very difficult to treat. 20-30% progress to end stage renal disease
• Secondary causes
○ Hep B>C (HY)
○ Lupus
○ Lung and colon cancer
○ Heavy metal
• Diagnosis - no serology; need biopsy

5.5. Describe relationship between lupus and membranous nephropathy (HY).


 Most common cause of death in patients with lupus is renal failure due to diffuse proliferative
glomerulonephritis (a nephritic syndrome). However, if lupus pt develop nephrotic syndrome,
they'll get membranous nephropathy.

6. What are histologic findings of membranous nephropathy?


• Capillary wall (basement membrane) thickens due to immune deposition; hence the name
membranous - see on H&E
• Any time there's membranous in name, there's immune deposition and thick glomerulus
membrane.

Fig - capillary wall thickening in MN.


• Small and frequent subepithelial deposition (IgG or C3 depositis)
○ Deposits give spike and hole appearance in PAS or silver stain

Renal Page 3.2


Fig - notice the small spikes and holes on PAS or silver stain. Holes are immune deposits as
they don't bind to silver. Spikes are new basement membrane laid on top of holes. They bind
to silver and stain dark.

○ Small and frequent deposits seen in EM. Immune deposition is subepithelial (below
epithelium - podocytes also called epithelium)

FIg - the dark deposits are immune deposits seen in MN.

○ Immune deposit is granular so immunofluorescence is granular

Fig - granular immunofluorescence seen in MN due to C3 and IgG immune deposite

7. Describe histology of Membranoproliferative glomerulonephritis (cause nephrotic or nephritic or


both).
• Membranous implies thick capillary membrane and immune deposition
• Proliferative means mesangial cell in glomerulus proliferate
• Thick capillary wall makes tram track appearance on H&E. Proliferation of mesangium cuts the
immune deposit in half creating two lines (tram tracks)

Renal Page 3.3


Fig - tram track appearance of membranoproliferative glomerulonephritis

7.5.What are two types of membranoproliferative glomerulonephritis and what are they associated
with?
• Divided to two types based on immune deposit type-
○ Subendothelial (type 1) - associated with hepatitis B, hepatitis C. Type 1 has more tram
tracks association.
○ Within basement membrane (type 2) - pt have C3 nephritic factor (autoantibody) (HY). This
antibody binds and stabilizes c3 convertase.
• Compliment system is hyperactive which causes the disease.
• Membranoproliferative disease can cause nephritic, nephrotic or both

8. Describe how diabetes leads to nephrotic disease.


• High blood glucose leads to non-enzymatic glycosilation of vascular baement membrane which
makes it more leaky leading to nephrotic syndrome
• Protein also leaks to vessel wall and leads to hyaline arteriolosclerosis).
• Non enzymatic glycosilation means that blood sugar is so high that they stick to basement
membrane without use of enzyme.

9. What's histology finding of diabetic nephrotic syndrome. What's its treatment


• Characterized by sclerosis of mesangium aka Kimmelstein-Wilson nodules - pathogmnenomic.
• Efferent arteriole is affected more than afferent. This causes high filtration pressure.
• Treat by ACE-i because it relieves pressure in efferent arteriole.

Renal Page 3.4


Fig - Kimmelstein-Wilson nodule

10. Describe how amyloidosis leads to nephrotic disease (most common organ affected in systemic
amyloidosis is kidney).
• amyloid deposits in mesangium leading to nephrotic syndrome
• characterized by apple-green birefringence in polarized light
• mesangial nodules in diabetes and amyloidosis are immunofuorescence negative. Diabetic
nodules are PAS positive, but amyloid nodules are PAS negative, why?

Renal Page 3.5


12.4 Nephritic Syndrome

1. What are features of nephritic syndrome?


• Hallmark is glomerular inflammation and bleeding
• RBC casts and dysmorphic RBC in urine hallmark of glomerular bleeding
○ Tram hosefall proteins and RBC make RBC cast
○ Dysmorphic RBC is caused by
• Can present with some proteinurea (<3.5g/day)
• Oligurea and azotemia
• Salt retention with periorbital edema

2. What are biopsy findings of nephritic syndrome?


• Hypercellular and inflamed glomeruli
• Immune deposits activate complement. C5a attracts neutrophil which cause damage

2.5. What are some causes of nephritic syndrome?


S.N. Pathology Description
1 Post Strep GN Occur in kids after group A B-hemolytic strep infection (if the
pathogen has M protein virulence factor)
2 IgA Seen in kids after mucosal infection
Most common nephropathy worldwide
3 Alport syndrome Inherited mutation in type IV collagen
4 Goodpasture Can lead to RPGN
5 Diffuse proliferative GN, Can lead to RPGN
Post strep in adults
6 Wegner Granulomatosis Can lead to RPGN
7 Microscopic polyangitis Can lead to RPGN
8 Chug Strauss syndrome Can lead to RPGN

3. Describe etiology of post- streptococcus glomerulonephrits (PSGN).


• Occurs after group A B-hemolytic strep (specially nephritogenic strains) infection of skin (impetigo)
or pharynx.
• The nephritogenic strain carries M protein virulence factor that increases chance of nephrotic
syndrome
• Impetigo - highly contagious skin infection of kids

4. What is presentation of PSGN.


• Hematuria (cola colored urine
• Oligurea
• Hypertension
• Periorbital edema
• Usually seen in kids but may occur in adult

5. What are biopsy findings of PSGN.


• Hallmark - subepithelial humps on EM
• Granular immunofluorescence- humps are made from immune deposition
• Hypercellular inflamed glomeruli

Renal Page 4.1


Fig - subepithelial hump seen in PSGN

6. What is prognosis of PSGN.


• Supportive. The reason is because the immune deposits start subendothelially and move to
epithelial site where they make a hump. Eventually, they just fall off. So disease heals by itself.
• Some adults develop RPGN

Rapidly progressive glomerulonephritis (RPGN)


7. What is RPGN?
• Rapidly progressive glomerulonephritis is a nephritic syndrome that progress to renal failure in
weeks to months.
• Hallmark - biopsy shows cresents in bowman space. Crescent made up of fibrin and macrophage
(HY)

Fig - cellular crescent seen in RPGN. Note crescent made of fibrin and macrophage (inflammatory
stuff - not collagen)

8. What are etiologies of RPGN?


• Etiologies can be distinguished based on immunofluorescence pattern
IF Pattern Disease Comment
s
Linear (anti-basement Goodpasture syndrome
membrane Ab)
Granular (immune complex PSGN (most common)
deposition) Diffuse proliferative GN
Negative IF (pauci-immune) Wegner Granulomatosis (C-ANCA positive)
Microscopic Polyangitis (P-ANCA)
Chrug-Strauss Syndrome (P-ANCA, granuloma,
eosinophila, asthma)

Renal Page 4.2


eosinophila, asthma)

9. Describe Goodpasture syndrome


• Antibodies are made against collagen in glomerulur and alveolar basement membrane
• Pt presents with hematuria and hemoptysis
• Classically seen in young adult males

10. Describe diffuse proliferative glomerulonephritis (nephritic syndrome).


• Diffuse Ag-Ab deposition in sub-endothelial location
• Most common type of renal disease in SLE (lupus) (VHY) - most common cause of death in lupus
patients is renal failure

11. How do you distinguish Wegner granulomatosis vs Goodpasture syndrome?


• Both of them may have hemoptysis and hematuria with RPGN. However, if you see sinus
problems, then suspect Wegner. Because in Wegner's, nasopharynx is affected.
• Distinguish by IF as well

13. Describe IgA nephropathy (nephritic)


• Most common nephropathy worldwide
• IgA deposition seen in mesangium - gives granular IF

Fig - mesangial immune deposition seen in IgA nephropathy

14. What is presentatin of IgA nephropathy


• Most commonly seen in kids after mucosal infection (recall that IgA is dumped in mucosal layer)
• When infection goes away, hematuria decreases as well. New infection leads to new episode of
hematuria. This can slowly lead to renal failure

15. Describe Alport's syndrome


• Inherited defect of type IV collagen (recall type IV collagen is found in basement membrane)
• GBM becomes thin and splits
• Presents with isolated hematuria, sensory hearing loss and ocular disturbances

Renal Page 4.3


12.5 Urinary Tract Infection (UTI)

1. What are common features of UTI?


• UTI are infection of urethra (urethritis), bladder (cystisis) or kidney (pyelonephritis). Higher up the
infection, more likely it is systemic
• Most UTI are ascending infections
• Risk factors are - being female, urinary stasis, sexual intercourse, catheders

2. What is cystisis?
• Inflammation of bladder
• Presentation-
○ Dysuria (pain with urination)
○ Urinary frequency and urgency
○ Suprapubic pain
○ Lack of systemic signs

3. What are lab findings in cystisis?


• Urinalysis : cloudy urine with >10 WBCs/hpf (high power field)
• Dipstick: positive leukocyte esterase (due to pyuria - pus in urine; wbc make this)
• positive nitrites (bacterias convert nitrates to nitrites
• Gold standard is culture. It yields >100,000 colony forming unites

4. What are etiology of cystisis?


• E. Coli - 80% of cases
• Staphylococcus saprophyticus - increased incidence in young, sexually active women
• Klebsilla pneumoniae
• Proteus mirabilis - gives alkaline urine with ammonia scent
• Enterococcus faecalis

5. What does sterile pyuria suggest?


• Sterile pyuria is pus in urine (>10 WBC/hpf and positive leukocyte esterase) but negative bacteria
culture of urine
• It suggests urethritis with due to Chlamydia trachomatis or Neisseria gonorrhoeae

6. Describe presentation of pyelonephritis.


• Mostly caused due to vesicoureteral reflux (reflux of urine)
• Presentation-
○ Fever, flank pain (you won't see fever in cystisis because there's no systemic signs there) -
flank pain is due to sensitization of nerves of renal capsule
○ WBC casts, leukocytosis - WBC coming up through tubules creates a cast
○ Symptoms of cystisis - dysuria, urinary frequency, urgency, suprapubic pain

7. What are common pathogens for pyelonephritis?


• E coli (90% of times)
• Klebsiella species
• Enterococcus faecalis

8. Describe chronic pyelonephritis.


• It is interstitial fibrosis and atrophy of tubules due to multiple bouts of acute pyelonephritis
• Seen due to vesiculoureteral reflux (in chidren) or obstruction - malformation of ureter entering

Renal Page 5.1


• Seen due to vesiculoureteral reflux (in chidren) or obstruction - malformation of ureter entering
bladder increases can increase chances of vesiculouretal refux

9. What are clinical features of chronic pyelonephritis?


• See cortical scarring with blunted calyces
• Scarring of upper and lower pole is characterstic of vesiculoureteral reflux
• Histology
○ See "thyoidization of kidney" - atrophic tubules have proteinaceous mateiral resembling
thyroid follicles.
○ Waxy casts may be present in urine

Fig - thyroidization of kidney seen in chronic pyelonephritis

10. pelvic inflammatory disease (PID) - infection of the upper part of the female reproductive system
namely the uterus, fallopian tubes, or the ovaries.(pyelonephritis version of cystitis)
□ Untreated PID can lead to fibrosis that can lead to infertility and ectopic pregnancy

Renal Page 5.2


12.6 Nephrolithiasis

1. What are risk factors for kidney lithiasis?


• High concentration of solute in urine/ low urine volume

2. What is presentation of lithiasis?


• Colicky pain(pain that starts and stops suddenly; occurs due to muscular contraction of hollow
tube)
• Hematuria and unilateral flank tenderness
• Stone usually passed in hours; if not, may need surgery

3. Explain different type of kidney stones.


Compositio Frequency Causes Treatment Comments
n
Calcium Most common Most common- idiopathic HCTZ (calcium sparring
oxalate type hypercalcuria (exclude diuretic)
and/ (usually see in hypercalcemia as a cause for
Calcium adults) hypercalcuria)
Phosphate Also seen in Chron's
Can be seen in ethylene glycol
ingestion
Ammonium Second most Most common - alkalization of Surgically remove stone Staghorn
Magnesium common urine by urease positive stone - nidus
Phosphate type pathogen (ex- proteus for infection
(AMP) vulgaris, klebsiella);

Aka struvite
stone
Uric acid Third most Most common in pt with Hydration and alkalization
common (5%) gout, hyperuricemia (lukemia of urine (KHCO3);
Radiolucent or myeloproliferative allopurinol for gout-
(others are disorders) decreases uric acid
radiopaque) Risk factors - acidic PH, hot,
arid climite, low urine volume
Cysteine Rare; most Associated with cystinuria Hydration and May form
commonly (genetic disorder where alkalinization of urine staghorn
seen in there's low reabsorption of stone
children cysteine from tubules)

Notes:
• In Chron's, damage to small intestine causes increased absorption of oxalate. This results in high
chance of forming calcium oxalate in urine

Renal Page 6.1


Fig - staghorn AMP stone. If you see staghorn stone in adult, think uric acid stone; if you see
in kid, think cysteine stone.

Renal Page 6.2


12.7 Chronic Renal Failure

1. What are causes of ESRD?


• Glomerular, tubular, inflammatory or vascular insults (anything can lead to it)
• Most common causes are diabetes, HTN, and glomerular disease

2. What are some presentations of ESRD?


• Uremia (symptom of azotemia)
• HTN - due to salt and water retention
• Hyperkalemia (kidney can't exchange sodium with potassium) leading to anion gap metabolic
acidosis
• Hypocalcemia -
○ Vit D is activated by 1 alpha hydroxylase in kidney in PCT
○ Kidney can't excrete phosphate and hyperphosphatemia leads to decrease in free calcium in
blood
• Anemia (renal peritubular interstitial cells makes erythropoietin) (HY).
• Renal osteodystrophy - damage to bone due to kidney problem
○ Osteitis fibrosa cystica - hypocalcemia leads to high PTH which decalcifies the bones;
eventually leads to fibrosis and cyst formation
○ Osteomalacia - when osteoid isn't mineralized (due to low calcium in blood)
○ Osteoporosis

2.5. What is presentation of uremia?


- Mental status change, seizure, encephalopathy
- Pericarditis
- Metallic taste and breath (due to urea), uremic frost on skin (skin deposition of urea) - pt have
nausea and don't eat much and causes anorexia
- Classic signs - pericarditis, uremic fretor, uremic frost

2.75. What are classic indication of dialysis?


- A - acidosis (anion gap acidosis due to hyperkalemia)
- E - electrolyte imbalance (hyperkalemia)
- I - ingestion
- O - overload (volume) - due to salt and water retention
- U - uremia

3. What is prognosis of ESRD?


- Patients need to put on dialysis or transplant

4. What are some changes that happen when patients are put on dialysis?
- ESRD kidneys are small and shrunken (small kidney = chronic kidney disease); putting on dialysis
leads to cyst formation
- Big and cystic kidney = polycystic kidney disease. Small and cystic kidney = ESRD kidney on dialysis
- Pt have high risk for renal cell carcinoma in the shrunken kidney(HY)
- Patient get local amyloidosis in the joints.

Renal Page 7.1


Questions

Is acute kidney injury, acute renal failure and uremia the same thing?

Renal Page 8.1


12.8 Renal Neoplasia

Angiomyolipoma
1. What's angiomyolipoma? What's its associated with?
- Hamartoma made up of blood vessels, smooth muscle and adipose tissue (clear from name)
- It's associated with tuberous sclerosis (HY) (syndrome with benign tumor primarily in brain, eyes,
heart, kidney, skin and lungs. Common presentation: seizures, developmental delay, intellectual
disability and autism)

Wilm's tumor (nephroblastoma) (most common malignant renal tumor in children; average age 3
years)
1. What's histologic hallmark of Wilm's tumor?
- Blastema (primitive kidney mesenchyme cells that make glomerulus, tubules and stromal cells)

2. What's presentation of Wilm's tumor?


- Average age of presentation - 3 years

Renal Page 9.1


- Average age of presentation - 3 years
- Unilateral flank pain, humaturia and HTN (due to renin production)

3. What are etiologies of Wilm's tumor?


- 90% of cases are sporadic. Syndromic tumor are associated with 3 following syndromes:
WAGR syndrome (HY) Denys-Drash Beckwith-Wiedemann syndrome
syndrome
W = Wilm's tumor - Wilm's tumor - Wilm's tumor
A = Aniridia (lack of iris) - Progressive - Neonatal hypoglycemia
G = genital abnormalities glomerular disase - Muscular hemihypertrophy (big muscles
R = Retardation (developmental - Male on one side of body)
and motor) pseudohermaphro - Organomegaly (specially tongue)
ditism
- Deletion of WT1 tumor - Mutation of WT1 - Mutation of WT2 gene cluster
suppressor gene (located in gene (imprinted genes at 11p15.5),
11p13) particularly IGF-2

Renal cell carcinoma


1. What are presentations of renal cell carcinoma? What are some paraneoplastic syndromes?
- Classic triad is hematuria, palpable mass and flank pain (see hematuria most often)
- Paraneoplastic syndromes (HY):
○ Erythropoietin production (reactive polycythemia vera)
○ Renin production (HTN)
○ PTHrP (hypercalcemia)
○ ACTH (cushings)
- Left sided varicocele (HY) - left spermatic vein drains to left renal vein; right spermatic vein drains
directly to IVC (sometimes the tumor can block renal veins)

2. What is pathogenesis of renal cell carcinoma (HY)?


- Mutation of VHL (3p) tumor suppressor gene
- It leads to increased IGF-1 (insulin like growth factor)(it promotes growth) and increased HIF
transcription factor (increases VEGF and PDGF)

3. What is staging of renal cell carcinoma?


- T - based on size and involvement of renal vein (occurs commonly and increases risk of
hematogenous spread to lung and bone)
- N - spread to retroperitoneal lymph nodes)

4. Differentiate sporadic and hereditary versions of renal cell carcinoma.


Sporadic tumor Heriditary tumor
- Classically seen in males (avg age 60) as - Classically seen in young adults and are often
single tumor in upper pole of kidney bilateral
- Major risk factor is smoking - Ex- Von Hippel-Lindau disease
• Autosomal dominant disorder

Renal Page 9.2


• Autosomal dominant disorder
• Associated with inactivation of VHL gene
(increased risk of hemangioblastoma of
cerebellum, pheo, and renal cell carcinoma)

5. What are gross and histologic features of renal cell carcinoma?


- Clear cell type is most common type of renal cell carcinoma (have clear cells)
- Gross exam reveals yellow mass)

Fig: gross feature (left) and microscopic feature (right) of renal cell carcinoma

Renal Page 9.3


12.9 Lower Urinary Tract Carcinoma

1. What constitutes the lower urinary tract?


- Renal pelvis, Ureter, Bladder, Urethra

2. What are 3 common malignant tumor of lower urinary tract and what are their risk factors?
Cancer Risk factor Common
location
Urothelial (transitional cell) carcinoma - Cigarette smoking (major) - polycyclic Bladder.
(MOST COMMON CANCER OF LOWER URINARY aromatic hydrocarbon (major),
TRACT- urothelium lines entire lower urinary tract) napthylamine
- Azo dyes (cancer seen in hair dressers)
- Long term cyclophosphamide or
phenacetin use
Squamous cell carcinoma - Arises after squamous metaplasia (urinary Bladder
tract doesn't have squamous cells)
• Chronic cystitis (older woman)
• Schistosoma hematobium (HY)
infection (middle eastern male)
• Long standing nephrolithiasis
Adenocarcinoma - No glandular cells in bladder: Bladder
• Arises from urachal remnant (HY)
(tumor at dome of bladder - HY)
• Cystitis glandularis - columnar
metaplasia due to chronic bladder
inflammation
• Exstrophy (failure to form caudal
portion of anterior abdominal and
bladder wall) - bladder exposed to
outside world which increases risk
in future to have cancer

Fig: extrophy
- Urachal remnant - urachus is remnant of channel between bladder and umbilicus (urine drains this way in
fetus during 1st trimester of pregnancy). It's lined by glandular cells. From 12th week of gestation, it's called
median umbilical ligament.

2. What are clinical characters of urothelial carcinoma?


- Generally seen in older adults as painless hematuria
- Tumor are multifocal and recur (entire urothelium is mutated via mechanism called field defect - ex- cigarette
smoking hits entire urothelial lining)

3. What are two pathways of urothelial carcinoma (HY)?


Flat Papillary
- Develops as high grade flat tumor and - Develops as low grade papillary tumor.
invades - Eventually becomes high grade papillary tumor and invades
- Associated with early p53 mutation (HY) - Not associated with early p53 mutation

Renal Page 10.1


LP is lamina propria

Renal Page 10.2


Chapter 13: Female Genital System
and Gestational Pathology
13.1 Vulva

1. Describe the following conditions about vulva.


Anatomy Skin and mucosa outside hymen (labias, mons pubis and vestibules)
Histology Lined by keratinized squamous epithelium
Bartholyn cyst - Unilateral, painful, cystic dilation of Bartholin gland (drains to inferior
vestibule)

Condyloma - Painless genital warts


- Cause:
• HPV 6, 11 more common STD (condyloma acuminatum - has
koliocytes - hallmark of HPV infected cells)
• Syphillis less common (condyloma latum)
- RARELY PROGRESS TO CANCER
Lichen sclerosis vs Lichen sclerosis Lichen simplex chronicus
lichen simplex
chronicus - Vulva epidermis thins and fibrosis - Hyperplasia of vulvar squamous
(sclerosis) happens in dermis - epithelium due to chronic
autoimmune etiology irritation and scratching
- Presents as leukoplakia with - Presents as leukoplakia with thick
parchment like vulvar skin leathery vulvar skin hyperplasia
- Most common in postmenopausal - Any age; usually young
women
- Benign - increased risk of SCC - Benign - no increased risk of SCC

Vulvar carcinoma
Epidemiology - Relatively rare (small portion of female genital cancer)

Female GU Page 3.1


Epidemiology - Relatively rare (small portion of female genital cancer)
- Generally seen in women of reproductive age (HPV cases) or >70 age (lichen
sclerosis cases)
Cause - HPV 16, 18 (most common)
- Non-HPV cases arise most often from long standing lichen sclerosis (generally
seen in >70 age)
Presentation - Presents as leukoplakia; need biopsy to distinguish from other causes of
leukoplakia
Pathophys of - Arises from vulvar intraepithelial neoplasia (VIN) - neoplasia limited to
HPV cases epithelium - dysplasia characterized by koliocytic change, disordered cellular
maturation, nuclear atypia

Extramammary paget disease


Definition - Presence of malignant epithelial cells in epidermis of vulva (carcinoma in situ) -
usually no underlying malignancy (contrast to Paget disease of nipple where
there's underlying carcinoma)
Presentation - Erythematous, pruritic, ulcerated vulvar skin

Fig: paget disease of nipple vs vulva (can occur in scrotum and other places
too)
Distinguish Paget cells - PAS+ve, keratin +ve and S100 -ve
from melanoma Melanoma - PAS-ve, keratin -ve and S100 -ve (PAS - shows mucus)

Female GU Page 3.2


13.2 Vagina

1. What type of cell lines the vagina?


- Non-keratinizng squamous epithelial cell

2. Describe vaginal adenosis.


Definition - Persistence of columnar epithelial cells in upper vagina
Cause - Use of diethylstilbestrol (DES) in utero - given to reduce pregnancy complication
(crosses placenta)
Pathophys - During development, squamous epithelium from lower 1/3rd of vagina (derived
from urogenital sinus) grows up to replace columnar cells (derived from
Mullerian ducts)
Complication - Progression to clear cell adenocarcinoma of vagina

Clear cells adenocarcinoma


Histology - Malignant proliferation of glands with clear cytoplasm
Cause - One is vaginal adenosis caused by diethylstilbestrol (DES)

Embryonal rhabdomyosarcoma (rhabdo = skeletal; myo = muscle)


Histology - Malignant proliferation of immature skeletal muscle
- +ve immunohistochemistry for desmin and myogenin
- May see cross striation of skeletal muscle
Presentatio - Rare cancer
n - Bleeding and grape like mass protruding from vagina or penis (seen <5 years) -
mass called SARCOMA BOTRYOIDES

Fig: biopsy and sarcoma botryoides of embryonal rhabdomyosarcoma

Vaginal carcinoma
Histology - Malignant proliferation of squamous epithelium lining vaginal canal

Female GU Page 4.1


Cause - Mostly HPV 16, 18, 31, 33
Precursor lesion - Vaginal intraepithelial neoplasia (VAIN) - neoplasm in epithelum
Lymph spread - Cancer from upper 1/3rd of vagina - inguinal lymph nodes
- Cancer from lower 2/3rd of vagina - iliac nodes

Female GU Page 4.2


13.3 Cervix

1. What are differences between exocervix and endocervix?


Exocervix Endocervix
Cervix that’s visible on vaginal exam Cervix that’s not visible on vaginal exam
Lined by nonkeratinizing squamous epithelium Lined by single layer of columnar cells
- Transformation zone - juction between exocervix and endocervix.

Cervical intraepithelial neoplasia (CIN)


Definition - Characterized by koliocytic change
Progression - Can progress to carcinoma in situ and invasive squamous cell carcinoma of cervix
- Progression not inevitable
Grades of - CIN I - involves <1/3rd of thickness of epithelium
dysplasia - CIN II - involves <2/3rd of thickness of epithelium
- CIN III - involves almost complete thickness of epithelium
- Carcinoma in situ (CIS) - involves entire thickness of epithelium but hasn’t invaded
basement membrane
- CIN I, II and III are reversible (reversibility of I>II>III); CIS irreversible

HPV
Prognosis - Mostly affects lower genital tract, especially cervix in transformation zone
- Infection usually eradicated by acute inflammation; persistent infection can lead to cervical
dysplasia and cervical intraepithelial neoplasia (CIN)
- Most common cause of SCC of genitourinal tract
Pathophys - Produces proteins:
• E6 - inactivates p53
• E7 - inactivates Rb
Vaccinatio - HPV 6, 11
n • Prevent against condolymata acuminata
- HPV 16, 18
• Prevent against CIN and cervical cancer

Cervical carcinoma
Types - Squamous cell carcinoma (80% of cases)
- Adenocarcinoma (15% of cases)
- Other - clear cell carcinoma, transitional cell carcinoma
Presentation - Mostly seen in middle aged woman (40-50 years)
- Presents as postcoital bleeding
Risk factor - HPV infection (main) - can cause both SCC and adenocarcinoma
- Immunosuppression (cervical carcinoma AIDS defining illness)
- Smoking
Complication - Hydronephrosis and post renal failure common cause of death
• Tumor invades through anterior uterine wall to bladder and blocks the ureters

Screening of cervical carcinoma - pap smear (THE MOST SUCCESSFUL SCREENING TEST DEVELOPED TO DATE)
- Cervical cancer went from being the most common to one of least common gynecologic cancers in US

Female GU Page 5.1


- Cervical cancer went from being the most common to one of least common gynecologic cancers in US
Goal - Catch dysplasia (CIN) before it develops to carcinoma
- Usually, it takes 10-20 years for CIN to develop to carcinoma
Frequency - Do pap smear every 3 years from >21 years
Process - Using a brush, scrape cells from transformation zone
- Dysplastic cells can be low grade (CIN I) or high grade (CIN II, III)
Confirmatory - Colposcopy - take chunks of tissue for biopsy
test
Limitations - Pap smear great for squamous cell carcinoma but not for adenocarcinoma of cervix
- If transformation zone not sampled, then false negative results may be seen
Normal
histology and
pap smear

Fig: right - normal squamous epithelial cells (small nuclei with abundant cytoplasm) present
in exocervix; left – normal squamous epithelial cells (red arrows), normal columnar
epithelial cells from endocervix – blue cytoplasm and cilia sometimes seen
Abnormal pap

Female GU Page 5.2


Abnormal pap
smear

Fig: black arrows indicate normal squamous epithelial cells with small round nuclei. Red
arrow - koliocytes - abnormally large and pleomorphic nuclei

Female GU Page 5.3


13.4 Endometrium and Myometrium

Basics
Endometrium - Mucosal lining of uterus
Myometrium - Smooth muscle underlying the endometrium
Hormone cycle of Proliferative Estrogen driven Endometrium grows
endometrium phase
Secretory phase Progesterone driven Endometrium prepares for
implantation
Menstrual Due to loss of Endometrium falls
phase progesterone

1. Describe the following pathologies.


Ascherman - Secondary amenorrhea due to loss of basalis (stem cell) and scarring
syndrome - Result of overagessive dialation and curettage (D&C)
Anovulatory - Lack of ovulation
cycle - Due to lack of ovulation, corpus luteum doesn't develop and progesterone isn't
made. Endometrium develops by estrogen but fails to be maintained; so it falls
off
- Common cause of dysfunctional uterine bleeding during menarche and
menopause

2. Differentiate acute and chronic endometritits.


Acute endmetritis Chronic endometritis
Bacterial infection of endometrium Chronic inflammation of endometrium characterized
by presence of plasma cells and lymphocytes in
biopsy (Plasma cell necessary to made diagnosis)
Cause: Causes:
- Usually due to retained product of - retained products of conception,
conception (after delivery or - IUD,
miscarriage) - serves as nidus of - TB, or
infection - pelvic inflammatory disease (gonorrhea,
chlamydia)
Presents as fever, abnormal uterine Presents as abnormal uterine bleeding, pain, and
bleeding and pelvic pain infertility

Endometrial polyp
Definition - Polyps formed due to hyperplastic protrusion of endometrium

Female GU Page 6.1


Presentatio - Abnormal uterine bleeding
n
Cause - Classic cause is tamoxifen (has anti-estrogenic effect on breast but pro-estrogenic
effect on endometrium)

Endometriosis
Definition - Endometrial glands and stroma outside uterine endometrial lining (the glands
follow normal menstruation cycle)
Cause - Most likely due to retrograde menstruation with implantation at an ectopic site
Common - Ovary (most common) - makes hemorrhagic (aka chocolate) cyst
sites - Uterine ligament - cause pelvic pain
- Pouch of douglas - pain with defecation
- Bladder wall - pain with urination
- Bowel serosa - abdominal pain and adhesions
- Fallopian tube mucosa - scarring increases risk for ectopic tubal pregnancy
- Adenomyosis - if endometriosis occurs in myometrium of uterus
- Sites classically appear as yellow-brown 'gun powder' nodules

Fig: common sites of endometriosis (left); chocolate cyst (right)


Presentatio - Pain that occurs prior to, during or after menstruation
n - Pain can occur during sexual intercourse, urination or bowel movement
- Some women have severe disabling pain and some have chronic pain in low back or
pelvis
Complicati - Increased risk of carcinoma at site of endometriosis (especially in ovary - aka
on endometoid tumor which is a malignant tumor of surface epithelium)

Endometrial hyperplasia
Definition - Hyperplasia of endometrial gland relative to stroma
Cause - Unopposed estrogen
• Obesity
• Polycystic ovary syndrome
• Estrogen replacement
Presentation - Classic is postmenopausal uterine bleeding
Classification - Based on architectural growth pattern (simple or complex)
- Or based on presence or absence or cellular atypia
Prognosis - If cellular atypia present, may progress to carcinoma (major complication

Female GU Page 6.2


Endometrial carcinoma (most common invasive carcinoma of female genital tract)
Definition - Malignant proliferation of endometrial glands
Epidemiology - Most common invasive carcinoma of female genital tract
Presentation - Classic is postmenopausal uterine bleeding
Causes - Via hyperplasia pathway (75% of cases) or sporadic pathway (25% of cases)

1. Differentiate endometrial carcinoma that arises by hyperplasia pathway or sporadic pathway.


Hyperplasia pathway Sporadic pathway
- Risk factors are increased estrogen - Carcinoma arises sporadically in atrophic endometrium
exposure: without any evident precursor lesion
• Early menarche/late - P53 mutation common; tumor is agressive
menopause
• Nulliparity (no babies)
• Infertility with anovulatory
cycle
• Obesity
- Average age of presentation 60 - Average age of presentation is 70 years
years
- Histology is endometroid (normal - Histology is serous and characterized by papillary
endometrium) structures with psammoma bodies

Leiomyoma (fibroma) - most common tumor in females


Definition - Benign neoplastic proliferation of muscle in myometrium
Epidemiolo - Most common tumor in females
gy - Common in premenopausal women; often multiple

Cause - Related to estrogen exposure


Presentatio - Usually asymptomatic; if symptomatic, may see abnormal uterine bleeding,
n infertility and pelvic mass
- Enlarges during pregnancy; shrink after menopause
- DO NOT PROGRESS TO LEIOMYOSARCOMA

Leiomyosarcoma
Definition - Malignant neoplastic proliferation of muscle in myometrium
Epidemiology - Often seen in postmenopausal women
Cause - Arise de novo; do not arise from leiomyoma

Female GU Page 6.3


Presentation - Single lesion with area of necrosis and hemorrhage
- Histology shows cellular atypia, mitotic activity and necrosis

Female GU Page 6.4


13.5 Ovary

1. Describe functional anatomy of ovary.


- Functional unit of ovary is follicle. Follicle has an oocyte surrounded by Granulosa and theca cells.
Cell type Location +ve Response
stimulus
Granulosa Located right around FSH Convert androgen from theca cells to
cells follicle estrogen (estradiol)
Theca cells Surround Granulosa LH Produce androgens
cells
- Estradiol surge causes LH surge which leads to ovulation (beginning of secretory phase of
endometrial cycle)
- After ovulation, follicle becomes corpus luteum which primarily secretes progesterone (drives
the secretory phase and prepared endometrium for possible conception)

2. Define hemorrhagic corpus luteal cyst and follicular cysts.


Cyst type Presentation/cause
Hemorrhagic corpus - Caused by hemorrhage into corpus luteal cyst. (Corpus luteal cysts are
luteal cyst formed if corpus leutem fails to disintegrate and instead persists)
- Normal; commonly found during early phase of pregnancy
Follicular cyst - Caused due to degeneration of follicles
- Normal in women (if present in small number) and have no clinical
significance

Polycystic ovarian disease (PCOD) : LH:FSH >2


Definition - Presence of multiple ovarian follicular cysts
Epidemiology - Affects 5% of women of reproductive age
Pathophysiol - Main finding is LH:FSH>2
ogy • High LH stimulates theca cells to produce lots of androgen (gives
hirsuitism)
• Androgen converted to estrone in adipose tissue. It gives -ve feedback to
FSH secretion; Granulosa cells and eventually the follicle degenerates
making cyst
Presentation - Presentation based on LH:FSH>2
• Hirsuitism (Due to excess androgens)
• Infertility (due to degenerated follicles)
• HIGH RISK OF ENDOMETRIAL CARCINOMA (due to high estrone)
• Insulin resistance and type 2 diabetes (10-15 years later)
- Classic pt is obese young woman wih infertility, oligomenorrhea, and hirsutism.

Female GU Page 1.1


13.6 Ovarian Tumors

1. What are the types of ovarian tumors?


- Ovary has 3 cell types: surface epithelium, germ cells and sex cord-stroma. Each cell type can give tumor
Tumor type Tumor subtype Epidemiology/remarks
Surface - Common (have cyst): - Most common ovarian tumor (70% of cases)
epithelium • Serous tumor (filled with water) - BRCA1 - Worst prognosis of all female genital tract cancer (often detected
mutation associated with serous late) - presents as vague abdominal discomfort or sign of bladder
carcinoma compression (urinary frequency)
• Mucinous tumor (filled with mucin)
- Less common (occur from metaplasia):
• Endrometriod tumor
• Brenner tumor
Germ cells - Tumor subtype based on tissue made by germ - Second most common ovarian tumor (15% of cases)
cells - All are malignant except cystic teratoma
Oocytes - Dysgerminoma (most common
malignant germ cell tumor)
Placental - Choriocarcinoma
tissue
Fetal - Embryonal carcinoma
tissue - Cystic teratoma (most common
germ cell tumor)
Yolk sac - Endodermal sinus (yolk sac)
tumor (most common germ cell
tumor in kids) - Schiller-Duval
bodies

Sex cord - Granulosa-theca Granulosa and theca


stroma tumor cells
- Sertoli-Leydig cell Sertoli and leydig cells
tumor
- Fibroma Fibroblasts
-
Surface epithelial tumors
1. How are serous and mucinous tumor classified into benign, borderline, and malignant?
Benign (aka cystadenoma) Borderline Malignant (aka cystadenoCARCINOMA)
- Has single cyst with simple, flat lining - In between feature between benign - Has complex cyst with thick, shaggy lining
ad malignant
- Better prognosis than malignant but
has malignant potential

- Mostly seen in premenopausal women - Mostly seen in postmenopausal women


(30-40 years) (60-70 yrs)

Female GU Page 2.1


(30-40 years) (60-70 yrs)

2. What ovarian cancer is BRCA1 mutation associated with?


- Serous carcinoma (serous cystadenocarcinoma)
- Pt can get prophylactic salpingo-oophrorectomy along with bilateral mastectomy as prophylasxix.

3. Differentiate between endometroid tumor and brenner tumor.


Endometroid tumor Brennor tumor
Tumor made of endometrial like gland (may arise from endometriosis) Tumor made up of bladder like
epithelium
Mostly malignant Mostly benign
15% endometroid carcinoma associated with independent endometrial carcinoma Benign; bladder epithelium; brennor
(endometriod type)

4. Describe the following features of surface epithelial tumors of female reproductive tract.
Distant metastasis Peritoneum (via direct contact) - see omental caking
Serum marker CA-125 - monitor treatment response and screening for recurrence

Germ cell tumors


1. Describe the following germ cell tumors
Dysgerminoma Choriocarcinoma Embryonal Cystic teratoma Endodermal sinus (yolk
carcinoma sac) tumor
- Malignant tumor with cells Malignant tumor - Malignant tumor - Tumor made up of fetal Malignant tumor that
that resemble oocytes (large composed of composed of tissue derived from >2 mimics yolk sac
cells, clear cytoplasm, cytotrophoblasts and large primitive embronic layers (ex-skin,
central nuclei) - fried egg syncytiotrophoblasts cells hair, cartilage, thyroid etc)
apperance
Malignant Malignant (trophoblasts Malignant - Mostly benign but if Malignant
are very invasive (aggressive) immature tissue (neural or
somatic malignancy)
present, indicates
malignant potential
- Most common malignant - Most common germ cell - Most common germ
germ cell tumor tumor cell tumor in kids
- Seminoma is male
counterpart (histologically
indistinguishable)

Fig: Schiller-Duval
Fig: hair and teeth seen in bodies (glomerulus like
cystic teratoma structure) classically
Fig: fried egg appearance on
Fig: Tumor is small, seen on biopsy.
biopsy
hemorrhagic and spreads
hemotageneously early
- Good prognosis; responds to - Poor response to
radiotherapy chemotherapy
- Serum LDL maybe elevated - High Beta-HCG - Hyperthyroidism in case of
characterstic (made by Struma ovarii (teratoma
syncytiotrophoblasts) - mainly composed of
may lead to thecal cysts thyroid tissue)
in ovary

Female GU Page 2.2


Sex cord-stromal tumors
1. What are sex cord stromal tumors?
- Tumors resemble sex cord stromal tissue of ovary

2. Differentiate between three types of sex cord stromal tumors.


Granulosa-theca cell tumor Sertoli-Leydig cell tumor Fibroma
Tumor of granulosa and theca cells Composed of Sertoli cells that make Tumor of fibroblast
tubules and Leydig cells between tubules
Malignant, but minimum risk of metastasis Benign tumor
Presentation based on excess estrogen Cells may produce androgens: - Associated with pleural effusions and
production - See hirsuitism and virilization (more ascites (meigs syndrome) - syndrome
- Prior to puberty - precocious puberty masculine) resovles with removal of tumor
- Reproductive age - menorrhagia and
metrorrhagia
- Postmenopause - endometrial
hyperplasia with postmenopausal
uterine bleeding

Fig: characterstic intracytoplasmic Reinke


crystals of Sertoli-Leydig cell tumor

Metastasis
1. Describe the two common tumor metastasis to ovary.
Source Presentation
Krukenberg tumor Gastric carcinoma (diffuse - Mucinous tumor
type) - Mostly bilateral (if unilateral, think primary mucinous
carcinoma of ovary)

Pseudomyxoma peritonei (myxoma = Mucinous tumor of - Huge amount of mucus in peritoneum "jelly belly"
tumor of connective tissue with mucus appendix
and gelatinous material)

Female GU Page 2.3


Pseudomyxoma peritonei (myxoma = Mucinous tumor of
tumor of connective tissue with mucus appendix
and gelatinous material)

Female GU Page 2.4


13.7 Gestational Pathology

Ectopic pregnancy
Definition - Implantation of fertilized ovum at site other than uterine wall
- Most common site is fallopian tube
Risk - Key risk is scarring (secondary to pelvic inflammatory disease or endometriosis)
Presentation - Lower quadrant abdominal pain a few weeks after a missed period
Prognosis - Surgical emergency; bleeding and fallopian tube rupture can occur

Spontaneous abortion
Definition - Abortion <20 weeks of gestation (usually during first trimester)
Epidemiology - Common; occurs in up to 1/4th of pregnancies
Presentation - Vaginal bleeding with passage of fetal tissue
- Cramp like pain
Cause - Most often due to chromosomal abnormalities (especially trisomy 16)
- Hypercoagulable states (ex - antiphospholipid syndrome)
- Congenital infection
- Exposure to teratogens (especially during first 2 weeks of pregnancy)

Teratogens
1. What is common timewise effect of teratogen exposure?
0-2 weeks of gestation Spontaneous abortion
3-8 weeks Risk of organ malformation
3-9 months Risk of organ hypoplasia

2. What are the effects of following teratogens?


Teratogen Effects
Alcohol - Most common cause of mental retardation
- Facial abnormalities and microcephaly
Cocaine - Intrauterine growth retardation and placental abruption
Thalidomide - Limb defects
Cigarette smoke - Intrauterine growth retardation
Isotretinoin - Spontaneous abortion
- Hearing and visual impairment
Tetracycline - Discolored teeth
Warfarin - Fetal bleeding
Phenytoin Digit hypoplasia and cleft lip/palate

Placenta previa, placenal abruption and placenta accreta


1. Differentiate the above three.
Placenta previa Placental abruption Placenta accreta
Placenta implants Separation of placenta from Placenta implants to myometrium
and closes cervical decidua (modified mucous with little or no intervening decidua

Female GU Page 7.1


and closes cervical decidua (modified mucous with little or no intervening decidua
os (opening) membrane that lines the uterus - Accreta - placenta grows
during pregnancy) superficially on myometrium
prior to delivery of fetus - Increta - placenta grows into
myometrium
- Percreta - placenta grows
completely through
myometrium and may invade
bladder or bowel

- Presents as - Presents as painful 3rd trimester - Presents with difficult delivery of


painless 3rd bleeding and fetal insufficiency the placenta and postpartum
trimester bleeding bleeding
- Often requires C- - Often requires hysterectomy
section delivery
- COMMON CAUSE OF STILLBIRTH

Preeclampsia, eclampsia and HEELP


Preecclampsia Eclampsia HELLP
- Pregnancy induced hypertension, - Preeclampsia + - Preeclampsia + thrombotic
proteinuria, and edema, usually arise seizures microangiopathy involving liver
in 3rd trimester (see hemolysis, Elevated Liver
- HTN may be severe leading headache enzymes and Low Platelets) - think
and visual abnormality as HUS of liver
- Caused due to abnormality In
maternal-fetal vasculr interface in
placenta
- Resolves with delivery - Warrant - Warrant immediate delivery
immediate
delivery

Sudden infant death syndrome


Definition - Death of infant (1 month - 1 year old) without obvious cause
Presentation - Infants often expire during sleep
Risk factors - Sleeping on stomach
- Exposure to cigarette smoke
- Prematurity

Female GU Page 7.2


Hydatidiform mole (molar pregnancy)
Hydatidiform mole Benign tumor that develops in uterus, caused by genetic error caused
due to abnormally fertilized egg.

Presentation - Uterus expands as if a normal pregnancy is present


- Uterus is much larger and B-HCG is much higher than expected for
date of gestation
- Grape like large villi pass through vagina in 2nd trimester of pregnancy
Classification
Treatment - Suction curettage
- Monitor B-HCG subsequently to ensure entire mole is removed
completely
Complication - Choriocarcinoma may arise after spontaneous abortion, normal
pregnancy or hydatidiform mole or as a spontaneous germ cell tumor
- Choriocarcinoma that arise from gestational pathway respond well to
chemotherapy; those from germ cell pathway don't.

1. Differentiate partial and complete hydatidiform mole.


Partial mole Complete mole
Genetics Normal ovum fertilized by two sperm Empty ovum fertilized by two sperm
(or 1 sperm with double (or 1 sperm with double
chromosome); 69 chromosome chromosome); 46 chromosome
Fetal tissue Present Absent
Villous edema Some villi are hydropic (edematous); Most villi are hydropic
some are normal
Trophoblastic - Focal proliferation present around - Diffuse proliferation around hydropic
proliferation hydropic villi villi
Risk for Minimal 2-3%
choriocarcino
ma

Female GU Page 7.3


Chapter 14: Male Genital System
Pathology
14.1 Penis

1. Describe the following penis conditions.


Pathologies Presentation Cause
Priaprism - Continuous penile erection, unrelated to sexual
arousal (medical emergency as ischemia and
thrombosis can lead to gangrene

Hypospadis - Opening of urethra on inferior surface of penis - Due to failure of


urethral folds to close

Epispadias - Opening of urethra on superio surface of penis - Due to abnormal


positioning of genital
tubercle
- Associated with bladder
exstrophy (bladder
located outside body
cavity)

Phimosis - Very tight foreskin on glans penis

Paraphimosis - Foreskin trapped behind glans penis

Condyloma - Warty growth on genital skin - HPV 6, 11


acuminatum Characterized by

Male GU Page 1.1


Condyloma
acuminatum - Characterized by
koliocytic changes

Lymphogran - Necrotizing granulomatous inflammation of inguinal - Chlamydia trachomatis


uloma lymphatics and lymph nodes (serotype L1-L3)
venereum - Perianal involvement may result in rectal stricture
(heals by fibrosis)

Squamous cell carcinoma of penis


Risk factors - HPV 16, 18, 31, 33 (2/3rd of cases)
- Lack of circumcision (foreskin as nidus of irritation and inflammation if not
maintained well)
Precursor Bowen disease (carcinoma in situ of penile shaft or scrotum)
lesions: - Presents as leukoplakia
- Presents in 5th decade of life

Erythroplasia of Queyrat (carcinoma in situ on glans) - Bowen's disease of glans)


- Presentas as erythroplakia

Male GU Page 1.2


-

Bowenoid papulosis (insitu carcinoma)


- Presents as multiple reddish papules
- Seen in younger pt (40's) compared to Bowen disease and erythroplasia of
Queyrat

Male GU Page 1.3


14.2 Testicle

Cryptorchidism (crypt = underground vault; orchid = penis)


Definition - Failure of testicle to descend to scrotal sac (testis develop in abdomen)
- Mostly unilateral

Epidemiolog - MOST COMMON CONGENITAL MALE REPORDUCTIVE SYSTEM


y ABNORMALITY
- Affects 1% of male births
Presentation Very prominent lydig cells on histo
Treatment - Most cases resolve spontaneously
- Orchiopexy <2 years if doesn't resolve (surgery to bring testis to right
position)
Complication - Testicular atrophy with infertility
- Increased risk for seminoma

Orchitis (inflammation of testis)


1. What are causes of orchitis?
Cause Notes Increased risk for
sterility?
Chlamydia trachomatis and/or - Seen in sexually active young adults Yes
Neisseria gonorrhoeae - Libido not affected because Leydig
cells are spared
E Coli and Pseudomonas - Seen in older adults
- Caused by spread of UTI pathogen to
reproductive tract
Mumps - Seen in teenagers (if <10 years, Yes
orchitis not seen)
Autoimmune orchitis - Characterized by granulomas
involving seminiferous tubules

Testicular torsion
Definition Twisting of spermatic cord which obstructs spermatic vein leading to
hemorrhagic testicular infraction in ~6 hrs
Cause Congenital failure of testes to attach to inner lining of scrotum (via processus
vaginalis)
Presentatio Sudden testicular pain and absent cremasteric reflex (nerves present in cord)

Male GU Page 2.1


Presentatio Sudden testicular pain and absent cremasteric reflex (nerves present in cord)
n

Varicocele, hydrocele and spermocele

1. Differentiate varicocele, hydrocele and spermocele.


Varicocele Hydrocele Spermatoce
le
Definiti - Dilation of spermatic vein - Fluid collection in tunica Cystic
on vaginalis dialation of
epididymis
Cause - Mostly left sided as left testicular vein - Incomplete closure of
drains to left renal vein; right testicular processus vaginalis and
vein goes straight to IVC. (seen in renal its communication with
cell carcinoma because carcinoma peritoneal cavity - in kids
frequently involves renal vein) - Blockage of lymphatic
- Seen in large percent of infertile males drainage - adults
- Presents as 'bag of worms' with scroal
swelling

Male GU Page 2.2


14.3 Testicular Tumors

1. Describe following features of testicular tumors.


Presentation - Firm, painless testicular mass that can't be transilluminated
Biopsy - Do radical orchiectomy (mass not biopsied due to risk of seeding the scrotum)
Epidemiology - Most tumors are malignant germ cell tumors

2. What are the types of testicular tumors?


- Classification is based on whether they arise from germ cells or sex cord-stroma; lymphoma also a cause.
Germ - Tumor subtype based on tissue made by germ cells (can use seminoma vs non seminoma for clinical
cells purposes)
( mostly Seminoma - Seminoma (most common testicular tumor)
maligna
nt) Mixed germ cell tumor - Mixed germ cell tumor
(>95% - Prognosis based on worst component
of Placental tissue - Choriocarcinoma
testicul
Fetal tissue - Teratoma
ar
- Embryonal carcinoma
tumors)
Yolk sac - Endodermal sinus tumor (most common germ cell tumor in kids)

Sex cord - Leydig cell tumor - Produce androgens


stroma - Precocious puberty in children or gynecomastia in adults
(mostly - May see Reinke crystals on biopsy
benign)

Fig: characteristic intracytoplasmic Reinke crystals of Leydig cell tumor

- Sertoli cell tumor - Usually clinically silent

Lymphoma - Most common cause of testicular mass in males >60 years


- Mass often bilateral
- Most common type is diffuse large B cell lymphoma
- Don't see epithelial carcinoma in testes because they aren't surrounded by epithelium They're surrounded by
tunica vaginalis (mesenchyme)

Germ cell tumors


1. Describe the following features of germ cell tumors.

Male GU Page 3.1


1. Describe the following features of germ cell tumors.
Epidemiolo - Most common testicular tumor (>95% of cases)
gy - Mostly seen in 15-40 year olds
Risk factor - Cryporchidism
- Klienfelter syndrome
Division Seminoma Non-seminoma
Most common
testicular tumor
55% of germ cell 45% of germ cell tumors
tumors
Highly responsive to Variable response to treatment
radiotherapy
Metastasize late Metastasize early
Excellent prognosis Poor prognosis
Include teratoma, embryonal carcinoma, endodermal sinus tumor,
choriocarcinoma and mixed germ cell tumor

2. Differentiate the following testicular tumors.


Seminoma Teratoma Embryonal Endodermal sinus Choriocarcinoma
carcinoma (yolk sac)
Most common Most common
testicular tumor testicular tumor in
children
Malignant Malignant (in Malignant Malignant Malignant; spreads
females, it's mostly early
benign)
- Large cells with clear - Mature fetal - Immature cells - Schiller-Duval - Tumor of
cytoplasm and tissue from >2 that may bodies on histology cyncytiotrophoblasts
central nuclei embryonic layers produce glands (glomerulus like) and cytotrophoblasts
- Forms homogenous - Forms (placenta like tissue
mass with no hemorrhagic but absent vilil)
hemorrhage or mass with
necrois necrosis

- B-HCG rarely - AFP or B-HCG - AFP or B-HCG - AFP increased - B-HCG increased
increased increased incresed (syncytiotrophoblasts
produce it)
- Alpha subunit of HCG
similar to FSH, LH and
TSH; may see
hyperthyroidism and
gynecomastia

Male GU Page 3.2


14.4 Prostate

1. Describe histology of prostate gland.


- Consists of gland and stroma (connective tissue) which is maintained by androgens (stromal cells
make DHT by 5 alpha reductase from testosterone)
- Glands are 2 cell layer thick
○ Inner luminal cells
○ Outer basal cells

Fig: G is for glands; S for stroma

2. Describe acute vs chronic prostatitis.


Acute prostatitis Chronic prostatitis
- Acute inflammation of prostate (mostly due to - Chronic inflammation of prostate
bacteria)
- Causes:
• Chlamydia and gonorrhoeae in young adult
• E coli and pseudomonas in old adults
- Presentation: - Presentation:
• Dysuria with fever and chills • Dysuria with pelvic or lower back
• On DRE, prostate is tender and boggy pain
- Prostatic secretion: - Prostatic secretion:
• WBC high • WBC high
• +ve bacterial culture • -ve bacterial culture

Benign prostate hyperplasia


Cause - Age related change (present in most men >60 years)
- No increased risk of cancer

Male GU Page 4.1


- No increased risk of cancer
Pathophys - Related to increased dihydrotestosterone (DHT)
• DHT made from testosterone by 5alpha-reductase in stromal cells
• DHT acts on androgen receptors on stromal and epithelial cells resulting in
hyperplastic nodules
Clinical - Presentation based on pinching of urethra
features • Problems starting and stopping urine stream; dribbling
• Impaired bladder emptying - high risk of infection and hydronephrosis
• Nocturia; polyuria
• Hypertrophy of bladder wall muscle - increased risk for bladder diverticula
• Microscopic hematuria maybe present

Labs - Slightly increased prostate specific antigen (PSA) - made by prostatic glands and it
liquefies semen (still <10 ng/ml)
Treatment - Alpha1 antagonist (terazosin) to relax smooth muscle - also lowers BP
- Alpha1a antagonist (tamsulosin) - don't act on vascular smooth muscle; use for
normotensive pt
- 5alpha reductase inhibitor (reduce androgenerigc stimulation of prostate and
reduce its size)
• decrease conversion of testosterone to DHT
• Takes months to produce affect
• Helps with baldness
• Side effect - gynecomastia and sexual dysfunction

Prostate adenocarcinoma
Epidemiolog - Most common cancer in men
y - 2nd most common cause of cancer-related death in men
Risk factors - Age
- Race (AA>Caucasians>Asians)
- Diet high in saturated fats
Presentatio - Mostly clinically silent (tumor arises on posterior peripheral region of prostate;
n don't see urinary symptoms)
Screening - PSA and DRE from 50 years age
- PSA:
• Normal serum PSA increases with age due to BPH (~2.5 for 40-49; ~7.5 for
70-79)
• PSA > 10 ng/ml highly worrisome for any age
• Also see decreased free PSA as cancer makes PSA that’s bound.
Diagnosis Biopsy:
• See small invasive glands with prominent nucleoli
• Ducts maybe only one layer thick instead of having basal and laminal layer

Male GU Page 4.2


Fig: notice nucleus with prominent nucleoli
Grading - Based on tumor architecture alone (not on nuclear atypia)
- Multiple region of tumor have different architecture
- Gleason scoring:
• Take two distinct areas and grade each (1-5). Total score = 10
• High grade = worse prognosis
Metastasis - Lumbar spine or pelvis common site
- See osteoblastic metastasis
• Presents as lower back pain
• Increased serum alk phosphatase, PSA and prostatic acid phosphatase (PAP)
Treatment - Prostatectomy for localized
- Hormone suppression for advanced disease (goal to reduce testosterone and DHT)
• GnRH analog (ex - leuprolide) - reduces LH and FSH production
• Flutamide - competitive inhibitor of androgen receptor

Male GU Page 4.3


Chapter 15: Endocrine Pathology
15.1-15.2 Anterior and Posterior Pituitary Gland

Hormones by hypothalamus Hormones made by ant. pituitary


1. Thyrotropin releasing hormone (TRH) 1. Thyrotropin (aka thyroid stimulating hormone)
2. Gonadotropin releasing hormone (GNRH) 2. Gonadotropin (FSH and LH)
3. Corticotropin releasing hormone (ACRH) 3. Adrenocorticotropin (ACTH)
4. Growth hormone releasing hormone 4. Growth hormone
5. Somatostatin 5. Prolactin (lactotropin) - most common pituitary adenoma
6. Prolactin inhibiting hormone (dopamine)

Hormones in post. Pituitary (made in hypothalamus)


1. ADH (aka vasopressin)
2. Oxytocin - uterine contraction during pregnancy
Anterior pituitary
1. What type of tumor is pituitary adenoma?
- Benign tumor of anterior pituitary cells.
- Can be functional (hormone producing) or non-functional (silent)

2. What are clinical presentation of non-functional tumor of pituitary?


- Bitemporal heminopsia (due to mass effect)
- Hypopituitarism (due to mass effect)
- Headache

3. What are most common functional tumor of ant. Pituitary?


- Prolactionma (most common pituitary adenoma) - 50% of adenomas
- Growth hormone (somatostatin) cell adenoma - 10-15% of adenomas
- ACTH cell adenoma - 10% of adenoma
- TSH cell, LH producing, and FSH producing adenomas are rare

4. What are clinical presentation of prolactinoma? How do you treat?


- In females - galactorrhea and amenorrhea (prolactin inhibits GNRH synthesis and release)
- In males - decreased libido and headache. Males don't get galactorrhea because they don't have
sufficient breast tissue.
- Treat - give prolactin inhibiting hormone (aka dopamine), dopamine agonists (bromocriptine or
cabergoline) or surgery

5. What is functional unit of breast?


- Terminal duct-lobular unit. Lobular unit has glands that make milk. Males only have terminal duct but
not lobular unit so can't make milk. Females develop lobular unit after puberty.

Endocrine Page 1.1


Fig - rough sketch of terminal duct (TD) and lobular unit (LU)

6. What is clinical presentation of Growth hormone adenoma?


- In kids - gigantism
- In adults - acromegaly
○ Enlarged bones of hand, feet, jaws, large tongue.
○ Growth of visceral organs (cardiac failure is most common cause of death in these patients)
- Secondary diabetes mellitus present (because GH inhibits glucose intake by cells; GH induces
gluconeogenesis by liver)

7. How do you diagnose growth hormone adenoma? How do you treat?


- Elevated GH and IGF-1 (insulin like growth factor) - IGF is made by liver in response to GH
- Lack of GH suppression in response to oral glucose (why)
- Treatment-
○ Octreotide (somatostatin analog)
○ GH receptor antagonist
○ Surgery

Hypopituitary
8. When do we see hypopituitarism?
- When >75% of pituitary parenchyma is lost

9. What are some causes of hypopituitarism?


- Pituitary adenoma (common cause in adults ) - mass effect or apoplexy (bleeding in adenoma)
reduces hormone production
- Carniopharyngioma (common cause in kids) - same reason as above
- Sheehan syndrome (HY) - during pregnancy, there's high need of hormones so pituitary doubles in
size but its blood supply doesn't. Blood loss during parturition can trigger pituitary infraction.
○ Presents as poor lactation, loss of pubic hair (HY) and fatigue.
- Empty sella syndrome - herniation of arachnoid or pia into pituitary can damage pituitary.
○ Can occur congenitally too

Posterior pituitary
10. What are clinical presentation and causes of central diabetic insipidus? What is diagnosis and
treatment?
- Presentation (based on loss of free water)
○ Polyuria and polydipsia
○ Hypernatremia and high serum osmolality
○ Low urine osmolality and specific gravity
- Causes - damage (tumor, trauma, infection or inflammation) to hypothalamus or pituitary

Endocrine Page 1.2


- Causes - damage (tumor, trauma, infection or inflammation) to hypothalamus or pituitary
- Diagnosis - urine osmolality doesn't increase in response to water deprivation
- Treatment - Desmopressin (ADH analog)

11. What are clinical presentation and causes of nephrogenic diabetic insipidus?
- Presentation - same as central DI but no response to desmopressin
- Causes - mutation or drugs (demeclocycline - old antibiotic, not used often now; and lithium)

12. What are clinical presentation and causes of syndrome of inappropriate ADH (SIADH)?
- Presentation -
○ Hyponatremia and low serum osmolality
○ Mental status change and seizure. - due to nerve swelling
- Causes - ectopic production (ex - small cell carcinoma of lung)
○ CNS trauma
○ Pulmonary infection
○ Drugs (cyclophosphamide)
- Treatment - free water restriction, demeclocycline

Endocrine Page 1.3


15.3-15.7 Thyroid

1. Differentiate T3 and T4
T4 (thyroxine) T3 (triiodothyronine)
Both are made from tyrosine 10 times more potent than T4
Most of it made from T4
Thyroid secretes T4>>T3
T1/2 = 7 days T1/2 = 7 days
T4--> T3 conversion reduced in severe illness

2. How are thyroid hormones transported in blood?


- Tyrosine binding protein (carries 70%)
- Albumin (carries 20%)
- Transthyretin (pre-albumin) (carries 10%)
- Free T4 is 0.04% and free T3 is 0.03%

3. How are T3 and T4 made in thyroid gland? What is the key enzyme?
- Key enzyme is thyroid peroxidase
- Thyroglobulin are long chain of tyrosine that's stored as colloid in thyroid gland. T3/T4 are made
from it.
- Steps
○ Iodine is oxidized to oxidized iodine by thyroid peroxidase (oxidation step)
○ Oxidized iodine reacts with tyrosine to make monoiodotyrosine
○ Oxidized iodine reacts with monoiodotyrosine to make diiodotyrosine
○ The last two steps are called organification
○ Monoiodotyrosine + diiodotyrosine = triiodothyronine
○ Diiodotyrosine + diiodotyrosine = thyroxine
○ The last two steps are called coupling

4. What are functions of thyroid hormones?


a. Increase or decrease gene transcription by binding to nuclear receptor
b. Imp in development (Cretinism is mental retardation and dwarfism caused due to
impairment of brain and skeletal development due to hypothyroidism)
c. Increase BMR, O2 consumption and free radical formation
d. Exacerbate diabetes mellitus (increase glycogenolysis and increase hepatic gluconeogeneis)
(hyperthyroid = DM)
e. Increase LDL receptors (hyperthyroidism = low serum LDL)
f. Increase ionotropy (contractility), chronotropy (HR),
g. Increase sensitivity to catecholamine for B1 receptors(hyperthyroidism = hyperactive SANS)
h. Pulm - maintains hypoxic and hypocapnic drive (hypothyroid = hypoventilation and
respiratory arrest)
i. GI - increase gastric motility (hypothyroid= constipation, hyperthyroid = diarrhea)
j. Skeletal - increase bone reasborption and decrease bone formation (hyperthyroid =
osteopenia)
k. Neuromuscular - (hyperthyroid = increased reflex, anxiety, hyperactivity; hypothyroid =
decreased reflexes, fatigue and sluggish)

5. What causes thyroglossal duct cyst? (HY)


- Thyglossal duct is a path for descent of thyroid from tongue to its location in neck. The cyst dies

Endocrine Page 2.1


- Thyglossal duct is a path for descent of thyroid from tongue to its location in neck. The cyst dies
out normally; if it persists, it may cause cystic dilation and seen as anterior neck mass.

6. What causes lingual thyroid?


- If thyroid tissue persists at base of tongue, it's present as a mass.

Hyperthyroidism (thyrotoxicosis)
7. What are presentation of hyperthyroidism?
a. Increased basal metabolic rate (due to increased synthesis of Na/K pump) (HY) -
i. wt loss despite increase hunger
ii. Tremor, anxiety, insomnia, and heightened emotions
iii. Heat intolerence and sweating
b. Increased SANS (due to increased expression and activity of B1 s)
i. Cardiac - tachycardia, arrhythmia (esp in elderley)
c. Hyperglycemia (increased gluconeogenesis and glycogenolysis) (HY)
d. Hypocholesteremia and low serum LDL (increased LDL receptors) (HY)
e. Diarrhea and malabsorption (increased GI motility)
f. Osteopenia and hyercalcemia (increased bone reabsorption)
g. Decreased muscle mass and weakness
h. Oligomenorrhea

8. What is epidemiology of Grave's disease? What is it?


- Most common cause of hyperthyroidism
- Classically occurs in women of childbearing age (this group has high incidence of autoimmune
disease)
- Grave's is autoantibody IgG generation that stimulates TSH receptor leading to hyperthyroidism
(type II hypersensitivity)

9. What are clinical presentation of grave's disease?


- Diffuse goiter - TSH hormone is a trophic hormone that leads to hyperplasia and hypertrophy
- Tibial myxedema and exopthalmus

10. What causes exopthalmos and pretibial myxedema (characterstic finding) in grave's?
○ Fibroblast behind eye and in tibia has TSH receptors. Excitation leads to glycosaminoglycan
(chondroitin sulfate and hyaluronic acid) buildup, inflammation, fibrosis, and edema.
○ Myxedema feels like dough. And myx refers that edema is not caused by water.
11. What is histology of Grave's?
- Histology shows irregular follicle and scalloped colloid and chronic inflammation

Endocrine Page 2.2


Fig - Thyroid in grave's. Notice irregular follicles. Also, the white space between colloid and
thyroid tissue is called scalloped and is classic occurrence in Grave's.

12. What is treatment of Grave's?


- B-blockers
- Antithyroid durgs (methimazole, propylthiouracil, thyoamide) - concentrate in thyroid and block
thyroid peroxidase; also prevent T4 --> T3 conversion in peripheral tissue
- I-131 - thyroid takes it and gets destroyed (permanent hypothyroidism major complication)
- Total thyredctomy

12.5. What are lab findings in Grave's?


- Increased total and free T4
- Decreased serum TSH
- Hypocholesteremia (HY)
- Hyperglycemia (HY)

13. Differentiate thyroid storm vs myxedema coma


Thyroid storm (hyperthyroid emergency) Myxedema coma
(hypothyroid emergency)
Presentation - fever (>400C), sweating, tachycardia/afib, delirium, Presentation- mental
nausea, vomiting status change from
confusion to coma
Cause - increased catecholamines action and massive T3/T4 excess. Hypoglycemia,
Most common trigger - acute stress such as surgery, childbirth, MI. hypothermia,
hypothermia,
hypoventilism
Treatment High incidents of death
- B-blockers, propylthiouracil, and steroids
- give Iodine salt (wolff-chaikoff block - increased iodine in
blood leads to decreased iodine uptake by thyroid and
decreased production of thyroid hormones)
Treatment- high dose of
levothyroxine
Cause- infection, stroke, in
patients with
hypothyroidism

Endocrine Page 2.3


14. What causes multinodular goiter? Is it toxic?
- Multiple nodules and enlarged thyroid
- Caused due to relative iodine deficiency
- Usually nontoxic (rarely, some regions can be toxic, i.e., produce T3/T4)

Fig - multinodular goiter

Hypothyroidism
15. What is cretinism? What are it's findings?
- It's developmental delay caused due to hypothyroidism.
- Classic findings
○ Mental retardation and dwarfism due to poor brain and skeletal development
○ Enlarged tongue (due to myxedema)
○ Umbilical hernia

16. What are causes of cretinism?


- Maternal hypothyroidism during early pregnancy
- Thyroid agenesis (pt don't develop thyroid)
- Dyshormonogenic goiter (pt can't make thyroid hormones - ex - pt with mutation in thyroid
peroxidase gene)
- Iodine deficiency

17. What's classic findings in myxedema (hypothyroidism in old kids and adults)?
a. Myxedema - classically in tongue (large tongue) and larynx (gives deep voice) (HY) - due to
increased TSH
b. Decreased BMR and decreased SANS
i. Wt. gain despite normal activity
ii. Cold intolerane and decreased sweating
iii. Bradycardia
c. Hypoventilation and respiratory arrest - thyroid maintains respiratory drive
d. Slow mental activity
e. Muscle weakness
f. Hypoglycemia, hypercholesteremia
g. Oligomenorrhea (seen in both hyper and hypothyroidism)
h. Constipation

18. What are causes of hypothyroidism?


- Hashimoto's (most common cause)
- Iodine deficiency
- Drugs (lithium)

Endocrine Page 2.4


- Drugs (lithium)

Thyroiditis
19. What is presentation of hashimoto's thyroiditis? What chemical is is associated with? What are lab
results?
- It's an autoimmune attack to thyroid peroxidase and thyroglobulins
- Associated with HLA-DR5 (HY)
- Initially see increase T3/T4 (and low TSH) due to gland destruction. Later on, we see decreased
T3/T4 (and high TSH) due to gland destruction.
- Suspect other autoimmune disease - type 1 DM, pernicious anaemia, rheumatic disease.
- Lab results - anti TPO, anti-thyroglobulin, anti-microsomal antibodies, high TSH, low T3/T4

20. What is histologic appearance of Hashimoto's thyroditis? (HY)


- Chronic inflammation (see lymphocytes) with germinal cells.
- Presence of Hurthle cells (eosinophilic metaplasia of cells that line follicles).

Fig - Hashimoto's thyroiditis. CI is chronic inflammation. GC is germinal center. Little circles


on left are herthel cells.

21. What disease do pt. with Hashimoto's have increased risk of? How? What's presentation?
- B cell lymphoma.
- How - Germinal center makes post germinal center B cells. It makes marginal zone which results in
marginal zone lymphoma.
- Presentation is pt with long standing hypothyroidism that presents with enlarged thyroid.

22. What is SUBACUTE GRANULOMATOUS (DE QUERVAIN) THYROIDITIS? What's its presentation?
What's its prognosis?
- It's granulomatous thyroiditis that follows viral infection (subacute means it occurs after acute
process).
- Presentation - Hypothyroidism is transient, and pt has tender thyroid (if a young female has tender
thyroid, think subacute granulomatous thyroiditis).
- Prognosis - It's self-limited and don't progress to hypothyroidism.

23. What is Reidel Fibrosing thyroiditis? What is presentation?


- Chronic inflammation of thyroid with extensive fibrosis
- Presentation -
○ hard as wood thyroid that's non tender
○ Fibrosis may extend to local structures - ex - airway
Clinically mimics anaplastic carcinoma but pt are younger and usually female(in anaplastic,

Endocrine Page 2.5


○ Clinically mimics anaplastic carcinoma but pt are younger and usually female(in anaplastic,
pt are older)

Thyroid cancer
24. What are basics of thyroid cancer?
- Most nodules are distinct and solitary
- Most nodules are likely to be benign than malignant

25. What causes positive and negative in radioiodine uptake study?


- Positive is when thyroid takes radioiodine injected in blood - Graves, nodular goiter
- Negative - adenoma and carcinoma (do biopsy by FNA)

26. Describe follicular adenoma (adenomas are benign; adenocarcinoma are cancerous).
- Follicle proliferate in a benign way and are surrounded by a fibrous capsule
- Called follicular because tumor also makes thyroid follicle.
- Tumor mostly non-functional (don't secrete hormone)

Fig - follicular adenoma. Red line is the capsule that divides adenoma (bottom half) from
normal thyroid (top half)

27. What are 4 types of thyroid carcinomas (malignant stuff)?


a. Papillary carcinoma
b. Follicular carcinoma
c. Medullary carcinoma
d. Anaplastic carcinoma

28. What is epidemiology, risk factor, prognosis and histologic feature of papillary carcinoma?
- Most common thyroid carcinoma (80% of thyroid carcinoma)
- Major risk - exposure to ionizing radiation in childhood
- Prognosis - excellent even though often spreads to cervical nodes
- Histology -
- Papillae of cells seen (so called papillary carcinoma
- Diagnosis is made by nuclear features -
 Coffee bean nucleus (presence of nuclear groove)
 Orphan eye annie nucleus (nucleus has white stuff resembling white of eye)
 Psammomma bodies (concentric calcification of papillaes).

Endocrine Page 2.6


Fig - Papillary carcinoma. red circles indicate orphan eye annie nucleus. Turquoise circle
shows coffee bean nucleus.

Fig - Psammoma bodies in papillary carcinoma shown in black circle.

29. What is histologic feature of follicular carcinoma? Can you diagnose by FNA?
- Similar to follicular adenoma (has fibrous capsule) but cells invade through capsule (hallmark)
- FNA can't distinguish between follicular adenoma and follicular carcinoma because capsular
invasion can't be assessed by FNA.
- Even though most carcinomas spread via lymph nodes, follicular carcinoma spreads
hematogenously (by blood).

Endocrine Page 2.7


Fig - follicular carcinoma. Red line shows the capsule and the break in it.

30. What are 4 carcinomas that spread by blood instead of lymph? (HY)
- Renal cell carcinoma
- Follicular carcinoma of thyroid
- Hepatocellular carcinoma
- Corneal carcinoma

31. Describe medullary thyroid carcinoma (MTC). How is it diagnosis?


- Malignant proliferation of parafollicular C cells that produce calcitonin
- Diagnosis -
○ Malignant cells in amyloid stroma - indicates MTC. (Calcitonin deposits in tumor as amyloid
(cause localized amyloidosis), and +ve calcitonin immunostain)
○ Pt has high level of calcitonin which can lead to hypocalcemia

Fig - MTC biopsy. All the pink stuff seen is calcitonin amyloid.

32. Describe familial cases of MTC (HY). What's significance of RET oncogene (HY)?
- Often associated with MEN 2A or 2B phenotype. (MEN = muliple endocrine neoplasia)
- MEN 2A - often see MTC, pheochromocytoma and parathyroid adenomas

Endocrine Page 2.8


- MEN 2A - often see MTC, pheochromocytoma and parathyroid adenomas
- Men 2B - often see MTC, pheochromocytoma and ganglioneuroma of oral mucosa
- Familial cases of MTC is classically associated with mutation in RET oncogene
- If a person has RET oncogene, do a prophylactic thyroidectomy

33. Describe anaplastic carcinoma. How do you diagnose?


- Has the worst prognosis of all thyroid carcinomas
- It is undifferentiated and classically seen in old people
- Tumor has +ve keratin stain
- Often invades local structures leading to dysphagia or respiratory compromise.
- Clinically similar to Reidel fibrosing thyroidiyis but cancer is often seen in old people and
thyroidiyis in young females.

Fig - highly malignant cells in anaplastic carcinoma that don't resemble anything seen in thyroid
usually

Endocrine Page 2.9


15.8 Parathyroid

1. What is vitamin D? How do we get it?


- Two most imp vitamin D are vit D3 (cholecalciferol) and vit D2 (ergocalciferol)
- Cholecalciferol (D3) is made from 7-dehydrocholesterol by UV in skin
- Ergocalciferol (D2) is taken from food.
- Both of those are activated by liver in unregulated way, and kidney in regulated way

2. How does kidney activate vitamin D?


- Vit D itself is a prohormone. Kidney uses alpha 1 hydroxylase to make 1,25 OH D (active
vitamin D).

3. What are functions of PTH hormone?


- Chief cells make PTH that increase free serum calcium
- PTH acts on 3 main tissue
○ Stimulate kidney to increase vit D activation
○ Increase Ca absorption from kidney and phosphate excretion (phosphate excretion key
because it increases free Ca in blood)
○ Increase Ca and PO4 absorption from gut - this action is via Vit D
○ Increase osteoclast activity (PTH activates osteoblast which secretes M-CSF
(macrophage colony stimulating factor) which increase osteoclast differentiation and
activation) (HY)

4. What regulates blood PTH hormone?


- PTH release is highly sensitive to serum ca.
- Vitamin D also reduces PTH release
- Increase Ca ---> Gq and Gi activation. Gq increase calcium release in parathyroid cells --->
Decreases PTH synthesis. Gi decreases cAMP which reduces PTH synthesis
- Low Ca ---> Gs activation. Gs increases cAMP which increases PTH synthesis

5. What are functions of Vit D?


- Main function is to maintain bone mineralization
- Increase Ca and PO4 reabsorption in kidney
- Increase Ca and PO4 absorption in gut
- Decrease PTH secretion

6. What are function of calcitonin and phosphatonin?


- Tone down serum Ca and PO4
- Phosphatonins are important because intestinal absorption of phosphate is unregulated

Primary hyperparathyroidism
7. Explain primary hyperparathyroidism.
- Excess PTH; most common cause is PT adenoma (80%)
- Other causes - PTH hyperplasia, PTH carcinoma

8. Describe presentation of PT adenoma (i.e., symptoms of hypercalcemia).


- Benign; mostly asymptomatic
- If symptomatic
○ Nephrolithiasis - kidney stone (classic is calcium oxalate)
○ Nephrocalcinosis - example of metastatic calcification - Ca deposits in tubules
○ CNS disturbance - depression, seizure
Constipation, peptic ulcer, acute pancreatitis (HY) - think Ca as an enzyme activator that

Endocrine Page 3.1


○ Constipation, peptic ulcer, acute pancreatitis (HY) - think Ca as an enzyme activator that
activates pancreatic enzymes.
○ Osteitis fibrosa cystica - massive reabsorption of bone leading to fibrosis and cyst
formation
- Treatment - surgery

9. What are lab findings in primary hyperparathyroidism?


- Increase serum PTH
- High serum Ca
- Low serum phosphate (PTH increase phosphate excretion)
- High urine cAMP (HY)
○ PTH works in kidney via Gs to increase cAMP. Some cAMP gets to urine.
- High serum alkaline phosphatase (HY)
○ Alkaline phosphatase generates alkaline environment in bone so that new bone can be
laid down. Alk Phos is a sign of osteoblast activity. Remember that PTH first activates
osteoblast which in turn activates osteoclast. Osteoblast activation leads to high alk
phos.

Secondary hyperparathyroidism
10. What are some causes for 20 hyperparathyroidism?
- Most common is chronic renal failure (HY)
○ Decreased phosphate excretion --> reduced free calcium in blood as most of it is bound
to phosphate ---> increased PTH production and increased bone reabsorption

11. What are lab findings in 20 hyperparathyroidism?


- Increased PTH
- Decreased serum Ca (remember that the whole process is driven by high PO4. It binds to free
serum Ca and reduces it)
- Increased serum PO4
- Increased alkaline phosphatase (increased PTH leads to increase alk phos)

Hypoparathyroidism
12. What are causes of hypoparythoroidism?
- Autoimmune
- DiGeorge syndrome (failure of develop 3rd and 4th pharyngeal pouch. Parathyroids develop
from 3rd and 4th pharyngeal pouch).

13. What are its presentation - due to low Ca?


- Numbness and tingling (specially perioral)
- Muscle spasm (ex - when you put BP cuff on them or touch their face)
- Low serum Ca and PTH

Pseudohypoparathyroidism
14. What causes pseudohypoparathyroidism?
- Caused due to end organ resistance of PTH
- Hypocalcemia with increased PTH level (also see this is secondary hyperparathyroidism; but 20
hyperparathyroidism has increased serum phosphate levels too)
- Autosomal dominant form (mutation of Gs) associated with short stature and short 4th and
5th digit

Endocrine Page 3.2


15.9 Endocrine Pancreas

Insulin - major anabolic hormone. Upregulates GLUT4 receptors in muscle and fat. Increased
glucose leads to glycogen synthesis, fat synthesis, protein synthesis
Glucagon - major catabolic hormone. Causes gluconeogenesis from AA, lipolysis and
glycogenolysis.

1. Describe the anatomy of endocrine pancreas.


- Composed of cluster of cells called islet of Langerhans
- Single islet has many types of cells; each type producing one type of horome
- Insulin is made by B cells; lie in center of islet
- Glucagon is made by alpha cells.

Type 1 DM
2. Explain Type 1 DM
- Autoimmune destruction of B cells by T lymphocytes (type 4 hypersensitivity) - see inflammation
of islets
- Autoantibodies against insulin - can be seen years before disease
- Associated with HLA DR3 and DR4

3. What's presentation of type 1 DM?


- Classically seen in children
- Presentation (of insulin deficiency)
○ High blood glucose
○ Weight loss despite polyphagia (high hunger), low muscle mass - due to unopposed
catabolic action of glucagon
○ Polyuria, polydipsia, glycosuria
- Treatment - lifelong insulin

4. Explain mechanism of DKA (feared complication of type 1 DM)


- Excess serum ketoacids; often triggered by stress (ex-infection)
- Epinephrine increases glucagon that exacerbates lipolysis. Free fatty acids generated go to liver
and get converted to ketones.

5. What is presentation of DKA?


- Hyperglycemia (>300 mg/dl) - due to unopposed glucagon actoin
- Anion gap metabolic acidosis (from ketoacids)
- Hyperkalemia (HY)
○ insulin stimulates Na/K pump. Also, acidosis is compensated by exchanging H+ from serum
to K in cells. Much of the serum K will be lost in urine. So person has loss of total K from
body; but has hyperkalemia
- Kussmaul respiratin (to compensate for acidosis), dehydration (from diuresis), mental status
change, fruity breath

6. How do you treat DKA?


- Give fluids - to treat dehydration
- Insulin
- Give potassium - because when you give insulin, K will go inside the cells; also serum K goes down
as acidosis is treated

Type 2 DM
7. Explain type 2 DM.

Endocrine Page 4.1


7. Explain type 2 DM.
- End organ insulin resistance
- Risk factor - obesity; Obesity reduces insulin receptors (HY)
- Higher genetic predesposition compared to type 1 DM

8. Describe presentation of type 2 DM.


- Initially, see high insulin. Later on B cells get exhausted so see low insulin
- Histology shows amyloid deposition in islets

Fig - amyloid deposition in Islets in type 2 DM. Amyloid is made of amylin - a protein
produced with insulin.

9. What is presentation and diagnosis of type 2 DM?


- Polyuria, polydipsia, hyperglycemia, often clinically silent
- Diagnosis
○ Random glucose (>200 mg/dl)
○ Fasting glucose (>126 mg/dl)
○ Glucose tolerance test (>200/dl two hours after glucose load)

10. How do you treat type 2 DM?


- Weight loss and exercise first line
- Drugs
- Insulin

11. Describe hyperosmolar non-ketotic coma.


- MOA - High glucose levels (>500 mg/dl) leads to life threatening diuresis
- Don't see ketoacidosis because some insulin is present which prevents lipolysis and fat breakdown
- Hypotension
- Coma (due to osmolar effect in brain)

12. What are two major complications of diabetes? Give examples of diseases.(HY)
- Non enzymatic glycosilation (NEG) of vascular basement membrane
○ NEG of large and medium vessels lead to artherosclerosis - ex - peripheral vascular disease
leading to amputation
○ NEG of small vessel leads to hyaline arteriolosclerosis - ex - diabetic nephropathy (nephrotic
syndrome)
○ NEG of hemoglobin leads to HbA1c - long term marker of glycemic control
- Osmotic damage

Endocrine Page 4.2


- Osmotic damage
○ Some cells in body can take sugars without insulin
 Schwann cells - aldose reductase reduces glucose to sorbitol in Schwann cells. Sorbitol
leads to osmotic damage. Leads to peripheral neuropathy.
 Pericytes of retinal blood vessel - sorbitol cause osmotic damage and pericytes die.
Leads to anurysm of retinal blood vessel. Rupture leads to blindness.
 Lens - sorbitol build up leads to cataract.

13. What are some pancreatic endocrine tumor?


- MEN 1
○ Parathyroid hyperplasia and pituitary adenoma, pancreatic endocrine tumor
- Insulinoma -
○ Tumor makes insulin
○ Presentation-
 Mental status change due to severe hypoglycemia that is relieved by glucose
 Lab - Low glucose, high insulin, high C peptide (C peptide is made along with insulin)
- Gastrinoma
○ Gastrin induces parietal cells in stomach to make acid
○ Presentation
 Treatment resistant peptic ulcers (aka Zollinger Ellison syndrome)
 Ulcers can extend to jejunum
- Somatostatinoma
○ Somatostatin reduces acid production and contraction of gall bladder
○ Presentation
 Achlorhydria (low acid production in stomach)
 Cholelithiasis and steatorrhea (due to reduced bile release)
- VIPoma
○ Vasoactive intensinal peptide greatly stimulates secretion of water and electrolytes in
intestine, reduces gastric acid production.
○ Presentation
 Watery diarrhea
 Hypokalemia
 Achlorhydria

14. What are two proteins that are made together with insulin?
- Amylin - in type 2 DM, we see amyloid buildup due to amylin buildup
- C peptide - can be tested in blood to see if pt is overadminstering insulin

Endocrine Page 4.3


15.10-15.11 Adrenal Cortex and Adrenal Medulla

Crotex - hormones are made from cholesterol (cortex is yellow due to cholesterol)
Medulla- hormones made from tyrosine

1. What are three layers of adrenal?

Zona glomerulosa Mineralocorticoid (Aldosterone)


Zona fasciculata Glucocorticoid (Cortisol)
Zona reticularis Sex steroids
Pneumonic GFR for the zones.

2. Describe metabolism of cortisol.


 Transport
o 75% transported by transcortin (aka corticosteroid binding globulin) (made by liver)
o 15% bound to albumin
o 10% in free form
 Receptor - nuclear receptor. Present in all body cell
 Can bind strongly to mineralocorticoid receptor as mineralocorticoid itself.

3. How do you get hypokalemia from too much licorice ingestion?


 Cortisol is inactivated to cortisone in kidney tubules by 11-B-hydroxysteroid dehydrogenase type 2
(reverse reaction by type 1 dehydrogenase). Licorice inactivates the type 2 enzyme, leading to active
cortisol in kidney tubules.
 As cortisol has high mineralocorticoid action, it leads to potassium excretion in kidney tubules leading to
hypokalemia.

4. What are functions of glucocorticoids (called glucocorticoid because it increases glucose in blood)?
 Increased glucose output by liver and decrease glucose intake by muscle, adipose (to increase glucose
supply to brain and heart)
 Decreased insulin sensitivity
 Decreased immune activity via
o Decreased arachidonic acid production (due to inhibition of phospholipase A2)
o Low IL-2 production (IL2 important for proliferation of T cells)
o Low histamine production
 Increased alpha one receptor production and sensitivity to catecholamines (lack of cortisol causes
extreme vasodialation, extreme hypotension and death)
 Negative feedback to ACTH

5. How does adrenal insufficiency cause hypotension?


- ↓glucocorticoid (GC)  too much vasodialation
- ↓mineralocorticoid (MC) hypovolemia (too much Na and water wastage from kidney )

6. What are clinical features of Cushing’s syndrome (too much GC)?


- Muscle weakness (AA in muscles are used for gluconeogenesis) with thin extremities
- Moon faces, buffalo hump, truncal obesity (high glucose in blood leads to insulin release; insulin is an
anabolic hormone and leads to fat storage)
- Abdominal striae (cortisol impairs collagen synthesis; blood vessels in stomach ruptures leading to striae
formation)
- HTN (increased sensitivity and production of alpha 1 receptor)

Endocrine Page 5.1


- HTN (increased sensitivity and production of alpha 1 receptor)
- Osteoporosis
- Immune suppression

7. What are causes of cushings?


Cause Adrenal gland size
Exogenous cortisol (most common) Both atrophied
Primary adrenal adenoma, hyperplasia, carcinoma (aka cushing's One secreting cortisol big, other
disease - 2nd most common) atrophied
ACTH secreting pituitary adenoma Both large
Paraneoplastic ACTH secretion (ex - small cell carcinoma of lung) Both large

8. How do you distinguish between cushing's due to ACTH made by pituitary vs pituitary made somewhere
esse (paraneoplastic)?
- Do a dexamethasone suppression test. ACTH produced by pituitary will go down but not the
paraneoplastic one.

9. What is presentation of hyperadlosteronism?


- Hypernatremia, HTN
- Hypokalemia, metabolic alkalosis
- Aldosterone causes principal cell of kidney to take Na and excrete K.

10. Describe features of primary hyperaldosteronism.


- Most commonly due to adrenal adenoma. Less common - hyperplasia and carcinoma
- High aldosterone and low renin

11. Describe features of secondary hyperaldosteronism


- Causes- fibromuscular dysplasia (classically seen in young woman where renal artery is stenosed),
atherosclerosis of renal artery
○ As kidney sees low blood flow, RAAS is activated leading to high aldo
- See high renin and high aldo

11. Describe cause of congenital adrenal hyperplasia (occurs in both androgen)


- Most commonly seen due to deficiency of 21- hydroxylase (required to make cortisol and aldo)
- Deficiency of cortisol leads to excess production of ACTH leading to hyperplasia of adrenals (remember
ACTH is a trophic hormone)
- Other causes - 11 hydroxylase and 17 hydroxylase deficiency (leads to low cortisol)

12. What is presentation of congenital adrenal hyperplasia?


- Excess sex steroids (hormone production shunted towards sex hormone production due to deficient 21-
hydroxylase)
o Clitoral enlargement
o Precocious (early) puberty in males
- Life threatening hypotension (cortisol deficiency)
- Hyponatremia, hypovolemia; hyperkalemia, acidosis (aldo deficiency)

13. How does 11 hydroxylase deficiency present in contrast to 21 hydroxylase deficiency?


- In 11 hydroxylase deficiency, we don't see effects of hypomineralocorticoid; but see effect of
hypocortisol and excess sex hormone
- Reason - Both 21 and 11 hydroxylase are required for cortisol production. 21 hydroxylase can produce
weak mineralocorticoid, and 11 hydroxylase is needed to make strong mineralocorticoid.

Endocrine Page 5.2


Adrenal insufficiency
14. What is presentation of Waterhouse-Friderichsen syndrome (acute adrenal insufficiency)?
- Commonly seen in kid with Niserria meningitis infection that causes DIC and then bilateral necrosis of
adrenal glands
- See massive hypotension

Fig - sac of blood adrenals classically seen in waterhouse friderichsen syndrome

15. What are causes of chronic adrenal insufficiency?


- Autoimmune (most common cause in developed world)
- TB (most common cause in developing world)
- Metastatic carcinoma to adrenals (lung cancer loves to go to adrenal)

16. What are presentation of chronic adrenal insufficiency?


- Hypotension (low cortisol)
- Hyponatremia, hypovolemia; hyperkalemia, acidosis (aldo insufficiency)
- Weakness
- Hyperpigmentation (HY) - ACTH is made from POMC. POMC also induces melanin synthesis. High ACTH
means high POMC which leads to high melanin synthesis - classically seen in oral mucosa and skin. - only
see in 10 adrenal insufficiency.
- Vomiting and diarrhea (low cortisol)

17. Describe pheochromocytoma (tumor of adrenal medulla)


- Medulla is made of neural crest derived chromaffin cells (HY)
- Pheo is tumor of chromaffin cell
- Classic finding - brown tumor (because chromaffin cells are brown)

Fig - pheochromocytoma. Yellow parts seen in left and right are adrenal cortex.

18. What are presentation of pheochromocytoma?


- Episodic HTN, headache, palpitation
- Orthostatic hypotension may be seen - because alpha receptors are sensitized to high levels of

Endocrine Page 5.3


- Orthostatic hypotension may be seen - because alpha receptors are sensitized to high levels of
catecholamines
- Diagnosis -
o increased serum metanephrines
o Increased urine metanephrines and VMA
o Epi and NE are metabolized to metanephrine and normetanephrine respectively. MAO converts
both of them to VMA.
- Treatment - surgery (HY - give phenoxybenzamine (irreversible alpha 1 blocker) before surgery because
mechanical stress of adrenal can leak out epi and NE giving pt HTN and too much bleeding).

19. What are rule of 10's involving pheo?


- 10% bilateral
- 10% familial
- 10% malignant
- 10% located outside adrenal (HY - a common site is urinary bladder - classic presentation is a patient
who experiences headaches and palpitation while urinating).

20. What are associations of pheo?


- MEN 2A (MTC, pheo and parathyroid adenoma) and
- 2B (MTC, pheo and mucosuloganglio neuroma esp in oral mucosa). MTC can kill pt so people with MEN
go prophylactic thyroidectomy. Ret oncogene linked to MTC.
- VHL disease (autosomal dominant mutation of von hippel lindau tumor suppressor gene - increased risk
of hemangioblastoma of cerebellum, renal cell carcinoma, pheo)
- NF type 1

21. Why don't you give beta blocker in pheo to control HR?
- B2 is a vasodialator. B1 increases HR. If B blocker is given, we have uncontrolled alpha action (very
severe vasoconstriction).

Endocrine Page 5.4


Chapter 16: Breast Pathology
16.1 Introduction

1. Describe the anatomy and histology of breast.


- Anatomy:
○ Breast can develop anywhere along the milk line (a straight line from vulva to axilla)
○ In females, lobules and ducts are present in highest density in upper lateral part of breast
○ In males, lobules and ducts are present in highest density in subaerolar area
○ Estrogen and progesterone cause hyperplasia of breast lobules
- Histology:
○ Terminal duct and lobular unit is the functional unit of breast. Lobules make milk and ducts
drain them
○ Lobules and ducts are lined by two cell layers:
 Luminal cell layer: columnar epithelial cell that makes milk in lobules
 Myoepithelial cell layer: Outer layer; contracts to expell milk outside

2. What is galactorrhea? What are its causes?


- Galactorrhea is milk production outside pregnancy
- Causes:
○ Nipple stimulation
○ Prolactinoma
○ NOT A SIGN OF CANCER

Breast Page 1.1


16.2 Inflammatory Conditions

1. Describe the following inflammatory conditions of the breast.


Cause Presentation Treatment
Acute - Staph Aureus infection - Erythematous breast - Continue
mastitis - Associated with breast feeding (fissures - Purulet nipple drainage
develop in nipple and bacteria enter) discharge - Dicloxacill
- May have abscess in
Periductal - Usually seen in smokers. Smoking causes - Subareolar mass with
mastitis relative Vit a decifiency. Vit A deficiency - nipple retraction - due
causes squamous metaplasia of periductal to inflammation and
cells. Cells produce keratin and block the fibrosis
duct resulting inflammation
Mammary - Chronic inflammation that causes dilation - Periareolar mass with
duct ectasia (ectasia) of subareolar duct green-brown nipple
(dialation) discharge
- Plasma cells on biopsy
Fat necrosis - Usually related to trauma - Mass on physical exam
or abnormal
calcification on
mammography (due to
saponification)
- Calcification and giant
cells on biopsy

Breast Page 2.1


16.3 Benign Tumors and Fibrocystic Changes

1. Describe the following benign tumor and fibrocystic changes in the breast.
Epidemiology Presentation
Fibrocystic Most common change in premenopausal - Presents as vague irregularity
change women (hormone mediated) - seen in 30-60% (lumpy breast) in upper outer
of women quardant
- Cyst look blue-dome on gross
exam

Fig: fibrocystic change presents as fibrosis of


stromal cells and cystically dilated ducts

Intraductal Classically seen in premenopausal women - Bloody or serous nipple


papilloma discharge
- Usually present in one of the
main lactiferous duct below
areola and may cause nipple
retraction

Fig: intraductal papilloma (mass has both


epithelial and myoepithelial layers)
Fibroadenoma - Most common benign neoplasm of breast - Well circumscribed mobile
(classically seen in premenopausal woman) - marble like mass - move freely
hormone sensitive - mass grows during (contrast to infiltrative ductal

Breast Page 3.1


hormone sensitive - mass grows during (contrast to infiltrative ductal
pregnancy and maybe painful in menstruation carcinoma that's immobile)
cycle - Benign - 1.5-2x increased risk of
- Growth of fibrous part squeezes the lumen of cancer (FA)
duct

Fig: Fibroadenoma (growth of both lobular


and stromal cells)

Fig: gross speciman shows well demarked,


capsulated tumor
Phyllodes Classically seen in postmenopausal woman - Fibroadenoma like tumor but
tumor much larger with overgrowth of
(phyllodes = fibrous part
leaflike) - Leaf like projection on biopsy
(tumor of stromal cell - cells
between lobules)
- Maybe malignant in some cases
- Most common in 5th decade

Fig: fingerlike projection of Phyllodes tumor


(stromal tumor)
- Only phyllodes tumor is mainly seen in postmenopausal woman and maybe malignant. All others
are seen mainly in premenopausal woman and not usually malignant

Breast Page 3.2


2. What are the different types of fibrocystic changes and their associated breast cancer risk?
Breast cancer risk Presentation
Fibrous, cysts and No increased risk
apocrine metaplasia
Ductal hyperplasia and 2x increased risk in Sclerosing = hard (fibrous); adenosis = too
sclerosing adenosis both breasts many glands; calcification maybe seen
Atypical hyperplasia 5x increased risk in Hyperplasia maybe lobular or ductal
both breasts

3. How do you distinguish intraductal papilloma from papillary carcinoma of breast?


Intraductal papilloma Papillary carcinoma
Both present as bloody nipple discharge
More common in premenopausal women As it's cancer, its more common in
postmenopausal women
Papillary growth has both epithelial and Papillary growth has epithelial cells but lacks
myoepithelial cells myoepithelial cells

Breast Page 3.3


16.4 Breast Cancer

1. What are the risk factors for breast cancer?


- Risk factors are associated with estrogen exposure
○ Female gender (female:male = 100:1 for breast cancer incidence)
○ Age - cancer usually seen in postmenopausal woman with exception of hereditary breast cancer
○ Early menarche/late menopause (increases estrogen exposure)
○ Obesity (fat cells converts testosterone to estrogen)
○ Atypical hyperplasia
○ First degree relative with breast cancer
○ Race - AA at more risk
○ BRACA +ve (BRACA 1 = risk of ovarian cancer and triple neg breast cancer; BRACA 2 = breast cancer in males)

2. What are the characters of the following types of breast cancer?

Histology Mass? Remarks?


Ductal - Cell proliferate in duct without invading basement membrane - No mass
carcinoma in - Histologic subtypes present:
situ • Comedo type: high grade cells with necrosis in duct with
calcification

Fig: DCIS mammography


Fig: DCIS - note cellular proliferation, necrosis and centrally located
calcification
Invasive ductal - MOST COMMON INVASIVE CARCINOMA OF BREAST - >80% of cases - Rock hard immobile mass - grossly see
carcinoma - Invasive cancer that produces duct like structure in desmoplastic stroma classic 'stellate' appearance
- Subtypes: - Most common of all breast cancer
• Tubular carcinoma: - Inflammatory has worst prognosis; others
○ has well differentiated ducts without myoepithelial layer in have relatively good; Invasive lobular has
desmoplastic stroma better prognosis than invasive ductal.
○ Good prognosis
• Mucinous carcinoma:
○ ducts in abundant extracellular mucin
○ Good prognosis
• Medullary carcinoma:
○ high grade ductal cells associated with lymphocytes and
plasma cells
○ Increased incidence of BRCA1 carriers
○ Good prognosis
• Inflammatory carcinoma -
○ carcinoma in dermal lymphatics Fig: 'stellate' invasive ductal carcinoma
○ Poor prognosis (tumor already in lymph)
○ Presents as inflamed, swollen breast due to blockage of
lymphatics - orange peel appearance; can be mistaken for
acute mastitis

Breast Page 4.1


Fig: Peau d'orange (orange peel)
appearance of inflammatory carcinoma

Fig: Tubular carcinoma (left); mucinous carcinoma (right)

Fig: medullary carcinoma (left); inflammatory carcinoma (right)


Lobular - Cells proliferate in lobules without invading basement membrane - No mass, no calcification - usually
carcinoma in - Often multifocal and bilateral discovered incidentally
situ - Characterized by dyscohesive cells lacking E-cadherin adhesion protein - Often bilateral
- Treatment:
• Tamoxifen (to reduce risk of carcinoma)
• Follow up closely because it can progress to invasive carcinoma

Invasive lobular - Cells characteristically grows in single file (aka Indian file) and may show - Often bilateral and multiple lesions in
carcinoma signet ring morphology - cells don't make duct because they lack E- same location
cadherin - Better prognosis than invasive ductal
- Usually bilateral carcinoma
- Associated with lobular carcinoma in situ
in 90% of cases
- Has diffuse invasive pattern of spread so
difficult to detect by physical or radiologic
exam

Fig: Small runs of invasive lobular carcinoma (arrows) with two adjacent
foci of LCIS.

3. What are the prognostic factors for breast cancer?


- TNM staging
○ Metastasis is most important prognostic factor but pt present early so not very useful
○ Spread of tumor to axillary lymph nodes (N) is most useful prognostic factor - Sentinel lymph node biopsy used to assess axillary
lymph nodes

4. What is sentinel lymph node biopsy?


- Many years before, doctors use to take out all lymph nodes in axilla to check for spread of breast cancer. In many patients, there was no

Breast Page 4.2


- Many years before, doctors use to take out all lymph nodes in axilla to check for spread of breast cancer. In many patients, there was no
spread and they had to suffer upper extremity edema due to lack of lymph nodes
- Then doctors started to inject dye in tumor and check which lymph nodes in axilla the dye moved to. If the lymph nodes with dyes didn't
had metastasis, the doctors didn't take out all the lymph nodes. If the nodes had metastasis, they would then proceed to take out all lymph
nodes. This process is called sentinel lymph node biopsy.

5. What are the predictive values of breast cancer treatment?


- Imprtant predictive values of treatment are presence of absence of overexpression of estrogen receptor (ER), progesterone receptor (PR),
and HER2/neu receptor. (HER2/neu are receptors in RAS/MAPK pathway).
Overexpression of ER, PR (nuclear receptor) Good response to antiestrogenic agents (ex-
tamoxifen)
Overexpression of Her2/neu receptor (cell surface receptor) Good response to trastuzumab (anti HER2
receptor Ab)
Triple negative receptor (none of above overexpressed) - usually seen in African Poor pharmacological prognosis
American women

6. What is Paget's disease of nipple?


- Extension of ductal carcinoma in situ to lactiferous ducts and skin of nipple producing rash. Paget cells are present.

Fig: Paget disease of nipple


Hereditary breast cancer
Epidemiology 10% of breast cancer cases
Presentation - Seen in premenopausal cancer
- Presence of multiple tumor
- Having multiple first degree relative with breast cancer
Mutations - BRCA1- breast (medullary carcinoma - type of invasive ductal carcinoma) and ovarian carcinoma
(serous carcinoma)
- BRCA2 - breast carcinoma in males
Value of prophylatic bilateral - Decreases risk of carcinoma but not to zero
mastectomy

Male breast cancer


Epidemiology - 1% of all breast cancers
Common type - Invasive ductal carcinoma
Presentation - Subareolar mall in older males (most breast tissue in males is in subareolar area - in females, it's in upper outer
quadrant of breast)
- May have nipple discharge
Genetic - Klinefelter syndrome (XXY)
associations - BRCA2 mutations

Breast Page 4.3


Chapter 17: Central Nervous System
Pathology
17.1 Developmental Anomalies

Neural tube defects


Defn - Incomplete closure of neural tube due to folate deficiency prior to conception. Two types
Anencephaly Spina bifida
- Absence of skull and brain - Failure of posterior vertebral arch to close
- Results in polyhydramnios as fetal - Types:
swallowing of amniotic fluid is impaired a. Spina bifida occulta (asymptomatic)-
- Vertebra is not fused because the plates of spinal arch can't
fuse because neural tube didn't close properly
- Asymptomatic; dimple of patch of hair overlying vertebral
defect

b. Spina bifida - cystic protrusion of underlying tissue


1. Meningocele - meninges protrude
2. Meningomyelocele - meninges and spinal cord protrude

Detecti - Can be detected by elevated alpha-fetoprotein (AFP) in amniotic fluid and maternal blood.
on - AFP most abundant protein in young fetus blood (albumin of fetus) - made by fetal liver and yolk sac.
- AFP elevated in other cases too - hepatocellular carcinoma, liver metastasis, yolk sac tumor, germ cell tumor.
Anatom - Neural plate invaginates early in gestation to make neural tube
y - Walls of neural tube - makes CNS
- Hollow lumen of neural tube - makes ventricles and spinal cord canal
- Neural crest - makes peripheral nervous system

Neurology Page 1.1


Cerebral duct stenosis
Defn - Stenosis of cerebral aqueduct (of sylvius) - connects 3rd and 4th ventricles; leads to
hydrocephalous
- CSF is made by choroid plexus lining the ventricles
Epid - MOST COMMON CAUSE OF HYDROCEPHALOUS IN NEWBORNS

- CSF leaks from 4th ventricle to subarachnoid space via foramen of Magendie
(middle) and foramen of Luschka (lateral)

Dandy walker malformation


Defn - Congenital failure of cerebellar vermis to develop so that 4th ventricle is massively
dialated and cerebellum is absent; often accompanied by hydrocephalous
- Presents as hydrocephalous, increased ICP and motor problem.
- Associated with other abnormalities of CNS and malformation of heart, limbs etc.

Neurology Page 1.2


Arnold-Chiari malformation
Defn - Congenital downward displacement of cerebellar vermis and tonsils through foramen
of magnum, leading to obstructive hydrocephalous
- Often associated with meningomyelocele - cerebellum is pulled downward

Neurology Page 1.3


17.2 Spinal Cord Lesions

Seringomyelia
Defn - Cystic degeneration of spinal cord usually on C8-T1 that affects the spinothalamic
(anteriolateral) pathway and later on affects anterior horn (motor neurons) and
lateral horn (SANS)
- Typically spares dorsal column pathway (fine touch and position)

Presentati - Sensory loss of pain and temperature in upper extremities and 'cape like'
on distribution (anterior white commissure of spinothalamic tract)
- Muscle atrophy and weakness (anterior horn damage)
- Horner's (lateral horn - hypothalamospinal tract damage)

Poliomyelitis
Defn - Anterior horn neurons damage due to poliovirus infection
Presentation - Presentation based on lower motor neuron sign
• Flaccid paralysis with muscle atrophy
• Babinski -ve
• Impaired reflexes

Werding-Hoffman disease
Defn - AR inherited degeneration of anterior motor horn
- Presents as floppy baby and death within a few years after birth

Neurology Page 5.1


Amotrophic lateral sclerosis (ALS)
Defn - Degeneration of both upper and lower motor neuron of corticospinal tract
Cause - Most cases are sporadic
- Some familial cases due to mutation in zinc-copper superoxide dismutase mutation
(SOD1) - causes free radical injury in neurons
Presentati - Lower motor neuron signs:
on • Flaccid paralysis and muscle atrophy
• Fasciculations
• Negative Babinski
- Upper motor neuron signs:
• Spastic paralysis with hyperreflexia
• Increased muscle tone
• +ve Babinski
- Sensory system intact (distinguish from syringomyelia)
- Atrophy and weakness of hand early sign

Fredreich ataxia
Defn - Degenerative disorder of cerebellum and spinal cord due to unstable trinucleotide
repeat (GAA) in frataxin gene (autosomal recessive)
Pathophys - Frataxin gene imp for mitochondrial iron regulation; loss results in iron buildup with
free radical damage
Presentati - Loss of multiple spinal tracts:
on • Loss of vibration and proprioception
• Muscle weakness in lower extremities
• Loss of deep tendon reflex
- Presents in early childhood; pt are wheelchair bound in few years
Associatio - hypertrophic cardiomyopathy
n

Neurology Page 5.2


17.3 Meningitis

Defn - Inflammation of leptomeninges (arachnoid and pia)


Cause Neonates - Group B strep (no 1), E. coli, Listeria, H flu (non-vaccinated
infants)
Teens and adults - N. meningitis, Strep pneumo
Immunecompromise - Fungi (cryptococcus
d
Viral - Most common is coxsackie (feco-oral transmission; kids)

Presentation - Classic triad of headache, fever and nuchal rigidity


- Photophobia and vomiting
- Altered mental status may be present
Diagnosis - Lumbar puncture
• SC ends at L2; put needle between L4 and L5 - level of iliac crest; cauda equina
continues to S2
CSF findings Cells Glucose Protein
Bacteria Neutrophils Low ?
Virus Lymphocytes Normal ?
Fungal Lymphocytes Low ?

Complicatio - Commonly seen in bacterial meningitis


ns • Hydrocephalus, hearing loss and seizures - due to fibrosis
• Death - due to herniation and cerebral edema

Neurology Page 6.1


17.4 Cerebrovascular Disease

Stroke
Epdm - No. 3 cause of death in USA
- Neurons are susceptible to ischemia and undergo necrosis within 3-5 mins
Types

Global cerebral ischemia


Etiologi - Low perfusion (ex- atherosclerosis)
es - Acute decrease in blood flow (ex - cardiogenic shock)
- Chronic hypoxia (ex- anemia)
- Repeated episodes of hypoglycemia (ex- insulinoma)
Present - Presentation based on duration and magnitude of insult
ation

Ischemic stroke (focal cerebral ischemia)


Defn - Regional ischemia that causes focal neurological defects lasting >24 hours
- If symptoms last <24 hours, it's called transient ischemic attack (TIA)
Subtype Thrombotic Embolic Lacunar
s
Cause - Due to rupture of artherosclerotic plaque - Due to emboli from - Due to hyaline
left heart (ex- a-fib) arteriolosclerosis
(complication of HTN
and diabetes)
Location - Mostly at branch points (ex- bifurcation - Usually middle cerebral - Mostly lenticulostriate
of internal carotid and middle cerebral artery vessels
artery at cirlce of Willis)
Presentat - Pale infarct at periphery of cortex - Pin point hemorrhagic - cause small cystic
ion infarct over triangular infraction

Neurology Page 3.1


ion infarct over triangular infraction
area - If internal capsule
involved --> pure motor
stroke
- If thalamus involved -->
pure sensory stroke

Outcom - Liquefactive necrosis


e 12-24 - Eosinophilic change in neurons (red neurons)
hours - Necrosis after 24 hours

1-3 days - Neutrophil infiltration


3-7 days - Microglial infiltration
2-3 weeks - Gliosis (reactive astrocytes line space)

HEMORRHAGIC STROKE
Intracerebral hemorrhage
Defn - Bleeding in brain parenchyma - classically due to rupture of Charcot-Bouchard micro-aneurysm of
lenticulostriate vessels in Basal ganglia (contrast to ischemic lacunar stroke of lenticulostriate vessels due
to arteriolosclerosis)

Present - Headache, nausea, vomiting, and eventual coma


ation
Preventi - HTN treatment will reduce incidence by half

Neurology Page 3.2


Preventi - HTN treatment will reduce incidence by half
on

Subarachnoid hemorrhage
Defn - Bleeding to subarachnoid space (blood vessels are outside pia but inside the arachnoid layer)

Presentation - Worst headache of life (aka thundercloud headache) with nuchal rigidity
Cause - Rupture of berry aneurysm (85% of cases)
• Most commonly located in branch point of anterior communicating artery
• Aneurysm lacks media layer
- AV malformation
- Anticoagulated state
Berry aneurysm - Marfan syndrome
association - Dominant polycystic kidney disease

Neurology Page 3.3


17.5 Trauma

Subdural and epidural hematoma


Epidural hematoma Subdural hematoma
Defn - Collection of blood between dura - Collection of blood between below dura
and skull
Cause - Classically due to rupture of - Due to tearing of bridging veins that lie
middle meningeal artery from between dura and arachnoid - commonly in
trauma to temporal bone elderly (have atrophied brain) with trauma
Presentati - Lucid interval may precede - Progressive neuro sign (takes months for
on neurologic sign (so disease called bleeding to accumulate)
talk and die disease)
Imaging - Lens shaped lesion on CT - Crescent shaped lesion on CT

Fig: red arrow- acute; blue arrow - chronic


hemorrhage
Cause of - Herniation - Herniation
death

Herniation (displacement of brain tissue due to mass issue of increased ICP)


Tonsilar herniation Subfalcine herniation Uncal herniation
Defn - Cerebellar tonsils - Cingulate gyrus - Temporal lobe uncus herniates
herniates under herniates under falx under tentorium cerebeli
foramen magnum cerebri
Presenta - Compression of - Compression of - CN3 palsy - eye moving down and
tion brainstem can lead to anterior cerebral out and dialated pupil
cardiopulmonary artery leads to - Posterior cerebral artery
arrest infraction compresion - infraction of posterior
cerebral arery (contralateral
homonymous hemianopsia)
- Rupture of paramedian artery -
Duret (brainstem) hemorrhage

Neurology Page 7.1


17.6 Demyelinating Disorders

Defn - Disease where myelin is damaged; axons are generally preserved


Myelin types - Oligodendrocytes - myelinate CNS; single cell myelinate multiple
neurons
- Schwann cells - myelinate PNS; single cell myelinate single neuron

Leukodystrophy
Defn - Inherited mutation in enzymes necessary to produce or maintain
myelin
Metachromatic - MOST COMMON LEUKODYSTROPHY
leukodystrophy - Deficiency of arylsulfatase. Sulfatides can't be degraded and
accumulate in lysosome of oligodendrocytes
- Autosomal recessive
Krabbe disease - Deficiency in galactocerebrosidase. Galactocerebroside accumulates
in m
- Autosomal recessive
Adrenoleukody - Impaired addition of coenzyme A to long-chain fatty acids (X-linked
strophy defect)
- Accumulation of fatty acids damages adrenal glands and white
matter of brain

Multiple sclerosis
Defn - Autoimmune destruction of CNS myelin and oligodendrocytes
Epid - Most common chronic CNS disease of young adults (20-30 years)
- More common in women and who live far away from equator
Genetics - Associated with HLA-DR2
Diagnosis - MRI reveals white matter demyelination
- Lumbar puncture:
• Increased lymphocytes and immunoglobulin
• oligoclonal IgG bands on high resolution electrophoresis
• Myelin basic protein

Neurology Page 8.1


Presentation - Relapsing and remitting neurological features (comes and goes)
• Vision:
○ Optic nerve damage - blurred vision in one eye
○ MLF damage - internuclear opthalmoplegia
• Hearing:
○ Vertigo and scanning speech (like drunk)
• ANS:
○ Bowel, bladder, and sexual dysfunction
• Spinal cord:
○ Lower extremity loss of sensation and weakness
• Cerebral white matter:
○ Hemiparesis or unilateral loss of sensation
Treatment - Acute attack with high dose steroids
- Long term treatment with interferon beta slows disease progression

Subacute sclerosing panencephalitis (Dawson disease)


Presentati - Progressive, debilitating encephalitis that leads to death caused by
on persistent infection of brain by measles virus
- Primary infection occurs in infancy; neuro signs arise 6-15 years later
- Initially presents as dementia, personality alterations, and loss of
movement control. Slowly progresses to speech loss of speech, loss of
ability to walk and dysphagia. Then, pt will be blind, mute, loss bodily
functions and be in vegetative stage and/or comatose and evenually die.
- Viral inclusions in neurons (gray matter) and oligodendrocytes (white
matter)

Progressive multifocal leukoencephalopathy


Defn - Rapidly progressive neurologic signs (vision loss, weakness, dementia and
eventually death in few months) caused due to JC virus damage of
oligodendrocytes (mainly during immunesuppression - AIDS, Rituximab, or
leukemia)
Imaging

Fig: multifocal white mater damages seen on T2 MRI (water is bright)

Neurology Page 8.2


Central pontine myelinolysis
Defn - Focal demyelination of pons due to rapid correction of hyponatremia
- Increasing Na concn too fast from low to high - makes your pons die
- From high to low - makes your brain blow
Presentation - Acute bilateral paralysis (locked in syndrome)

Neurology Page 8.3


17.7 Dementia and Degenerative Disorders

1. What are some basic findings in neurodegenerative diseases?


• Characterized by loss of neurons in gray matter
• Accumulation of intra and/or extracellular proteins
• Cortex degeneration: dementia; brainstem and basal ganglia degeneration: movement disorders
• Increased incidence with age

Alzhimers
2. What is clinical presentation of Alzhimer's (no. 1 cause of dementia)?
• Begins with short term memory loss and then long term memory loss
• Loss of motor skills and language
• Change in behavior and personality
• Pt become mute and bed ridden; infection is common cause of death
• Focal neurologic defect in late stage

3. What are the sporadic and familial etiologies of Alzhimers?


Mutation
Sporadic (95% of cases) • Seen in elderly • APO E epsilon 4 allele = increased risk
• APO E epsilon 2 allele = decreased risk
Familial cases (5% of cases) • Early onset • Presenilin 1 and 2 mutation
• Down's syndrome (dementia seen around 40)
4. What are gross and microscopic findings in alzhimers?
• Gross:
○ Cerebral atrophy and dilation of ventricles
• Microscopic
○ extracellular AB amyloid deposits
▪ Neuritic plaques ( A-Beta amyloid deposits with entangled neuritic process) (neuritic
process - dendrites and axon)
▪ cerebral amyloid angiopathy - A-Beta amyloid plaques deposit around vessels -
increases risk of hemorrhage
○ Intracellular neurofibrillary tangles (tuft of hyperphosphorylated tau protein): tau is
microtubule associated protein
○ Loss of cholinergic neurons in nucleus basalis of Myenert

Fig: Neuritic plaques (left) and neurofibrillary tangles (tau tangle) on right

5. How are A-beta (AB) amyloid plaques formed?


• AB amyloid is made from amyloid precursor protein (APP) encoded by chromosome 21. If APP
undergoes beta cleavage instead of normal alpha cleavage, AB amyloid is fomed.
• AB amyloid formation increases due to mutation in APP or trisomy 21.

Neurology Page 4.1


• AB amyloid formation increases due to mutation in APP or trisomy 21.

6. How is alzhimer's diagnosed?


• Clinically: diagnosis of exclusion
• Histology at autopsy or biopsy

Vascular dementia
7. Describe vascular dementia?
• 2nd most common cause of dementia
• Occurs due to multifocal white matter infraction
• Associated with HTN, artherosclerosis and vasculitis (they decrease blood flow to brain resulting in
infraction)

Frontotemporal dementia (pick disease)


8. What is clinical presentation of frontotemporal dementia (Pick disease)?
• Early language (termporal) and behavioral (frontal) difficulties which progresses to dementia
• Hallucination
• REM disturbances
• Personality change
• Usually seen in 40-50 year old (can be confused with midlife crisis)

9. What are biopsy findings of Pick disease?


• Round aggregate of Tau protein (compare to neurofibrillary tangle of Alzhimer's which is
triangular shaped)

Parkinson's and dementia with lewy bodies


10. What are biopsy finding of parkinson's disease?
• Loss of dopaminergic neurons in substantia nigra (part of basal ganglia) (compared to losing
neurons everywhere in Alzhimer's) (damage of negrostriatal pathway - used dopamine to initiate
movement)
• Lewy bodies (intracellular round, eosinophilic inclusions of alpha-synuclein) in affected neurons

Fig: Lewy bodies composed of alpha-synuclein

11. What are clinical presentation of Parkinsons?


• TRAP with dementia in late stage:
• Tremor (pill rolling tremor at rest that disappears with motion)
• Rigidity (cogwheel in extremities)
• Akinesia/bradykinesia (expressionless face:hypomimia, slowing of voluntary movement)

Neurology Page 4.2


• Akinesia/bradykinesia (expressionless face:hypomimia, slowing of voluntary movement)
• Postural instability and shuffling gait

11.5 Describe the pathophysiology of Parkinson's.


(add picture from karen's book)

12. What chemical exposure is associated with parkinsons?


• MPTP (a chemical used to cut illegal drugs)
• Most commonly, parkinson's is idiopathic.

13. What is biopsy and presentation of dementia with lewy bodies?


• Biopsy: presence of lewy bodies in cortex (parkinson's has lewy bodies in substantia nigra)
• Presentation: parkinsonian features, hallucinations
• Think dementia with lewy bodies as parkinson's disease with early onset dementia (within 1 year).

Huntingtons disease
14. What is genetic features of Huntington's disease?
• Autosomal dominant disorder (chro 4) with expanded trinucleotide repeat (CAG - glutamate) in
huntington gene
• Damage to GABAergic neurons in caudate nucleus (GABA = inhibitory)
• See anticipation (expansion of repeat during spermatogenesis causes this) - early onset of disease
with each generation (anticipation)

Fig: C is caudate (makes head that ends at lateral ventricle floor), P is putame. C+P = striatum
(called striatum because there's stripe running in between C and P).

Neurology Page 4.3


Fig: Cross section of huntington's pt brain showing small caudate that leads to hydrocephalus ex
vaculo (hydrocephalous due to loss of brain mass)

15. What is presentation of huntington's disease?


• Average age of presentation: 40
• Presents with chorea and progresses to dementia and depression

16. What is common cause of death in huntington's disease?


• Suicide

Normal pressure hydrocephalous


17. What is presentation of normal pressure hydrocephalous? How can you improve symptoms?
• Can causes dementia (idiopathically)
• Increased CSF leading to dilation of ventricles - causes stretching of corona radiata (nerve fibers
runing at side of ventricles).
• 3 W's caused due to stretching of corona radiata-
○ wet (urinary incontinence)
○ Wobbly (gait disturbance)
○ Wacky (personality changes)
• Symptoms improve with lumbar puncture. Treatment is VP shunt (ventricle-peritoneum shunt)

Spongiform encephalopathy
18. What is pathophysiology of spongiform encephalopathy?
• PrPc is normal CNS protein which is in alpha helical structure. It's converted to PrPsc which is in
beta pleated shape.
• PrPsc is congo red +ve (amyloid).
• PrPsc is resistant to damage by autoclave or sterilization or by proteases. Also, it can convert other
PrPc to PrPsc.

19. What are biopsy findings in spongiform encephalopathy?


• Intracellular vacuoles (called spongiform changes)

Neurology Page 4.4


Fig: Spongiform changes (intracellular vacuoles)

19.5. What are etiologies of spongiform encephalitis?


• Sporadic (idiopathically)
• Inherited - ex - fatal familial insomnia
• Transmitted -ex - transplant of cornea

20. Describe etiology and presentation of Creutzfeldt-Jakob disease (CJD)?


• Most common spongiform encephalopathy
• Etiology: mostly sporadic. Rare: contact with prion-infected human tissue (human growth
hormone, cornea transplant)
• Presentation:
○ Rapidly progressing dementia (weeks-months)
○ Ataxia (cerebellar changes)
○ startle myoclonus (involuntary muscle contraction with minimum stimulation)
○ Periodic sharp waves on EEG (HY)
○ Death usually <1 year

21. What is familial fatal insomnia?


• An example of spongiform encephalopathy with severe insomnia and exaggerated startle
response.
• Occurs via inheritance of mutation

22. What is variant CJD?


• CJD that arises in exposure to bovine spongiform encephalopathy (mad cow)
• Seen in younger people
• Other stuff is exactly similar to CJD

Neurology Page 4.5


17.8 CNS Tumors

Adult (supratentorial) Kid (infratentorial)


Astrocytes - Glioblastoma (M) - most common malignant Pilocytic astrocytoma (B) - most common
tumor in adults 10 brain tumor in kids
Oligodendrocytes - Oligodendroglioma (M) -
Ependymal cells (line - Ependymoma (M)
ventricles)
Neurons - Medulloblastoma (M) (neural ectoderm
tissue)
Meninges - Meningioma (B) (females mainly) - most common
10 brain tumor in adults
- Schwannoma (B) - Craniopharyngeoma (B)
B = benign; M = malignant

Adult tumors
1. Glioblastoma multiforme (GBM)
Defn - Malignant tumor of astrocytes
Epid. - Most common primary malignant brain tumor in adults
Presentatio - Often crosses corpus callosum (so called butterfly glioma)
n - Histoogy shows necrotic center with pseudopallisading tumor cells and (pallisade - wall).
Psuedopallisade = fuzzy wall (kiwi seed look like)
- Tumor cells are GFAP +ve (stains for glial cells)

Fig: butterfly glioma (left); pseudopallisading tumor cells (middle); GFAP stain on right
Prognosis - Poor prognosis; one of rare brain tumors to metastasize easily

2. Oligodendroglioma
Defn - Malignant tumor of oligodendrocytes
Presentation - Calcified tumor often involving frontal lobe
- May present as seizures
- Histology shows 'fired egg' appearance of tumor cells

Neurology Page 2.1


3. Meningioma
Defn - Benign tumor of arachnoid cells
Epid. - Most common 10 brain tumor in adults
- More common in women because tumor expresses estrogen receptors
Presentation - Histology:
• Whorled pattern of cells
• Psammoma bodies may be present
- Seen as round mass attached to dura on imaging - can cause seizure by compression

Fig: psammoma bodies (left); cellular whorls (middle); gross (right)

4. Schwannoma
Defn - Benign tumor of schwann cells that involves cranial or spinal nerves
Cause - If bilateral, associated with neurofibromatosis type 2
Features - If present within the cranium, most frequently involves CN 8 (presents as loss of hearing and tinnitus)
- Tumor cells S-100 +ve

Fig: schwannoma of median nerve (right)

Neurology Page 2.2


Fig: schwannoma of median nerve (right)

Kids tumor

1. Pilocytic astrocytoma
Defn - Benign tumor of astrocytes
Epid. - Most common 10 brain tumor in kids
Presentati - Imaging: cystic lesion with mural nodule in cerebellum
on - Histology: Rosenthal fibers (thick eosinophilic corkscrew process of astrocytes) and eosinophilic
granular bodies; GFAP +ve tumor cells

2. Ependymoma
Defn - Malignant tumor of ependymal cells usually seen in 4th ventricle
Presentati - Biopsy:
on • perivascular pseudorosette is characteristic (pseudorosette because the central structure isn’t
part of the tumor. Originally, pseudorosette meant any rosette that didn’t have a truly empty
lumen)
• True ependymal rosette - rosette with central lumen as tumor's attempt to make CSF
- Tumor can result in hydorcephalous

Neurology Page 2.3


Fig: bottom right- perivascular pseudorosette; top left: true ependymal rosette .

3. Medullobastoma
Defn - Malignant tumor of neuroectoderm cells of cerebellum (granular cells)
Presentation - Histology:
• Homer-Wright rosettes may be present (type of true rosette)
• Small, round, blue cells on biopsy

Prognosis - Poor; tumor rapidly grows and spreads via CSF giving 'drop metastasis in cauda equina

4. Craniopharyngioma
Defn - Benign tumor that arises from epidermal remnant of Rathke's pouch (makes parts of pituitary
gland)
Presentation - Supratentorial mass in child or young adult (only supratentorial kid tumor we study)
- Can cause bitemporal hemianopsia due to compression of optic chiasm
- Calcifications can be seen - derived from 'teeth like' tissue

Neurology Page 2.4


Calcifications can be seen - derived from 'teeth like' tissue

Fig: calcified craniopharyngioma


Prognosis - Tends to recur after resection

Malignant brain tumor


Epidem. - Represent 50% of brain tumors
Source - Most common are breast, lung and kidney (BLacK mnemonic)
Appearance - Multiple and well circumcised at gray-white junction

Fig: metastatic melanoma; note the dark color tumors

Neurology Page 2.5


Virtigo

virtigo - room spinning around or you spinning in the room

CNS vertigo : originates in CNS


- no hearing loss
- vertical nystagmus
- focal neuro deficit
- associated CNS symptom (difficulty walking)

peripheral virtigo: originates in ears


- hearing loss
- no vertical nystagmus
- extremely nauseous (CNS vertigo has less nausea)
- head movement can trigger (wake up midnight because they moved their head)
- BPPV

Ddx for peripheral virtigo:


- ex - Meniere disease (vertigo that lasts 20 minutes)
- labyrinthitis (days
- ototoxicity

Ddx for central virtigo:


- MS
- tumor

age old male with PMH of ..... presents with .... most concerning with...

Neurology Page 9.1


Chapter 18: Musculoskeletal
Pathology
18.1 Skeletal System

Achondroplasia (without cartilage formation)


Defn - Impaired cartilage proliferation in growth plate that leads to dwarfism
Epidemiolog - Most common cause of short limbed dwarfism
y
Cause - Activating mutation in fibroblast growth factor receptor 3 (FGFR3) (autosomal d.)
- Most cases are sporadic (80%) and related to increased parental age
Presen. - Presentation based on impaired endochondral ossification; intramembranous
bone formation not affected
- Short extremities with normal sized head and chest
- Mental function, life span, and fertility not affected

Osteogenesis imprefecta (brittle bone disease)


Defn - Mutation in collagen type 1 leads to congenital bone defects
Presen. - Multiple bone fractures (can mimic child abuse)
- Blue sclera - due to thinning of scleral collagen revealing underlying choroidal veins
- Hearing loss - due to fracture of middle ear bones
- Dental imperfection due to lack of dentin (blue or brown teeth that easily rots)

Osteopetrosis (bone-that-is-like-stone)
Defn - Poor osteoclast function leading to abnormally thick and heavy bone that fractures

Musculoskeletal Page 1.1


Defn - Poor osteoclast function leading to abnormally thick and heavy bone that fractures
easily like a piece of chalk
Pathophy - Multiple pathways but one is mutation in carbonic anhydrase II that leads to loss of
s. acidic microenviroment required for bone reabsorption
Presen. - Bone fractures
- Anemia, thrombocytopenia, leukopenia with extramedullary hematopoiesis - due to
bony replacement of marrow (myelophthisic process)
- Hepatosplenomegaly - extramedullary hematopoiesis
- Vision and hearing impairment - due to impingement of cranial nerves
- Hydrocephalous - due to narrowing of foramen magnum
- Renal tubular acidosis - seen if carbonic anhydrase mutation present; CA important
for tubular reabsorption of HCO3- in tubules

Fig: osteopetrosis (left), normal head x-ray (right)


Treatmen - Bone marrow transplant
t

Rickets/Osteomalacia (kids vs adult form of disease)


Defn - Poor mineralization of osteoid (bone matrix produced by osteoblast) with Ca and
PO4 due to Vit D deficiency
Vit D - Derived from sunlight in skin - 85%, diet - 15%
biochem - Vit D2 (ergocalciferol)- ingested from plants; Vit D3 (cholecalciferol) - ingested from
milk or converted from 7-dehydrocholesterol by UV in skin
- Activation requires 25 hydroxylase activity in liver (becomes 25-OH-vit D) followed
by 1-alpha-hydroxylase activity in PCT of kidney (becomes 1,25-OH-vit D - active
form).
Vit D - Increase absorption of Ca and PO4 from kidney and intestine
function - Increase reabsorption of Ca and PO4 from bone
Picmonic

Musculoskeletal Page 1.2


Picmonic

Vitamin D = viking daisy


1st step of activation = 25 hydroxylase action in liver = liver giving 25 cents
2nd step of activation = 1 alpha hydroxylase action in kidney = kidney giving cent
Vit D - Liver or kidney failure
deficiency - Poor diet, lack of sun exposure
causes
Rickets - Pectus craniatum (pigeon breast)
presentati - Frontal bossing (enlarged forehead)
on - Rachitic rosary - osteoid deposition in costochondral junction
- Harrison sulcus - strong diaphragm pull the last ribs inward
- Bowing of legs

Osteomala - Increased risk for fracture


cia - Labs:
presentati • Low serum calcium and phosphate
on • High serum PTH
• High alkaline phosphatase (creates alkaline environment that is required for
osteoblast function)

Osteoporosis
Defn - Reduction of bone density due to loss of trabecular (spongy) bone that increases risk
of fracture

Musculoskeletal Page 1.3


-

Risk - Risk based on peak bone mass (attained ~30 years) and rate of bone loss after that
- Peak bone mass depends on genetics (Vit D receptor variation), diet and exercise
- Rate of bone loss is <1% per year after that. Bone loss increases due to lack of
exercise (space travel), poor diet and lack of estrogen
Clinic. - Reduced bone density on DEXA scan
features - Normal serum Ca, PO4, PTH - distinguish from osteomalacia (presentation of both
may be similar otherwise)
- Fracture in weight bearing bones - vertebra (ht loss and kyphosis), hip and distal
radius
Treatmen - Bisphosphonates - induce apoptosis of osteoclast
t - Exercise, Vit D, and Ca
- Estrogen replacement therapy debated (increases risk of breast and endometrium
cancer)
- Glucocorticoid speeds up bone loss

Paget disease of bone


Defn - thick, sclerotic bone that fractures easily due to imbalance between osteoblast and
osteoclast activity; first osteoclast overactive, then both equal activity, then
osteoblast overactive.
Epid - Mostly seen in >60 year olds (most common cause of isolated increase in alk
phosphate in >40 year olds)
Etiology - Unknown, maybe viral
Clinic - Skull is commonly affected
features • Hearing and vision loss - impengement of cranial nerves
• Lion like face and increased hat size
- Bone pain - due to micro fractures
- Biopsy - mosaic pattern of lamellar bone

Treatment - Bisphosphonate - induce apoptosis of osteoclasts


- Calcitonin - inhibit osteoclast function (to tone down blood calcium)

Musculoskeletal Page 1.4


- Calcitonin - inhibit osteoclast function (to tone down blood calcium)
Complicati - Risk of osteosarcoma during osteoblast overactivity phase
on - High output cardiac failure due to formation of AV shunt in bone

Osteomyelitis
Defn - Infection of marrow and bone
Types - Transient bacteremia - seen in kids - affects metaphysis
- Open wound bacteremia - seen in adults - affects epiphysis
Causes Most common cause (90%) - Staph aurues
Sexually active young adult - Neisseria gonorrhoeae
Sickle cell disease - Salmonella
Diabetics and IV drug users - Pseudomonas
Dog or cat bite/scratch - Pasteurella
Vertebral osteomyelitis (Pott disease) - Mycobacterium TB

Present. - Bone pain with systemic sign of infection (fever, leukocytosis)


- X-ray - shows lytic focus (abscess) - aka sequestrum, with surrounding sclerosis - aka
involucrum

Diagnosis - Blood culture


Treatmen - Antibiotics
t

Avascular necrosis (aseptic necrosis)


Defn - Ischemic necrosis of bone and bone marrow
Causes - Fracture (most common) - ex: avascular necrosis of femoral head
- Steroids
- Sickle cell anemia - vasoocclusive crisis n bone of hands and feet (presents as
dactylitis)
- Caisson disease (decompression sickness) - gas bubbles form in blood with
depressurization
Complicatio - Osteoarthritis and fracture
n

Musculoskeletal Page 1.5


18.2 Bone Tumors

Benign tumors
Osteoma - benign tumor most commonly seen on face
- Associated with garderner's synrome (osteoma + FAP (familial adenomatous polyposis) +
retroperitoneal fibrosis)

Chondroma - Divided as enchondroma (seen on medulla, mainly on small bones of hand and feet) or juxtacortical
chondroma (seen on bone surface)

Fig: enchondroma (left); juxtacortical chondroma (right)


Osteoid - Benign tumor of osteoblast with osteoid core (radiolucent) most commonly seen on diaphysis
osteoma - Size <2cm
- Localized pain, most severe at night, is an almost universal complaint
- Pain is relieved by asprin

Fig: osteoid osteoma (left); osteoblastoma (middle); oxteochondroma (right)


Osteoblasto - Similar to Osteoid osteoma
ma - Differences:
• Most commonly seen on vertebra
• Size >2cm

Musculoskeletal Page 2.1


• Size >2cm
• Pain not relieved by asprin
Osteochond - Most common benign tumor of bone
roma - Tumor of bone with overlying cartilage cap; arises from lateral part of metaphysis and bone is
contineous with marrow space
- Overlying cartilage can rarely transform to chondrosarcoma

Giant cell - Biopsy shows multinucleated giant cells and sromal cells
tumor - Soap bubble lesions seen most commonly in epiphysis of long bones - mostly distal femur or proximal
tibia
- Locally aggressive and may recur

Malignant tumors
Osteosarco Defn - Malignant tumor of osteoblast that has two peak incidence - most common in teenagers,
ma then in elderly
Risk - Familial retinoblastoma
factors - Paget disease (during the osteolbastic hyperactivity phase)
- Radiation exposure
Presentati - Occurs mostly in metaphysis of long bones (distal femur or proximal tibia mainly)
on - Imaging shows destructive mass with 'sunbrust' appearance due to periosteal reaction
and lifting of periosteum by adjacent tumor(codman triangle)

Musculoskeletal Page 2.2


Fig: sunburst appearance (left) and codman triangle (right)
Biopsy - Pleomorphic cells that produce osteoid

Chondrosa - Malignant tumor of cartilage seen in medulla that can grow out through cortex to make sessile
rcoma paracortical mass
- Seen mostly in central skeleton or pelvis

Ewing Defn - Malignant tumor of poorly differentiated neuroectodermal cells seen most commonly in
sarcoma diaphysis of males <15 years
Imaging - Onion skin appearance on X-ray
- Tumor arises in medullary cavity and comes outside the cortex to make soft tan-white
mass
Biopsy - Small round blue cells that resemble lymphocytes (can be confused with lymphoma or
chronic osteomyelitis) (presence of homer wright rosettes indicate neural differentiation)

Cause - Often associated with 11;22 translocation (ewing sarcoma gene (EWS) on ch 22 merges
with transcripton factor gene; precise pathophys unknown)

Musculoskeletal Page 2.3


Prognos - Often metastasizes but responsive to chemotherapy
i

-
Malignant - Most common malignant bone tumors
tumors - Classic site of origin (lead kettle - PB KTL) - Prostate>breast>kidney>thyroid>lung
- Usually causes osteolytic (punched out) lesion except prostate that cause oseoblastic (sclerotic) leison

Musculoskeletal Page 2.4


18.3 Joint

DJD RA Spondyloarthropathy
+ve Rheumatic factor Negative rheumatic factor
Affects both DIP and PIP Affects mainly PIP symmetrically Affects mainly DIP (sausage
(heubordin-bouchard nodes) fingers)- for psoriatic only
Pain worsens during day Pain gets better during day
Associated with HLA-DR4 Associated with HLA-B27
Degenerative joint disease
Defn - progressive degeneration of articular cartilage
- aka non-inflammatory arthritis (no redness or swelling in joints). Synovial fluid accumulates but
lymphocyte infiltrate are not seen. Lymphocyte infiltrate are hallmark in inflammatory
arthritis - gout and RA.
Risk factors - Obesity
- Age
- Trauma
Clinical ○ Commonly affects hip, lower lumbar, DIP and PIP joints
features ○ Symptoms worsen during day (HY)
Pathologic ○ Eburnation (bone rub) of subchondral bone
features ○ Pieces of bone might fall off in joint space (aka joint mice)
○ Enlarged DIP and PIP joints (heubordin-bouchard nodes) due to osteophyte formation (bone
spurs) (HY)

Fig: from left to right - characterstic fibrillation of articular cartilage; 1=bone eburnation, 2
=subchondral cyst, 3 = normal articular cartilate; joint mice; osteophytes

Fig - osteophytes leading to heubordin-bouchard notes on the left x-ray. Right is normal hand x-
ray

Rheumatoid arthritis
Defn - chronic systemic autoimmune joint disease that causes symmetrical polyarthritis and mainly

Musculoskeletal Page 3.1


Defn - chronic systemic autoimmune joint disease that causes symmetrical polyarthritis and mainly
affects PIP and MCP joints
- Classically seen in middle age woman
Genetics - Associated with HLA-DR4
Pathogene ○ Synovitis leading to formation of pannus (granulation tissue)
sis ▪ Contraction of pannus by myofibroblast can lead to
• ankylosis,
• joint distortion,
• damage of cartilage,
• osteopenia (wearing away of bone due to inflammatory processes)
Presentati - morning stiffness that improves with day
on - Symmetric PIP involvement
- DIP spared (HY) - both DIP and PIP involved in DJD.
- Fever, malaise, myalgia, wt loss (systemic autoimmune signs)
- Others:
- Rheumatoid nodules
- Vasculitis
- Baker cyst (swollen bursa behind knee)
- Pleural effusion, LAD, interstitial lung disease
Labs (HY) - Positive rheumatoid factor (IgM against Fc of IgG)
- Presence of neutrophils and protein in synovial fluid.
Complicaio - Anaemia -
ns ○ chronic inflammatory state produces hepsydin (acute phase protein). Hepsydin blocks ability
to use iron that's stored in macrophage. Results in anemia
- Secondary amyloidosis (liver makes acute phase protein SAA that gets converted to AA amyloid
deposition)

Seronegative Spondyloarthropathy
7. What are clinical features of spondyloarthropathy? Explain its types.
- Negative rheumatoid factor (aka seronegative)
- Affects axial skeleton
- Associated with HLA-B27
- Types
○ Ankylosing spondyloarthritis
 Seen in young adult males
 Presents with sacroiliitis and ankylosis of spine (bamboospine)
 Uveitis - eye inflammation
 Aortitis (weak aorta can lead to aortic regurgitation)
○ Reiter syndrome (reactive arthritis)
 Can't see (conjuctivitis)
 Can't pee (urethritis - pains with urination)
 Can't climb tree (arthritis)
 Usually seen in young males a week after GI or C trachomatis infection.
○ Psoriatic arthritis
 Affects 10% of psoriatic patients
 Affects axial and peripheral joints
 Affects DIP of hands and feet (leads to sausage finger and toes) (HY)

○ Infectious arthritis
 Most commonly seen with Neisseria gonorrhea (young adults)
 Staph aureus (old people or kids).
 Pt usually have only one joint affected - only the knee

Musculoskeletal Page 3.2


 Pt usually have only one joint affected - only the knee
 Pt have systemic signs like fever, increased WBC and elevated ESR
 Can be easily treated with antibiotics
Gout
8. What causes gout?
- Caused due to deposition of MSU (mono sodium urate) crystals in joint.
- Crystals arise due to hyperuricemia (uric acid is nucleotide breakdown product)
○ Occurs either by too much uric acid in blood
○ Or not enough filtration by kidney (more common cause)

9. What are some etiologies of hyperuricemia?


Overproduction Underexcretion (more common)
Psoriasis (increased skin turnover) Renal insufficiency
Strenous exercise Competition for excretion by
- Drugs (thiazides, salicylate)
- EtOH
- Lactic acid
- Ketosis
- Glycogen storage disease

10. What is presentation of acute gout?


- Most commonly seen as highly inflammatory monoarticular arthritis in great toe (podegra)
- Acute inflammation (crystals activate macrophage)

9. What are secondary causes of gout?


- Lukemia and Myeloproliferative Disorder - lots of cell turnover
- Lesch-Nyhan syndrome -
○ breakdown product of purine (xanthine and hypoxanthine) can be recycled. Patients who lack
hypoxanthine guanine phosphoribosyl transferase (HGPRT) (enzyme that recycles these breakdown
products) have high uric acid in blood. Uric acid is end product of purine breakdown.
- Renal insufficiency

11. Why do alcohol and meat exacerbate acute gout?


- EtOH compete with uric acid excretion in kidney.
- Meat has lots of DNA/RNA and the purine metabolism increases uric acid.

12. What is presentation of chronic gout?


- Tophi - UA crystals in soft tissue and joints
- Renal failure -
○ UA deposition in tubules
○ See white chalky deposits in kidney tubules with pink fibrosis
- Lab-
○ Hyperuricemia
○ Needle shaped crystals without birefringence with plane light in synovial fluid

13. What is pseudogout?


- It resembles gout but it's deposition of calcium pyrophosphate (not MSU)
- Knee is to pseudogout (some involvement of wrist) as big toe is to gout.
- Synovial fluid has weak positive birefringence under polarized light

14. What constitutes a positive birefringence?


- Crystals that are horizontal are yellow (low crystals are yellow)

Musculoskeletal Page 3.3


15. What lubricating substance is secreted by synovium?
- Synovial fluid (rich in hyaluronic acid)

Musculoskeletal Page 3.4


18.4-18.5 Muscle and Neuromuscular Junction

Dearmatomyositis vs polymysosits
Dermatomyositis Polymyositis
Defn - Inflammation of skin and skeletal muscle - Inflammation of skeletal muscle only
Cause - Unknown; some associated with carcinoma
Presentati - Bilateral proximal muscle weakness; distal - Other than skin involvement, mimics
on muscles can be affected late dermatomyositis (bilateral proximal muscle
- Rash on upper eyelids (heliotrope rash); malar weakness
rash
- Red papules on elbows, knuckles and knees
(Gottron papules)

Lab - Increased creatine kinase


- +ve ANA and anti-Jo-1 Ab
Biopsy - Perimysial inflammation (CD4+) with - Endomysial inflammation (CD8+) with necrotic
perifascicular atrophy on biopsy muscle fibers

X-linked muscular dystrophy


Defn - Degenerative disease with muscle wasting due to bad dystrophin protein and replacement of
muscle by fat
Pathophy - Dystrophin protein is important in anchoring muscle cytoskeleton to extracellular matrix
s - Most mutations are sporadic; large gene size predisposes to high mutation rate - DMD largest
human gene
Types Duchenne muscular dystrophy Becker muscular dystrophy
- Caused due to Deletion of dystrophin gene - Caused due to mutation of
dystrophin gene

Musculoskeletal Page 4.1


dystrophin gene
- Presents as proximal muscle weakness at 1 year age; distal - Clinically milder disease
muscles involved later
• Calf pseudohypertrophy characteristic finding (filled
with fat)
- Death due to cardiac or respiratory failure; myocardium
commonly involved

- Elevated serum creatine kinase

Neuromuscular junction
Myasthenia gravis vs Lambert-Eaton syndrome
Myasthenia gravis Lambert-Eaton syndrome
Defn - Ab against postsynaptic Ach receptor at NM - Ab against presynaptic Ca channel at NM
junction junction
Pt popn - Commonly seen in women - Seen as paraneoplastic syndrome; commonly
- 15% of cases associated with thymic hyperplasia in small cell carcinoma of lung
or thymoma
Presentati - Muscle weakness that worsens with use and - Muscle weakness that improves with use and
on improves with rest worsens with rest
- Early eye involvement - ptosis and diplopia (MOST
MUSCULAR DISEASE SPARE EYES)
Treatment - Thymectomy improves symptoms - Resection of cancer resolves disease
- Use anti-cholinesterase agents to increase Ach in - Anti-cholinesterase don't help
synapse

Musculoskeletal Page 4.2


18.6 Soft Tissue Tumor

Lipoma - Benign tumor of adipose tissue


- Most common benign soft tissue tumor in adult

Liposarcoma - Malignant tumor of adipose tissue


- Most common malignant soft tissue tumor in adults
- Lipoblast is characteristic cell - immature fat cell with multiple fat vacuole that create
scalloped appearance of nucleus

Fig: Top left - arrow points to scalloped nucleus in liposacoma; bottom left -
scalloped definition; right - normal adipose tissue biopsy
Rhabdomyoma - Benign tumor of skeletal muscle
- Cardiac rhabdomyoma is associated with tuberous sclerosis
Rhabdomyosarc - Malignant tumor of skeletal muscle
oma - Most common soft tissue tumor in children
- Rhabdomyoblast is characterstic cell - desmin positive
- Most common site is eyes, head and neck; vagina is classic site in young girls

Musculoskeletal Page 5.1


Chapter 19: Skin Pathology
19.1 Inflammatory Dermatoses

1. Describe the histology of epidermis.


Layer Character
Stratum basalis Stem cell layer
Statum spinosum Appears spinous due to desmosomes between keratinocytes
Stratum granulosum See granules in keratinocytes
Stratum corneum See keratin and anucleate cells

INFLAMMATORY DERMATOSES
Atopic dermatitis and contact dermatitis
1. Differentiate atopic dermatitis and contact dermatitis.

Atopic dermatitis Contact dermatitis


Pruritic, erythematous, oozing rash with vesicles Same presentation
and edema

Skin pathology Page 1.1


and edema
- Type I hypersensitivity reaction; associated with Type IV hypersensitivity. Some causes:
asthma and allergic rhinitis - Poison ivy, nickel jewelry
- Usually start in face in infancy; seen mostly on - Irritant chemical (ex- detergent)
flexor surface afterwards - Drugs (ex- penicillin)
Treatment: topical glucocorticoid and
remove offending agent

Acne vulgaris
Presentation - Comedones (whitehead and blackheads), pustules, and nodules
- Very common in adolescents
Cause - Chronic inflammation of hair follicle and associated sebaceous gland
Pathophysiol - Propionibacterium acnes - infection produces lipases that break sebum to
ogy proinflammatory fatty acids which results in pustules or nodule
- Hormone mediated - androgens increase sebum and excess keratin production.
They block follicles and produce comedones
Treatment - Benzoyl peroxide (antimicrobial)
- Vit A derivatives (reduce keratin production) - ex- isotretinoin

Psoriasis
Presentation - Well-circumscribed salmon-colored plaques with silvery scale, usually on
extensor surface and scales
- Pitting of nails
Pathophysiolo - Excess keratinocyte proliferation
gy
Etiology - Autoimmune: associated with HLA - C
- Environmental trigger may be present: ex - trauma
Histology - Acanthosis- epidermal hyperplasia
- Parakeratosis - retention of nuclei in stratum corneum
- Munro microabscess - neutrophil collection in stratum corneum
- Auspitz sign - thinning of epidermis above elongated dermal paiillae - results in
bleeding when scale is picked off
Treatment - Corticosteroids
- UV-A light with psoralen
- Immune modulating therapy

Lichen planus
Presentation - Pruritic, purple papules with reticular white lines on surface (wickham striae)
- Commonly seen on wrists, elbow, and oral mucosa
Histology - Inflammation of dermal-epidermal junction with a saw tooth appearance
Etiology - Unknown
- Associated with Hep C infection

Skin pathology Page 1.2


19.2 Blistering Dermatoses

Pemphigus vulgaris Bullous pemphigoid


• IgG mediated attack to desmosomes • IgG mediated attack to hemidesmosome
• Cell separate somewhere in stratum • Cells separate in dermal-epidermal junction -
spinosum - vesicles rupture easily vesicles don't rupture easily
• Fish net immunofluorecence • Linear immunofluorescence
• Histology - tombsonte cells, acantholysis • Histology -
(suprabasal vesicle), Nikolky sign
• Oral mucosa involved Oral mucosa spared
Pemphigus vulgaris (pemphigus = watery blister)
Cause IgG mediated attack to desmosomes (desmoglein) (type II hypersensitivity)
between keratinocytes
Presentation - Skin and oral mucosa bullae
Acantholysis Seperation of stratum spinosum keratinocytes (connected by
desmosome) results in suprabasal bullae
Tombstone Basal cells are still attached to dermus and appear as
cells tombstones
Nikolky sign Bullae are thin walled and rupture easily making shallow
erosions with dried crust
- Potentially fatal
Immunofluorec Fish net immunofluorescence pattern- IgG surrounding keratinocytes
ence

Bullous Pemphigod (pemphigoid = similar to pemphigus)


Cause - IgG mediated attack to hemidesmosomes (BP180) between basal cell and
basement membrane
Presentaiton - Blisters on skin, usually in elderly (bullae don't rupture easily - contrast to
pemphigus vulgaris)
- Oral mucosa is spared
- Clinically milder than pemphigus vulgaris
Immunofluoresce Linear pattern
nce

Skin pathology Page 2.1


Immunofluoresce
nce

Dermatitis herpetiformis
Cause - IgA deposition at tip of dermal papillae
Presentation - Grouped pruritic papule, vesicles and bullae

Association Strong association with celiac disease


Treatment Gluten free diet

Erythema Multiforme, Stevens-Johnson syndrome, and Toxic epidermal necrolysis


Erythema Presentatio - Hypersensitivity reaction with targetoid rash (target with
multiforme n multiple rings) and bullae
- Targetoid rash is due to central epidermal necrosis
surrounded by erythema

Cause - HSV (most common)


- Mycoplasma
- Drugs (penicillin and sulfonamides)
- Autoimmune (SLE)
- Malignancy

Stevens-Johnson • Erythema multiforme,bullae,necrosis,sloughing of skin with fever and


syndrome (SJS) mucous membrane involvement
• Typically 2 mucous membrane involved - lips, eyes
• Mostly associated with adverse drug reaction
Toxic epidermal Presentati - SJS that occupies >30% of body surface with diffuse
necrolysis on sloughing of skin resembling large burn
- Separation of cells at dermal-epidermal junction (scalded

Skin pathology Page 2.2


- Separation of cells at dermal-epidermal junction (scalded
skin syndrome separates at stratum granulosum)

Cause - Most commonly due to adverse drug reaction

Skin pathology Page 2.3


19.3 Epithelial Tumors

Seborrheic keratosis
Presentation - Benign squamous proliferation; common in elderly
- Keratin filled cysts (horn cysts) - on extremities or face, coin like stuck on
appearance

Histology Keratin pseudocyst on histology


Leser-Trelat - Sudden onset multiple seborrheic keratoses
sign - Suggests underlying carcinoma of GI tract

Acanthosis nigricans
Presentation - Epidermal hyperplasia with symmetric darkening of skin (velvet like skin)
- Often presents in axilla, neck, groin

Association - Hyperinsulinemia - diabetes, obesity, Cushing


- Visceral carcinoma - ex - gastric carcinoma

Basal cell carcinoma vs squamous cell carcinoma

Skin pathology Page 3.1


Basal cell carcinoma vs squamous cell carcinoma
Basal cell carcinoma Squamous cell carcinoma
Definiti Malignant proliferation of basal cell (stem cell) Malignant proliferation of supra-basal
on of epidermis cell of epidermis - that’s why
presentation is crusty
Epidemi Most common HUMAN CANCER; locally 2nd most common skin cancer
ology invasive but rarely metastasize
• 75% of non-melanoma skin cancer
- Seen from 50s-70s
Risk Risk factors related to UVB exposure: - Risk factor related to UVB exposure
- Albinism - Others:
- Xeroderma pigmentosum (nucleotide • Immunosupressive therapy
excision repair damage) • Arsenic exposure
- Prolonged sun exposure • Chronic inflammation (burn
scar or draining sinus tract)
Progres - Has stepwise progression like colon
sion cancer.
- Actinic keratosis - precursor lesion of
SCC and presents as hyperkeratotic
scaly plaque (0.07% of AK progress to
SCC per year)
- In Xeroderma pigmentosum, you see
tons of actinic keratosis at very early
age

Present - Pearly and shiny - Crusty, nodular mass


ation - Dialated (telangiectatic) vessel - Classic location is lower lip
- Classic location is upper lip

Histolog - Nodules of basal cell with peripheral palisading • Keratin pearls


y

Skin pathology Page 3.2


Histolog
y

Treatm - Excision - Excision


ent - Metastasis is uncommon - Metastasis is uncommon
- Keratoacanthoma - well differentiated SCC that develops rapidly and regress spontaneously;
presents as cup shaped tumor filled with keratin debris

Skin pathology Page 3.3


19.4 Disorders of Pigmentation and Melanocytes

Melanocytes basic
Melanocyte location Basal layer
Embryogenic development From neural crest cells
Melanin production - Make melanin from tyrosine in melanosomes
- Pass melanosomes to keratinocytes

Vitiligo, albinism, freckle (ephelis), and mesalma


Vitiligo Albinism Freckle Mesalma
- Localized loss of - Mass like
skin hyperpigmentation of
pigmentation cheeks

- Due to - Congenital enzyme - Due to increased number - Associated with


autoimmune defect (tyrosinase) of of melanosomes pregnancy and oral
damage of melanin production (melanocytes number contraceptives
melanocytes pathway stay the same
- May involve eyes only
or eye and skin
- Increased risk for SCC,
basal cell carcinoma,
and melanoma (sun
damage)

Nevus
Definition - Benign neoplasm of melanocytes
Presentatio - Flat macule or raised papule
n - Symmetric with sharp borders, evenly distributed
Types - Junctional nevus - melanocyte at dermal-epidermal junction; most common in kids
- Compound (intradermal) nevus - melanocyte extension into dermis; most
common in adults
Prognosis - Most common acquired nevi appear in early childhood; evaluate lesions after 20
years age
- Nevi changes is normal. Ex - hormones, pregnancy
- Dysplasia is precursor to melanoma

Melanoma
Definition - Malignant neoplasm of melanocytes - often fatal
Epidemiology - Significant risk of metastasis (contrast to basal cell carcinoma and SCC of skin)

Skin pathology Page 4.1


Epidemiology - Significant risk of metastasis (contrast to basal cell carcinoma and SCC of skin)
Risk - UVB induced DNA damaged
- Dysplastic nevus syndrome (autosomal dominant disorder characterized by
formation of dysplastic nevi that may progress to melanoma)
• >50 nevi; usually appear around puberty and continue to develop beyond
40
- De novo melanoma is more common than dysplasia from nevus

Fig: dysplastic nevus syndrome


Presentation - S-100 tumor marker
- Mole with ABCDE-
• A = asymmetry
• B = borders are irregular
• C = color is non-uniform
• D = diameter>6mm
• E = evolution over time

Growth types - Radial - horizontal growth across epidermis; low risk for metastasis
- Vertical - growth to deep dermis
• Depth of extension (Breslow thickness) most important prognostic factor
for metastasis
Genetics • Often associated with BRAF V600E kinase mutation (55% of cases) (HY)
Treatment • Vemurafenib (BRAF kinase inhibitor) for BRAF +ve tumors
• Surgical excision
Variant of

Skin pathology Page 4.2


Variant of Superficial spreading - Most common subtype
melanoma - Dominant early radiant growth
- Good prognosis

Lentigo maligna melanoma - Lentiginous proliferation (radial growth)


- Good prognosis

Nodular - Early vertical growth


- Poor prognosis

Acral lentiginous - Arises on palm or soles, often in dark skin


individuals
- Acral = affecting limbs

Skin pathology Page 4.3


19.5 Infectious Disorders

Impetigo and cellulitis


Impetigo Cellulitis
Presentati - Erythematous macules that progress to - Red, tender, swollen rash with fever
on pustules; rupture of pustules causes
honey crusted skin

Cause - Superficial bacterial infection: Staph - Deeper dermal and subcutaneous


aureus, Strep pyogenes infection S aureus or S pyogenes
Epidemiol - Commonly affects children
ogy
- Risk for necrotizing fascitis due to
infection with anaerobic 'flesh eating'
bacteria (surgical emergency)

Staphylococcal scalded skin syndrome


Presentation - Sloughing of skin with red rash and fever

Pathophysiology - Exfoliative A and B toxin results in epidermolysis of stratum


granulosum
Difference from toxic - Skin separation in TEN occurs at dermal-epidermal junction;
epidermal necrolysis separation in scalded skin junction occurs in stratum granulosum

Molluscum contagiosum
Presentation - Firm, pink, umbilicated papules

Skin pathology Page 5.1


-

Cause - Poxvirus infection


Histology - Affected cells show molluscum bodies (cytoplasmic inclusion)
Classic pt - Kids
- Sexually active adults
- Immunecompromised individuals

Skin pathology Page 5.2


Dermatologic morphology

Skin pathology Page 6.1

Potrebbero piacerti anche